Download as pdf or txt
Download as pdf or txt
You are on page 1of 74

Practice

Test Explanations
Step 1 Practice Test # 17 – Block 1
Question Total: 50

Step 1 Practice Test #17 Explanations


Block 1

1. 70yo M dies in a motor vehicle collision. Was undergoing evaluation for occult blood in the stool. Photo of transverse
colon shown. Dx?
- Tubular adenoma

Why it’s right: !"#$!%&$'()*+!)'!,'-)+.!,/*&(!(#$!0*'(!1*00*+!1,&'$'!*2!*11&3(!/3**4!)+!,+!*34$5!6,()$+(7!811&3(!#$5$!


0$,+'!)(9'!*11&3(!*5!: '($,3(#;<=!,+4!;*&!1,+9(!'$$!)(!>)(#!(#$!+,-$4!$;$!?!,!2$1,3!*11&3(!/3**4!($'(!#,'!(*!/$!4*+$!(*!2)+4!
)(7!811&3(!/ 3**4!1*+(5,'('!>)(#!*@$5(!/3**4<!>#)1#!;*&!1,+!:*@$5(3;=!'$$!)+!(#$!'(**37!"#$!0*'(!1*00*+!1,&'$'!*2!*11&3(!
/3**4!)+!,+!*34$5!6,()$+(!,5$!4)@$5()1&3*')'!,+4!,+.)*4;'63,'),7!A*(#!*2!(#$'$!1,&'$'!,5$!5$3,()@$3;!/$+).+7!"#)'!%&$'()*+!
)'!0,4$!,!/)(!0*5$!4)22)1&3(!/$1,&'$!)(!>,+('!;*&!(*! )4$+()2;!,! 0,3).+,+1;!65$1&5'*57!"#)'!)'!(#$!5$,'*+!B8A"'!,+4!
1*3*+*'1*6)$'!,5$!&'$4!)+!'15$$+)+.!2*5!1*3*+!1,+1$5! ?!(#$!0*'(!1*00*+!CD!(&0*5!,+4!*+$!*2!(#$!0*'(!1*00*+!
1,+1$5'!)+!.$+$5,37!A $1,&'$!(#)'!6,()$+(!)'!EF!,+4!#,'!/3** 4!)+!#)'!'(**3<!(#$!0*'(!3)-$3;!0,3).+,+(!65$1&5'*5
!)'!,!(&/&3,5!
,4$+*0,<! ,!/$+).+!3$')*+!/&(!,!65$1&5'*5!(*!,4$+*1,51)+*0,!*2!(#$!1*3*+ 7!
!
Take home point:!"#$!0*'(!1*00*+!1,&'$'!*2!/$+).+<!*11&3(!CD!/3$$4)+.!)+!*34$5!6,()$+('!,5$!4)@$5()1&3)()'!,+4!
,+.)* 4;'63,'),G!(#$!0*'(!1*00*+!1,&'$!*2! 65$H0,3).+,+(<!*11&3(!CD!/3$$4)+.!)'!,!(&/&3,5!,4$+*0,7! !

2. 38yo M with 1-week hx of watery, itchy eyes and a runny nose. Physical shows inflamed nasal mucosa. No congestion
in lower lung. Pharmacotherapy?
- Loratadine

Why it’s right: "#$!4),.+*')'!)'!#,;!2$@$57!"#$!*+3;!$I(5,!#)'(*5;!(#,(!)'!0)'')+.!)'!(#$!()0$!*2!;$,5!?!'65)+.!*5!2,33!>#$+!
'$,'*+,3!,33$5.)$'!,5$!,(!(#$)5!6$,-7!J@$+!)2!(#$!4),.+*')'!'$$0'!3)-$!,!1*00*+!1*34<!(#$5$!)'!+*!1*+.$ '()*+!*5!3*>$5!3&+.!
)+@*3@$ 0$+( 7!"#$!(5$,(0$+(!*2!1#*)1$!)'!,!#)'(,0)+$!,+(,.*+)'(!')+1$!(#$!1#$0)1,3!0$4),(*5!*2!#,;!2$@$5K'$,'*+,3!
,33$5.)$'!)'!#)'(,0)+$7!L*5,(,4)+$!)'!,!#)'(,0)+$!,+(,.*+)'( 7!!
!
Take home point:!"5$,(0$+(!2*5!#,;!2$@$5!)'!#)'(,0)+$!)+#)/)()*+!>)(#! ,! #)'(,0)+$!,+(, .*+)'(!'&1#!,'!3*5,(,4)+$7!!

3. 16yo girl with 3-day hx of fever, nonproductive cough, and fatigue. T 38.3, P 88/min, BP 102-70. PE shows pale
conjunctivae. CXR shows bilateral interstitial infiltrates. Blood spontaneously agglutinates while awaiting transport to the
laboratory. Antibody isotypes causing agglutination?
- IgM

Why it’s right: "#$!4),.+*')'!)'!)+2$1()*&'!0*+*+&13$*')' 7!"#$!5$,3!MNOLJ!>)33!+*(!.)@$!'&1#!,+!*/@)*&'!,.$!


?!,!($$+,.$5<!
>#)1#!)'!,!1*00*+!,.$!2*5!1*+(5,1()*+!/$1,&'$!0*+*!)'!,3'*!-+* >+!,'!-)'')+.!4)'$,'$7!N;06(*0'!)+13&4$!.$+$5,3!23& H
3)-$!';06(*0'! ?!2$@$5<!1*&.#<!2,().&$7!"#$!&3()0,($!13&$!#$5$!>*&34!/$!)2!(#$;!)+13&4$4!,+!$+3,5.$4!'63$$+!*+!6#;')
$I,07!N*0$()0$'<!+*(!,3>,;'<!0*+*!1,+!1,&'$!,!#$0*3;()1!,+$0),!4&$!(*!(#$!1)51&3,()+.!)00&+$!1*063$I$'!D.O!

2 ©Test Pirates, LLC. All Rights Reserved.


Step 1 Practice Test #17 Explanations

,.,)+'(!(#$!@)5&'!(#,(!(#$+!15*''!5$,1(!>)(#!(#$!6,()$+(9'!/3**4!1$33'<!(5)..$5)+.!,..3&()+,()*+!,+4!#$0*3;')'7!B*5!(#)'!
5$,'*+<!(#$!6,()$+(!#,'!6,3$!1*+P&+1()@,<!,!($33
H(,3$!').+!*2!,+$0),7!!
!
Take home point:!D+2$1()*&'!0*+*+&13$*')' !1,+!1,&'$!,!#$0*3;()1!, +$0),!(#5*&.#!D.OH,1()@,($4!#$0*3;()1!
,..3&()+,()*+7!
!

4. 24yo M with small tender blisters on his penis 3 days after unprotected sex. Photograph shown. Causal agent?
- Herpes simplex virus type 2

Why it’s right: B*5!'$I&,33;!(5,+'0)(($4!)+2$1()*+'<!(#$!2)5'(!%&$'()*+!(#,(!'#*&34!/$!,'-$4!)'<!6,)+!*5!6,)+3$''Q!"#,(!


+,55*>'!(#$!4),.+*')'!4*>+!/;!RFS7!T!'$1*+4!%&$'()*+!(*!,'-!)'<!0&3()63$!*5!')+.3$!3$')*+'Q!"#,(!+,55*>'!)(!4*>+!/;!
,+*(#$5!RFS7!D2!>$!,663;!(#)'!0$(#*4!(*!(#)'!%&$'()*+< !>$!-+*>!(#)'!)'!,!6,)+2&3!3$')*+<!'*!(#$!(*6!*2!(#$!4)22$5$+(),3!)'!
#$56$'!,+4!1#,+15*)47!"#)'!6,()$+(!#,'!0&3()63$!3$')*+'!U'0,33!($+4$5!/3)'($5'V7!W$56$'!,66$,5'!,'!0&3()63$!/3)'($5'!>#)3$
1#,+15*)4!.$+$5,33;!65$'$+('!,'!,!')+.3$!3$')*+7!D2!(#$!3$')*+!>
,'!6,)+3$''<!(#,(!>*&34!+,55*>!(#$!4)22$5$+(),3!(*!';6#)3)'!
,+4!.$+)(,3!>,5('7!T+4!';6#)3)'!)'!,!')+.3$!3$')*+!>#)3$!.$+)(,3!>,5('!65$'$+(!,'!0&3()63$!3$')*+'7!
!
!
Take home point:!WNXY!)'!,!6,)+2&3<!0&3()63$ H3$')*+!4)'$,'$!65$'$+()+.!,'!'0,33<!($+4$5!/3)'($5'!)+!13&'($5'7!
!

5. 42yo F with 3-year hx of an intermittent facial rash, including the forehead, eyelids, nose, and cheeks. Rash seems to
be getting worse since she moved from New York to Florida last year. Spicy foods precipitate a flushing reaction that
seems to exacerbate the rash. PE shows erythema over the nose and cheeks, with scattered telangiectasias and a few
papules. Dx?
- Rosacea

Why it’s right: W$5$!>$!#,@$!,!6,()$+(!>)(#!,!1#5*+)1!5,'#!UZ H;$,5 H#)'(*5;V!(#,(!65$'$+('!*+!(#$!2,1$<!'6$1)2)1,33;
<!(#$!
2*5$#$,4<!$;$3)4'<!+*'$!,+4!1#$$-'7!"#$!0,)+!4),.+*'$'!2*5!,!1#5*+)1<!2,1),3!5,'#!U>#)1#!,5$!,3'*!#).# H;)$34!2*5!(#$!
6&56*'$'!*2!(#$!MNOLJV!)+13&4$! 3&6&'<!4$50,(*0;*')()'<!$1[$0,<!,+4!5*',1$,7!L&6&'!)'!,!5,'#!(#,(!5$'6$1('!(#$!
+,'*3,/),3!2*34'<!>#)3$ !4$50,(*0;*')()'!15*''$'!(#$!+,'*3,/),3!2*34'!U*+$!>,;!(*!4)'()+.&)'#!/$(>$$+!(#$!(>*V7!J1[$0,!
,+4!5*',1$,!,5$!5,'#$'!/*(#!)+23&$+1$4!/;!(#$!$+@)5*+0$+(7! J1[$0,!)'!.$+$5,33;!>*5'$!)+!1*34<!45;!13)0,($'7!\*',1$,!)'!
(;6)1,33;!,3>,;'!>*5'$ !)+!>,50!13)0,($'<!$'6$1),33;!,2($5!'&+!$I6*'&5$ 7!D
+!1*+(5,'(<!6'*5),')'!U3$''!1*00*+3;!*+!(#$!2,1$V!
)065*@$'!>)(#!'&+KMX!(#$5,6;<!>#)1#!)'!>#;!*+$!( #$!(5$,(0$+('!*2!6'*5),')'!)'!&3(5,@)*3$(!3).#(!(#$5,6;7!!
!
"#$!-$;!';06(*0 !)'!(#,(!'6)1;!2**4'!$I,1$5/,($!(#$!5,'#7!N6)1;!2**4!1,+!0,-$!,!3*(!*2!'-)+!1*+4)()*+'!>*5'$!U)+13&4)+.!
,&(*)00&+$!'-)+!1*+4)()*+'V7!B*5!$I,0!6&56*'$'<!'6)1;!2**4!]!5,'#!^!5*',1$,7! T3'*<!,31*#*3!]!5,'#!^!5*',1$,7!T+4!2)+,33;< !
($3,+.)$1(,'),'! >)(#)+!(#$!5,'# !]!2,1),3!5,'#!^!5*',1$,7!!
!
Take home point:!_#5*+)1!2,1),3!5,'#!]!($3,+.)$1(,'),'!]!'6)1;!2**4!]!,31*#*3!^!5*',1$,<!&+()3!65*@$+!*(#$5>)'$7! !!

6. 53yo M returned from Africa, has fever, headache, and abdominal discomfort. Received appropriate vaccinations prior
to the trip. T 39.4C. A wright-stained peripheral smear shown. Dx?
- Malaria

Why it’s right: 8+3;!&+()3!5$1$+(3;!UYF`RV!,!@,11)+$!
>,'! 4$@$3*6$4!2*5!0,3,5),<!,+4!)(!)'!*+3;!RF
HaFS!$22$1()@$7!D(!)'!+*(!
>)4$3;!&'$4!2*5!(#)'!5$,'*+7!b$2$+4)+.!,.,)+'(!0,3,5),!)'!1#,33$+.)+.!/$1,&'$!+*(!*+3;!,5$!(#$5$!2)@
$!4)22$5$+(!'6$1)$'!*2!
0,3,5),<!(#$!3)2$!1;13$!*2!(#)'!
65*(*[*,3 !6,5,')($!)'!1*063$I7!!

3 ©Test Pirates, LLC. All Rights Reserved.


Step 1 Practice Test #17 Explanations

!
"#$!2,1(!(#,(!(#)'!6,()$+(!>,'!@,11)+,($4!)'!0$,+(!(*!0,-$!)(! 0*5$!1#,33$+.)+.!,+4!6*'')/3;!(5)1-!(#$!($'(H(,-$5!?!)2!(#$!
6,()$+(!>,'!@,11)+,($4!2*5!0,3,5),<!>*+9(!#$!/ $!)00&+$!(*!)(Q!c+*>)+.!(#$!2,1('!,/*&(!0,3,5),!@,11)+$!)'!-$;!(*!+*(!
.$(()+.!(5)1-$4!/;!(#)'!%&$'()*+7!B&5(#$5<!(
#$!';06(*0'!,+4!2)+4)+.'!,5$!#).#3;!'6$1)2)1!2*5!0,3,5),!)+!,!6,()$+(!>#*!
5$1$+(3;!(5,@$3$4!(*!T25)1,7!O,3,5),!65$'$+('!>)(#!2$@$5!,+4!#$,4,1# $!,+4!1,+!,3'*!1,&'$!,+$0),!,+4!'63$+*0$.,3;<!
5$'&3()+.!)+!,/4*0)+,3!6,)+7! !
!
"#$!4),.+*')'!)'!0,4$!/;!/3**4!'0$,5<! 4$0*+'(5,()+. !$)(#$5!(#$!(5*6#*[*)($!5)+.!2*50!*2!(#$!65*(*[*,!)+')4$!(#$!\A_!*5!
,+!\A_!'1#)[*+(!>)(#!0$5*[*)($'7!d#$+!,!0*'%&)(*!1,55;)+.!0, 3,5),!/)($'!,!#&0,+<!)('!',3)@,!(5,+'2$5'!(#$!'6*5*[*)($!
2*50!*2!0,3,5),7!"#$!'6*5*[*)($'!.*!4)5$1(3;!(*!(#$!3)@$5<!,+4!>)(#)+!#$6,()1!1$33'<!1#,+.$!(*!(#$!(5*6#*[*)($!2*50!,+4!
/$.)+!(*!5$65*4&1$!,'$I&,33;7!\$65*4&1()*+!3$,4'!(*!0$5*[*)($'!U1*33$1()@$3;!-+ *>+!,'!,!'1#)[*+(V<!>#)1#!,5$!
63,'0*4)&0!4,&.#($5!1$33'!(#,(!'65$,4!)+2$1()*+7! B)+,33;<!(#$!#;6+*[*)($!)'!,+!)06*5(,+(!'(,.$!(*!,3'*!/$!,>,5$!*2!
/$1,&'$!)(!)'!(#$!4*50,+(!2*50!*2!(#$!63,'0*4)&0!(#,(!1,+!5$')4$!)+!(#$!3)@$5!,+4!3$,4!(*!5$3,6')+.!,+4!5$1&55) +.!
)+2$1()*+!$@$+!,2($5!(5$,(0$+(!>)(#!1#3*5*%&)+$7!e5)0,%&)+$!+$$4'!(*!/$!,44$4!(*!(5$,(!(#$!#;6+*[*)($!2*507! c+*>)+.!
(#$'$!3)((3$!4$(,)3'!,/*&(!,!25$%&$+(3;
H($'($4!)+2$1()*+!>)33!$,5+!;*&!$I(5,!6*)+('!
*+!(#$!5$,3!MNOLJf !!
!
Take home point:!O,3,5),!65$'$+('!>)(#!').+'!,+4!';06(*0'!*2!#$,4,1#$<!2$@$5<!,+$0),<!,+4!'63$+*0$.,3;<!>#)1#!1,+!
1,&'$!,/4*0)+,3!6,)+<!,2($5!5$1$+(!(5,@$3!(*!0,3,5), H$+4$0)1!1*&+(5)$'!)+!T25)1,7!!

7. 68yo F with T2DM and hypertension that has been poorly controlled despite hydrochlorothiazide (HCTZ) treatment. BP
150/96, Labs show serum glucose concentration of 130 and proteinuria. In addition to current Rx, which is most
appropriate pharmacotherapy?
- Lisinopril

Why it’s right: "#$5$!,5$!,!2$>!+&0/$5'!(*!/$!,>,5$!*2!#$5$7 g*50,3!/3**4! 65$''&5$!)'!3$''!(#,+!*5!$%&,3!(*!`YFKhF7!"#$!
3,($'(!T0$5)1,+!W$,5(!T''*1),()*+!.&)4$3)+$'!5$1*00$+4!(5$,(0$+(!2*5!#).#!/3**4!65$''&5$!,/*@$!`ZFKhF!U65)*5!(*!
YF`E<!)(!>,'!,/*@$!`iFKjFV7!B)5'(H3)+$!(5$,(0$+(!2*5!#).#!/3**4!65$''&5$!)'!'()33!,!(#),[)4$!4)&5$
()17!D+!6,()$+('!
>)(#!
1*+1&55$+(!4),/$($'<!,+!,1$H)+#)/)(*5!#,'!/$$+!'#*>+!(*!/$!5$+,33;H65*($1()@$ <!,+4!'#*&34!,1(&,33;!/$!2)5'(H3)+$!
(5$,(0$+(!2*5!#;6$5($+')*+7!"#$5$2*5$<!)+!(#)'!6,()$+(<!>#*!#,'!&+1*+(5*33$4!/3**4!65$''&5$!*+!W_"k! 63&'!4),/$($'!,+4!
)'!)+!+$$4!*2!
/$(($5!/3**4!65$''&5$!1*+(5*3<!3
)')+*65)3!U,+!,1$
H)+#)/)(*5V!>*&34!/$!(#$!0*'(!,665*65),($!0$4)1,()*+!(*!&'$7!
"#$!%&$'()*+!$@$+!#)+('!,(!(#)'!0$4)1,()*+!,'!(#$!/$'(!1#*)1$!/$1,&'$!(#$!6,()$+(!#,'!65*($ )+&5),<!,!').+!*2!-)4+$;!
4,0,.$ !,+4!,+!)+4)1,()*+!2*5!,!5$+,33;
H65*($1()@$!0$4)1,()*+7! !
!
Take home point:!T+!T_JH)+#)/)(*5!'&1#!,'!3)')+*65)3!)'!5$+,33;H65*($1()@$!,+4!'#*&34!/$!2)5'(H3)+$<!#;6$5($+')*+!
(5$,(0$+(!2*5!6,()$+('!>)(#!/*(#!4),/$($'!,+4!#;6$5($+')*+!U@$5'&'!W_"k<!>#)1#!)'!2)5'( H3)+$!(5$,
(0$+(!)+!6,()$+('!>)(#!
#;6$5($+')*+!>)(#*&(!4),/$($'V7!! !!
!

8. 66yo M with stage IV colon cancer with 3-day hx of severe diarrhea after receiving chemotherapy with fluorouracil,
leucovorin, and irinotecan. Prescribed opioid antidiarrheal agent with no CNS effects. Which med?
- Loperamide

Why it’s right: ">*!0$4)1,()*+'!(#,(! ,5$!&'$4!,'!,+() H4),55#$,3!0$4)1,()*+'!)+13&4$!3*6$5,0)4$!U(#$!,+'>$5V!,+4
!
/)'0&(#!*5!'&15,32,($7!"#$!0$1#,+)'0!*2!3*6$5,0)4$!)'!,.*+)'0!*2!(#$!0&H*6)*)4!5$1$6(*5!)+!(#$!0;$+($5)1!63$I&' !*2!
(#$!3,5.$!)+($'()+$<!5$'&3()+.!)+!,+()
H0&'1,5)+)1!,1()@)(;!,+4!4$15$,'$4!CD!0*()3)(;7!"#$5$2*5$<!)(!)'!&'$4!(*!(5$,(!4),55#$,7!
D(!4*$'!+*(!15*''!(#$!/3**4!/5,)+!/,55)$5!/$1,&'$!eH.3;1*65*($)+!)+!(#$!)+($'()+,3!>,33!$223&I$'!(#$!0$4)1,()*+!/,1-!)+(*!
(#$!CD!(5,1(7!D(!,3'*!6,''$'!(#5*&.#!(#$!3)@$5!2)5'(!/$2*5$!6*($+(),33;!(5,@$33)+.!(*!(#$!/5,)+7!O$(*13*65,0)4$!)'!,!

4 ©Test Pirates, LLC. All Rights Reserved.


Step 1 Practice Test #17 Explanations

0$4)1,()*+!&'$4!(*!(5$,(!+,&'$,!,+4!@*0)()+.!/$1,&'$!*2!)('!,+() H4*6,0)+$5.)1!$22$1('!)+!(#$!CD!(5,1(<!/&(!1,+!,3'*!,22$1(!
(#$!_gN!,+4!1,&'$!6,5-)+'*+)'0!25*0!4*6,0)+$!,+(,.*+)'07! !
!
Take home point:!L*6$5,0)4$!)'!&'$4!(*!(5$,(!4),55#$,!/$1,&'$!)(!)'!,+!*6)*)4!,.*+)'(!,+4!,1('!,'!,! 0&'1,5)+)1!
,+(,.*+)'(! )+!(#$!CD!(5,1(7!D(!.$+$5,33;!4*$'!g8"!15*''!(#$!/3**4!/5,)+!/,55)$5!/$1,&'$!)(!)'!$223&I$4!)+
!(#$!.&(!,+4!0,-$'!
,!2)5'(!6,''!(#5*&.#!(#$!3)@$57!!
!!
!

9. 35yo M in ED with 2-hour hx of sever fatigue and dizziness. Had profuse, watery diarrhea for 8 hours despite a lack of
oral intake. Recently returned from a medical relief trip to a remove village in Honduras. T 36.7 C, P 122/min, BP 90/50.
PE shows dry skin and decreased capillary refill. Stool for occult blood is negative; stool is gray and turbid. Gram stain
shows gram-negative, comma-shaped bacteria; no erythrocytes of leukocytes. MOA of toxin?
- Activation of adenylyl cyclase

Why it’s right: "#$!($'( H0,-$5'!3*@$!(#$! 1#*3$5,!%&$'()*+7!"#$!4),.+*')'!#$5$<!)2!;*&!#,@$+9(!,35$,4;!.&$''$4<!)'!4),55#$,!
4&$!(*!_#*3$5,!U,3' *!-+*>+!,'!5)1$ H>,($5!4),55#$,V<!*2!X)/5)*!1#*3$5,$7!
"#$!0$1#,+)'0!*2!,1()*+!*2!( #)'!.5,0 H+$.,()@$!
/,1($5),!)'!(*I)+!5$3$,'$7!"#$!(*I)+!6$50,+$+(3;!,1()@,($'!,!C H1*&63$4!65*($)+!5$1$6(*5<!C
' <!>#)1#!,1()@,($'!$+[;0$!
,4$+;3;3!1;13,'$<! 1*+@$5()+.!TOe!(*!1TOe7!T'!'&1#<!(#$!,+'>$5!1#*)1$'!1*&34!)+13&4$!,+;!*2!(#$!2*33*>)+.!)+!(#,(!1#,)+!
*2!$@$+('!/$1,&'$!,33!)+15$,'$!)+!,1()@)(;<!)+15$,')+.!1TOe7!_#*3$5,!)'!+*(!(#$!*+3;!1*00,H'#,6$4<!.5,0 H+$.,()@$!
/,1($5),!?!'* !)'!_,06;3*/,1($5!P$P&+) 7!_#*3$5,!,3'*!#,'!,!3*+.H23,.$33&0!*+!*+$!$+4<!.5*>'!)+!,3-,3)+$!0,($5),3<!,+4!)'!
*I)4,'$!6*')()@$7!!
!
Take home point:!"#$!0$1#,+)'0!*2!,1()*+!*2!_#*3$5,!)'!(*I)+!5$3$,'$!(#,(!6$50,+$+(3;!,1()@,($'!C ' <!,1()@,()+.!$+[;0$!
,4$+;3;3!1;13,'$<!>#)1#!1*+@$5('!TOe! ! !1TOe7!!
!

10. 59yo F with gradual onset of lack of muscle control in her left arm and leg. Sx 1 mo ago after dx with metastatic
breast cancer. PE shows ataxia of left upper and lower extremities. Muscle strength, DTR, sensation, proprioception
normal. Metastatic tumor in which location?
- Cerebellum

Why it’s right: T(,I ),!)'!(#$!-$;!>*54!#$5$7!8+!(#$ !5$,3!$I,0 <!(#$;!>)33!65*/,/3;!4$'15)/$!,(,I),!)+'($,4!*2!',;)+.<!
:,(,I),<=!/$1,&'$!(#$;!>,+(!;*&!(*!-+*>!>#,(!,(,I),!0$,+'!,+4!+*(!P&'(!0,(1#!)(!>)(#!1$5$/$33,5!4;'2&+1()*+7! T(,I),!)'!
3*''!*2!1**54)+,()*+7!T!6,()$+(!>)(#!,(,I),!*2!*+$!3$.!>)33!#,@$!(5*&/3$!>,3-)+.! ,+4!/,3,+1)+.7!"#$!',0$!.*$'!2*5!,(,I),!*2!
,+!,50! ?!(5*&/3$!>)(#!#,+43)+.!)($0'!*5!1**54)+,()+.!,50!6*')()*+!>)(#!(#$!2$33*>!,50 7!T3'*<!>$!-+*>!(#$!3$')*+!)'!
#).#$5!&6!)+!(#$!/5,)+'($0K/5,)+!,+4!+*(!)+!(#$!'6)+,3!1*54!/$1,&'$!/*(#!&66$5!,+4!3*>$5!3)0/'!*+! *+$!')4$!,5$!
$22$1($47!e5*65)*1$6()*+!)'!13*'$3;!5$3,($4!(*!1**54)+,()*+<!/&(!)'!4)'()+1(3;!4)22$5$+(!)+!(#,(!)(!0$,+'!/$)+.!,>,5$!*2!,!
/*4;!6,5(K3)0/!)+!()0$!,+4!'6,1$7!T!1$5$/$33,5!3$')*+!1,+!,3'*!1,&'$!,+!)+($+')*+!(5$0*57!"#)'!%&$'()*+!>*&34!/$!0,4$!
0* 5$!1#,33$+.)+.!)2!(#$;!,'-$4!>#$5$!)+!(#$!1$5$/$33&0!(#$!3$')*+!)'! ?!)(!)'!+*(!(#$!@$50)'<!*5!1$+($5!*2!(#$!1$5$/$33&0!
/$1,&'$!(#,(!5$'&3('!)+!(5&+1,3!,(,I),!,+4!4;',5(#5),7!_$5$/$33,5!3$')*+'!,5$!)6')3,($5,3!(*!(#$)5!4$2$1('<!'*!)+!(#)'!1,'$!(#$!
3$')*+!>*&34!/$!)+!(#$!3$2(!1$5$/$33,5!#$0)'6#$5$7! !
!
Take home point:!_$5$/$33,5!#$0)'6#$5$!3$')*+'!5$'&3(!)+!)6')3,($5,3!4$2$1('!)+!0&'13$!1**54)+,()*+K1*+(5*3<!*(#$5>)'$
-+*>+!,'!,(,I),<!,'!>$33!,'!,+!)+($+()*+!(5$0*57!_$5$/$33,5!@$50)'!3$')*+'!1,&'$!(5&+1,3!,(,I ),!,+4!4;',5(#5),7!!
!

5 ©Test Pirates, LLC. All Rights Reserved.


Step 1 Practice Test #17 Explanations

11. Newborn delivered at 38 weeks' gestation weighs 1800 g. PE shows petechial rash, microcephaly, and
hepatosplenomegaly. Serologic test for CMV: IgG + in mother, + in newborn; IgM - in mother, + in newborn. Explanation?
- Congenital cytomegalovirus infection

Why it’s right: \,'#<!0)15*1$6#,3;!,+4!#$6,(*'63$+*0$.,3;!,5$!+*+'6$1)2)1!').+'!*2!(#$!"*\_W$N!)+2$1()*+'7!"#$!


6$($1#),3!5,'#!)'!0*5$!'6$1)2)1!,+4!,3'*!-+*>+!,'!(#$!/3&$/$55;!0&22)+!5,'#7!A3&$/$55;!0&22)+!5,'#!1,+!/$!'$$+!)+!/*(#
!
_OX!,+4!5&/$33,7! "#$!0*(#$5!)'!3)-$3;!+$.,()@$!2*5!D.O!/$1,&'$!'#$!#,4!(#$!)+2$1()*+!2)5'(!,+4!13$,5$4!)(!$,53)$5<!'*!D.O
4$15$,'$4!,+4!D.C!5$0,)+'7!_OX!)+2$1()*+!)+!)00&+$!1*06$($+(!6,()$+('!U*5!+*+ H2$(&'$'V!)'!&'&,33;!,';06(*0,()1<!/&(!
)2!';06(*0'!65$'$ +(<!(#$;!,5$!(;6)1,33;!')0)3,5!(*!0*+*+&13$*')'7!_OX!)'!,1(&,33;!(#$!0*'(!1*00*+!"8\_W!)+2$1()*+7!D(!
1,+!,3'*!1,&'$!P,&+4)1$!,+4!0$+(,3!5$(,54,()*+7! !
!
Take home point:!_OX!)+2$1()*+!)+!,!+$>/*5+!0,;!65$'$+(!,'!,!6$($1#),3!5,'#!U:/3&$/$55;!0&22)+!5,'#=V<!0) 15*1$6#,3;<!
#$6,(*'63$+*0$.,3;<!0$+(,3!5$(,54,()*+<!,+4!P,&+4)1$7! !

12. Female newborn is delivered at 38 weeks' gestation. Apgar 8 and 8 at 1/5 min. PE shows a bulging, fluid-filled mass
approximately 5 cm in diameter in the midline over the lumbosacral region. No spontaneous movements of the lower
extremities. Abnormality most likely occurred because of abnormal development during which period post-conception (in
days)?
- 15 to 40

Why it’s right: "#)' !+$>/*5+!#,'!,!+$&5,3!(&/$!4$2$1(<!$)(#$5!0$+)+.*1$3$!
U>#)1#!)'!#$5+),()*+!*2!P&'(!(#$!0$+)+.$'!
(#5*&.#!,!/*+$;!'6)+$!4$2$1(V!*5!0$+)+.*0;$3*1$3$!U#$5+),()*+!*2!/*(#!(#$!0$+)+.$'!,+4!(#$!'6)+,3!1*54V!/,'$4!*+!(#$!
2)+4)+.!*2!,!23&)4
H2)33$4!0,''!*@$5!(#$!3&0/*',15,3!5$.)*+7!A$1,&'$!(#$!6,()$+(!4*$'!+*(!#,@$!0*@$0$+(!*2!(#$!3*>$5!
$I(5$0)()$'<!0*5$!3)-$3;!(#$!4),.+*')'!)'!0$+)+.*0;$3*1$3$!1,&')+.!'6)+,3!1*54!4,0,.$!,+4!3*>$5!$I(5$0)(;!6,5,3;')'7!
g$&5,3!(&/$!4$2$1('!,5$!4&$!(*!+$&5*6*5$!2&')*+!2,)3&5$7!"#$!+$&5*6*5$'!2&'$!/;!>$$-!i<!*5!Yh!4,;'!6*'( H1*+1$6()*+7!
"#$5$2*5$<!(#$!,/+*50,3)(;!0*'(!3)-$3;!*11&55$4!/$(>$$+!4,;!`R!(*!iF7! !
!
Take home point:!g$&5,3!(&/$!4$2$1('! 5$'&3(!25*0!,+!)+
H&($5*!)+'&3(!(#,(!*11&5'!,/*&(!`R!(*!iF!4,;'!,2($5!1*+1$6()*+!
/$1,&'$!(#$!+$&5*6*5$'!2&'$!,5*&+4!(#$!i(#$!>$$-!6*'(H1*+1$6()*+7! !
!

13. 64yo M in ED 3 hours after SOB with exertion and extreme fatigue. Has ischemic heart disease. P 125/min, BP
105/60. ECG shows atrial fibrillation. Intravenous ibutilide is administered. Ten minutes later, ECG shows normal sinus
rhythm. Risk for which drug effect in the next 6 hours?
- Torsades de pointes

Why it’s right: "#$5$!,5$!,!2$>!45&.!1,($.*5)$'!(#,(!,5$!#).# H;)$34<!,+4!,33!(#$!45&.'
!>)(#)+!(#$'$!1,($.*5)$'<!(#$)5!O8T'<!
)+4)1,()*+'<!,+4!')4$!$22$1('!'#*&34!/$!0$0*5)[$47 "#$'$! 1,($.*5)$'! )+13&4$!
1,54)*<!+$&5*<!#$0$K*+1<!,+4!(#$!45&.'!)+!
(#$!/,')1!'1)$+1$!6#,50,1*3*.;!'$1()*+! ?!)7$7!1#*3)+*0)0$()1!,.$+('<!';06,(#*0)0$()1'7!T33 !1,54)*!45&.'!,5$!#).#H;)$34<!
)+13&4)+.!(#$!,+(),55#;(#0)1!45&.'<!13,''!D!(#5*&.#!DX7!D/&()3)4$!)'!,!13,''!DDD!,+(),55#;(#0)1!,3
*+.!>)(#!,0)*4,5*+$<!
'*(,3*3<!,+4!4*2$()3)4$7!"#$'$!45&.'!>*5-!*+!(#$!(#)54!6#,'$!*2!(#$!1,54),1!1;13$!>#$+!6*(,'')&0!)'!3$,@)+.!(#$!1$337!
N6$1)2)1,33;<!6*(,'')&0!5$3$,'$!)'!'3*>$4<!)+15$,')+.!(#$!,1()*+!6*($+(),3<!(#$!
$22$1()@$!5$25,1(*5;!6$5)*4<!,+4!65*3*+.)+.!
(#$!l"!)+($5@,37!T33!*2!(#$'$!$22$1('!,5$!0$,+(!(*!'($,4;!(#$!#$,5(!5,($!,+4!'(*6!(#$!,55#;(#0),7!W*>$@$5<!l"!
65*3*+.,()*+!)'!,!5)'-!2,1(*5!2*5!(*5',4$'!4$!6*)+($'<!>#)1#!1,+!*11&5!$@$+!,2($5!+*50,3!')+&'!5#;(#0!)'!*/(,)+$47!"#)'!
%&$'()*+!1*&34!/$!0 ,4$!0*5$!4)22)1&3(!/;!'#*>)+.!,+!J_C!*2!(*5',4$'!,2($5!,40)+)'(5,()*+!*2!)/&()3)4$!,+4!,'-$4!>#,(!)'!
(#$!+$I(!/$'(!'($6<!*5!(#$!(5$,(0$+(Q!T+'>$5m!O.Y]7! !
!
6 ©Test Pirates, LLC. All Rights Reserved.
Step 1 Practice Test #17 Explanations

Take home point:!D/&()3)4$!)'!,!13,''!DDD!,+(),55#;(#0)1!45&.!(#,(!>*5-'!*+!6#,'$!DDD!*2!(#$!1,54),1!


1$33!1;13$<!65*3*+.)+.!
(#$!l"!)+($5@,3<!>#)1#!)+15$,'$'!(#$!5)'-!2*5!(*5',4$'!4$!6*)+($'7!
!
!

14. 65yo F with 20-year hx of osteoarthritis of the hands now has pain radiating down the distal anterior thigh, knee,
medial leg, and foot. Bony outgrowth of vertebrae compressing one of the spinal nerves is suspected. Nerve root in which
intervertebral foramina is effected?
- L-3 to 4

Why it’s right: "#)' !6,()$+(!65$'$+('!>)(#!6,)+!)+!(#$!,+($5)*5!,+4!0$4),3!1*06,5(0$+('!25*0!(#$!-+$$!4*>+!(*!(#$!2**(7!
"#$!+$5@$'! (#,(!1*&34!/$!,22$1($4!#$5$!)+13&4$!(#$!2$0*5,3!ULY HLiV!,+4!(#$!1*00*+!6$5*+$,3!ULi HNYV7!"#$!:/$'(=
!,+'>$5!
>*&34!(#$+!/$!LZ HLi7!LY!0,;!,3'*!/$!,22$1($4<!/&(!(#,(!,+'>$5!1#*)1$!,3'*!)+13&4$'!L`<!,+4!4*$'+9(!)+13&4$!(#$!$%&,33;!
)06*5(,+(!LZ!,+4!Li!1#*)1$'7"! #$!2$0*5,3!+$5@$!1*+(5*3'!(#).#!23$I)*+!,+4!3$.!$I($+')*+!>#)3$!'&663;)+.!'$+',()*+!(*!(#$!
,+($5)*5!(#).#!,+4!0$4),3!3$.7!"#$!1*00*+!6$5*+$,3!1*+(5*3'!2**(!$@$5')*+<!4*5')23$I)*+<!,+4!(*$!$I($+')*+!>#)3$!
65*@)4)+.!'$+',()*+!(*!(#$!,+($5*3,($5,3!3$.!,+4!4*5 ',3!,'6$1(!*2!(#$!2**(!U(#$!63,+(,5!*5!'*3$!*2!(#$!2**(!5$1$)@$'!'$+'*5;!
+$5@$!2)/$5'!25*0!(#$!()/),3!+$5@$!ULi HNZV7!
!
!
Take home point:!e,)+!(#,(!'(,5('!,(!(#$!(#).#!,+4!5,4),($'!4*>+!(#$!-+$$<!0$4),33;!*+!(#$!3$.!,+4!2**(!)'!4&$!(*!+$5@$!
1*065$'')*+!,(!(# $!LYHLi!+$5@$!5**('7! n*&!0&'(!6)1-!(#$!:/$'(=!,+'>$5!1#*)1$!>#$+!)(!1*0$'!(*!(#$!MNOLJ !$@$+!)2!)(!)'!
+*(!,33H$+1*06,'')+. 7!

15. 38yo M with 3-year hx of T2DM. Taking an oral antihyperglycemic agent, he has tried diet and exercise. BMI 32. PE
normal. Hb A1c is 10%. Physician recommends initiation of insulin injections. Responds, "I know that insulin would help
control my blood sugar. But a lot of people in my family have diabetes, and insulin made them really sick at times. Patient
is at which stage of change?
- Contemplation

Why it’s right: "#$5$!,5$!2)@$!'($6'!(* !0,-)+.!,!1#,+.$ !U)+!#$,3(#<!)+!3)2$<!$(17V
m!65$1*+($063,()*+<!1*+($063,()*+<!
65$6,5,()*+<!,1()*+<!,+4!0,)+($+,+1$7!e5$1*+($063,()*+!)'!>#$+!,!6,()$+(!#,'!+*(!$@$+!1*+')4$5$4!0,-)+.!(#$!1#,+.$!
;$(!?!)+!(#)'!'1$+,5)*<!(#$!6,()$+(!0,;!-+*>!(#,(!)+'&3)+!#$36'!>)(#!/3**4!'&.,5<!/&(!4*$'!+*(!(#)+-!)(!,663)$'!(*!#)07!W$!
0,;!',;<!:)(!>)33!+*(!#$36!0$7=!"#)'!6,()$+(!#,'!,35$,4;!1*+($063,($4!&')+.!)+'&3)+!,+4!$@$+!3$,5+$4!*2!(#$!')4$!$22$1('!
25*0!2,0)3;!0$0/ $5'7!A&(!#$!#,'!+*(!;$(!0,4$!(#$!4$1)')*+!(*!&'$!)+'&3)+!*5!'$(!,!2&(&5$!4,($!(*!'(,5(!)+'&3)+7
!D2!(#$!
6,()$+(!>,'!)+!(#$!65$6,5,()*+!6#,'$<!#$!>*&34!/$!5$,4;!(*!'>)(1#!(*!)+'&3)+!,+4!#,@$!,! future date!*5!()0$!6$5)*4!'$(!
>#$+!6*)+(!#$!>*&34!'(,5(!)+'&3)+7 !"#$!,1()*+!6#,'$!>*&34!/$!&')+.!)+'&3)+<!,+4!*/@)*&'3;!0,)+($+,+1$!>*&34!/$!
1*+()+&)+.!(*!&'$!)+'&3)+!$@$+!)2!#$!$I6$5)$+1$'!')4$!$22$1(' 7!O,)+($+,+1$!0,;!,3'*!)+13&4$!1*+(5*33)+.!4)$(!,+4!$I$51)'$!
(*!)+15$,'$!(#$!*44'!(#,(!(#$!6,()$+(!1,+!$@$+(&,33;!1*0$ !*22!)+'&3)+7
!
!
Take home point:!"#$!2)@$!'($6'!(*!0,-)+.!,!1#,+.$ !,5$!65$1*+($063,()*+<!1*+($063,()*+<!65$6,5,()*+<!,1()*+<!,+4!
0,)+($+,+1$7!N*0$*+$!>#*!#,'!+*(!0,4$!,!+$1$'',5;!#$,3(#!1#,+.$!;$(!/&(!)'!,>,5$!(#,(!>*&34!)(!>*&34!#$36<!,+4!*2!
(#$!6*'')/3$!' )4$!$22$1('<!)'!)+!(#$!1*+($063,()*+!6#,'$7!
!
!

16. 24yo M with 2-day history of an itchy rash on his arms and legs. Returned from a camping trip in the woods 5 days
ago. PE shows edematous, erythematous rash with linear vesicles. Cause is activation of which cell types?
- T lymphocytes

7 ©Test Pirates, LLC. All Rights Reserved.


Step 1 Practice Test #17 Explanations

Why it’s right: "#$!4),.+*')'!)'!1*+(,1(!4$50,()()'!25*0! 1*+(,1(!>)(#!6*)'*+!)@;7!J@$+!)2!;*&!(#*&.#(!(#)'!>,'!6*'')/3;!


'6*5*(#5)I!'1#$+1-))!U:5*'$!#,+43$59'!4)'$,'$=V!')+1$!)(!1,+!,3'*!1,&'$!,!5,'#!(#,(!)'!3)+$,5!,+4!@$')1&3
,5<!(#)'!)'!,!%&$'()*+!
,/*&(!#;6$5'$+')()@)(;!5$,1()*+'<! ,+4!)(9'!#)+()+.!,(!6*)'*+!)@;!,+4!,!(;6$!DX!#;6$5'$+')()@)(;!5$,1()*+7!"#$!5,'#!)'!,3'*!*+!
#)'!,50'!,+4!3$.'<!>#)1#!)'!3$''!3)-$3;!4&$!(*!5*'$!#,+43$59'!4)'$,'$!(#,(!>*&34!/$!1*+2)+$4!(*!(#$!#,+4'!,+4!,50'7!"#$!
-$;!1$33!0$4),(*5!*2!(;6$!DX!#;6$5'$+')()@)(;!5$,1()*+'!)'!(#$!"!3;06#*1;($7!"!3;06#*1;($'!,1()@,($!0,15*6#,.$'!,+4!
(#$5$!)'!+*!,+()/*4;!)+@*3@$0$+(7!8(#$5!(;6$!DX!#;6$5'$+')()@)(;!$I,063$'!)+13&4$!1*+(,1(!4$50,()()'!4&$!(*!+)1-$3<!ee
($'(<!.5,2(H@$5'&'H#*'(!4)'$,'$<!0&3()63$!'13$5*')'<!,+4!C&)33,)+HA,55$!';+45*0$7!!
!
Take home point:!e*)'*+!)@;!1,&'$'!1*+(,1(!4$50,()()'<!,!(;6$!DX!#;6$5'$+')()@)(;!5$,1()*+7! !
!

17. 70yo M from china with poorly differentiated monoclonal carcinoma of the nasopharynx. DNA probes of neoplastic
cells are most likely to detect genome of which virus?
- Epstein-Barr virus

Why it’s right: JAX!1,&'$'!)+2$1()*&'!0*+*+&13$*')'<!A&5-)((9'KW*4.-)+9'!3;06#*0,<!,+4!+,'*6#,5;+.$,3!1,51)+*0,7!
T2($5!)+)(),3!)+2$1()*+<!JAX!5$0,)+'!3,($+(!)+!A!1$33'7!"#$!3*+.$5!(#$!JAX!5$0,)+'!3,($+(!)+!A!1$33'<!(#$!#).#$5!3)-$3)#**4!
JAX!1,+!3$,4!(*!,+!JAX H,''*1),($4!0,3).+,+1;7!e,()$+('!)+!3$''!4$@$3*6$4!1*&+(5)$'!1*+(5,1(!(#$!@)5&'!$,53)$5!)+!3)2$!,+4!
(#$5$2*5$!#,@$!,+!)+15$,'$4!1#,+1$!*2!4$@$3*6) +.!,+!JAXH,''*1),($4!0,3).+,+1;!3,($5!)+!3)2$7!!
!
!
Take home point:!L,($+(!JAX!)+!A!1$33'!1,+!1,&'$!+,'*6#,5;+.$,3!1,51)+*0,7!! !

18. Epidemiologic study of aniline dye, 500 workers with bladder cancer and 200 workers without. Exposed to aniline
dyes/Have Bladder cancer: Yes/Yes (250), Yes/No (50), No/Yes (250), No/No (150). Odds ratio?
- 3

Why it’s right: B*5!(#)'!%&$'()*+<!;*&!#,@$!(*!45,>!(#$!5)'-!2,1(*5!@'7!4)'$,'$!1#,5(!,+4!-+*>!(#,(!8\!^! U,K1VKU/K4V!^!
U,o4 VKU/o1Vm !
!!!!!!!!!!!!!!!!!!!!!!!!!!!!
b)'$,'$m!!!!!!;$'!!!!!!!!!!!!!+*!!
JI6*'&5$m!!!;$'!!!!!!!! ! !!!!!YRF!U,V!!!!!!RF!U/V
!
!!!!!!!!!!!!!!!!!!!!!
g*!!!!! !!!!!YRF!p!!!!!!!`RF!U4V!
!!!!!!!!!!!!!!!!!!!!!!!!!!!!!!!!!!!
! !!^!UYRFo`RF VKURFoYRF V!
! ! ! !!^!Z!
!
Take home point:!8\!^! U,o4VKU/o1V<!*5!*44'!(#,(!(#$!.5*&6!>)( #!(#$!4)'$,'$!>,'!$I6*'$4!(*!(#$!5)'-!2,1(*5!4)@)4$4!/;!
(#$!*44'!(#,(!(#$!.5*&6!>)(#*&(!(#$!4)'$,'$!>,'!$I6*'$4!(*!(#$!5)'-!2,1(*5 7!

19. 24yo M with hx of IVDA could not be aroused. Friend reports that the patient injected himself with a drug 6 hours
ago. Labs show drug concentration of 0.3. Assuming first-order one-compartment kinetics, has a half-life of 2 hours, and
a volume of distribution of 200 L in this patient. What is quantity of drug (in mg) injected?
- 480

Why it’s right: D2!)(!#,'!/$$+!a!#*&5'!
')+1$!(#$!45&.!>,'!)+P$1($4!,+4!(#$!#,32H3)2$!)'!Y!#*&5'<!(#$+!(#$5$!#,@$!/$$+!,!(*(,3!
*2!Z!#,32
H3)@$'7!N*!(#,(!0$,+'!(#$!45&.!1*+1$+(5,()*+!>,'!#,3@$4!Z!()0$'7!N*!)2!>$!'(,5(!25*0!(#$!$+4!,+4!65*1$$4!
/,1->,54'm!F7Z!! !F7a! ! !`7Y!! !Y7i7!"#$!'(,5()+.!1*+1$+(5
,()*+!>,'!Y7i!0.KL7!T+4!)2!(#$!@*3&0$!*2!4)'(5)/&()*+!)+!YFFL<!
(#$+!(#$!(*(,3!0.!*2!45&.!)+P$1($4!>,'!ihF0.<! ')+1$!ihF0.KYFFL!^!Y7i!0.KL7 !

8 ©Test Pirates, LLC. All Rights Reserved.


Step 1 Practice Test #17 Explanations

!
Take home point:!!B*5!#,32
H3)2$!%&$'()*+'<!)(!)'!$,')$
'(!(*!45,>!*&(!(#$!#,32

9 ©Test Pirates, LLC. All Rights Reserved.


Step 1 Practice Test #17 Explanations

Take home point:!T!5,+4*0)[$4!1*+(5*33$4!(5),3!'(&4)$'!(#$!$22$1('!*2!,!'6$


1)2)1!(5$,(0$+(
!*5!)+($5@$+()*+!*+!(>*!*5!
0*5$!.5*&6'7!!
!

23. 12yo boy immersed up to his neck in 60F water for 20 minutes. Physiological changes?
- Central blood volume INCREASED, ADH (vasopressin) DECREASED, Atrial Natriuretic Peptide INCREASED??

Why it’s right: "* !,+'>$5!(#)'!%&$'()*+!>$!0&'(!&+4$5'(,+4!$,1#!2,1(*5 !,'-$4!)+!(#$!%&$'()*+7!"#$!2)5'(m!1$+(5,3!/3**4!
@*3&0$7! W$5$<!>$!,5$!(,3-)+.!, /*&(!/3**4!@*3&0$!.*)+.!(*!(#$!1*5$!*2!(#$!/*4;<!(#$!@)(,3!*5.,+'<!,-,!+*(!(*!(#$!
$I(5$0)()$'7!N*!>#,(!#,66$+'!>#$+!'*0$*+$!)'!25$$[)+.!(*!4$,(#Q!"#$!/*4;!5$'6*+4'!/;!)+15$,')+.!1$+(5,3!/3**4!23*>!
(*!65$'$5@$!(#$!@)(,3!*5.,+' !?!(*!-$$6!(#$0!:>,50 7=!8-,;<!( #,(!*+$!>,'!$,' ;! ?!(#$!*(#$5!(>*<!+*(!'*!0&1#r !
!
T+()H4)&5$()1!#*50*+$!UTbWV!,+4!,(5),3!+,(5)&5$()1!6$6()4$!UTgeV!4*!(#$!$I,1(!*66*')($! *2!$,1#!*(#$5<!'*!)2!;*&!6)1-$4!
(#$!*6()*+!(#,(!'#*>'!(#$0!/*(#!.*)+.!&6!*5! /*(#!.*)+.! 4*>+<!;*&!#,@$+9(!$@$+!'($66$4!*+( *!(#$!63,;)+.!2)$34!;$(7!TbW!
5$'6*+4'!(*!,!3*>!@*3&0$!'(,($!(*!)+15$,'$!@*3&0$!U,+() H4)&5$()1rV<!>#)3$! Tge!5$'6*+4' !(*!,!#).#!@*3&0$!'(,($!,+4!
4$15$,'$'!/3**4!@*3&0$7!g$)(#$5!*2!(#$'$!#*50*+$'! )' !>#,(!)'!1,&')+.!,+!)+15$,'$!)+!1$+(5,3!/3**4!23*>!)+!(#)'!25$$[)+.H
(* H4$,(#!/*;! ?!*(#$5!#*50*+$'K$+[;0$'!,5$!1,&')+.!@,'*1*+'(5)1()*+<!$(17!D+'($,4<!(#$'$!#*50*+$'!,5$! responding!(*!
(#$!1$+(5,3!@*3&0$!'(,($<!,+4!(#,(!)'!>#,(!(#$!%&$'()*+!)'!,'-)+.7!D2!1$+(5,3!/3**4!23*>!)'!)+15$,'$4<!(#$5$2*5$<!(#$+!Tb
)'!.*)+.!(* !',;<!#$; f!(#$5$!)'!(**!0&1#!/3**4!@*3&0$ <!'(*6!5$(,)+)+.!>,($5<!,+4!4$15$,'$G!Tge!)'!.*)+.!(*!',;<!#$; f!(#$5$!
)'!(**!0&1#!/3**4<!4)&5$')'<!,+4!)+15$,'$!')+1$!(#,(9'!>#,(!Tge!4*$'7! !
!
Take home point:!_$+(5,3!/3**4!23*>! )+15$,'$'!)+!3*>!($06$5,(&5$'!(*!65$'$5@$!/3**4!23*>!(*!@)(,3!*5.,+'7!TbW!
5$'6*+4'!(*!)+15$,'$!/3**4!23*>!/;!4$15$,')+.!U')+1$!)('!P*/!)'!(*!)+15$,'$!/3**4!@*3&0$V! ,+4!Tge!5$'6*+4'!/;!)+15$,')+. !
U')+1$!)('!P*/!)'!(*!4$15$,'$!/3**4!@*3&0$V 7!!
!!

24. Randomized controlled study of 2000 patients with insomnia is conducted to evaluate the efficacy of a new
medication to treat this condition. Ten subjects from both the control and treatment groups do not complete the study
are not included in the analysis. Treatment group able to fall asleep 5 minutes faster than control (p=0.001). Neither
group report an improvement in quality of life. Conclude that new med is efficacious in treating insomnia. Type of error?
- Failure to distinguish between statistical significance and clinical significance

Why it’s right: N*0$()0$'!%&$'()*+'!*+!(#$!5$,3!($'(!,5$!(#)'!')063$7!b*+9(!/$!2**3$4!)+(*!6)1-)+.!,!(;6$!D!*5!(;6$!DD!$55*5!
P&'(!/$1,&'$!;*&!'6$+(!(#$!3,'(!Yi!#*&5'!65)*5!(*!(#$!($'(!15,00)+.!'(,('7!D2!;*&!5$,4!(#$!%&$'()*+!U;$'<!;*&95$!$I6$1($4!
(*!5$,4!(#$!%&$'( )*+<!1,5$2&33;V<!(#$+!;*&!>)33!'$$!(#,(!(#$!3)+$!,/*&(!'&/P$1('!+*(!1*063$()+.!(#$!'(&4;!)'!,!4)'(5,1(*57!
n*&!>)33!,3'*!5$,3)[$!(#,(!(,-)+.!,!45&.!(*!/$!,/3$!(*!2,33!,'3$$6!R!0)+&($'!2,'(
$5!(#,+!63,1$/*!)'!g8"!,!.**4!5$,'*+!(*!
(,-$!,!45&.7!"#$!6!@,3&$!*+3;!'#*>'!').+)2)1,+1$!/$1,&'$!(#)'!)'!,! 3,5.$!'(&4;!UYFFF!6,()$+('V7!J@$+!,!ZF!'$1*+4!
4)22$5$+1$!/$(>$$+!(#$!(>*!.5*&6'!0,;!65*@$!(*!/$!'(,()'()1,33;!').+)2)1,+(7!"#$;!$@$+!.*!,'!2,5!,'!(*!',;!(#,(!/*(#!
.5*&6'!4)4+9(!5$6*5(!%&,3)(;!*2!3)2$!)065*@$0$+(7!L*.)1 ,33;<!
(#)'!'(&4;!*+3;!65*@$'!,!'(,()'()1,33;!').+)2)1,+(!5$'&3(<!/&(!
13)+)1,33;<!+*!4)1$f
!
!
Take home point:!"#*&.#!'(&4)$'!0,;!'#*>!'(,()'()1,3!').+)2)1,+(!4&$!(*! ,!3,5.$!'(&4; !U#).#!6*>$5V<!(#$;!0,;!+*(!/$!
13)+)1,33;!').+)2)1,+(!$'6$1),33;!)2!(#$!4)22$5$+
1$!/$(>$$+!(#$!(>*!.5*&6'!)'!+*(!13)+)1,33;!5$3$@,+(7! !
!

25. 50yo M with increasing cough for 6 month and hemoptysis for 1 week. Smoked 1 pack per day for 32 years. Plays
squash, swims. CXR shows a 3 x 4-cm hilar mass. Cytological examination of sputum shows a non-small cell carcinoma.
Tells patient he has lung cancer. The patient responds, "How can this be happening to me? I eat right and exercise."

10 ©Test Pirates, LLC. All Rights Reserved.


Step 1 Practice Test #17 Explanations

Appropriate response?
- "It must be difficult for you to accept this diagnosis when you feel healthy."

Why it’s right: A5$,-)+.!/,4!+$>'!)' !,+!$(#)1'!(*6)1!25$%&$+(3;!,'-$4!*+!(#$
!MNOLJ !,+4!)+!5$,3!4*1(*5)+.!3)2$
7!"#$!/$'(!
>,;!(*!/5$,-!/,4!+$>'!)'!(*!,'-!*6$+ H$+4$4!%&$'()*+'!,(!2)5'(<!65*@)4$!5$,''&5)+.!,+'>$5'K2$$4/,1-<!,+4!+$@$5!
'6$1&3,($!,/*&(!(5$,(0$+(!?!)7$7!(#$!1&5$!)'!/$)+.!5$'$,51#$4!,+4!0,;!/$!,!6*'')/)3)(;7!82!1*&5'$<!13)+)1,3!(5),3'!>)(#!*6$+!
$+5*330$+(!)'!,!6*'')/)3)(;!)+!(#$!2&(&5$<!/&(!)(!)'!0&1#!(**!'**+!(*!4)'1&''!(#,(!(*6)17!8/@)*&'3;<!;*&!4*+9(!>,+(!(*!',;<!
;*&!#,@$!3&+.!1,+1$5<!#*>!4*!;*&!2$$3!

11 ©Test Pirates, LLC. All Rights Reserved.


Step 1 Practice Test #17 Explanations

0,3$<!)'!+*(!(;6)1,37!"*!4),.+*'$!,+-;3*')+.!'6*+4;3)()'<!(#$!,+'>$5!)'!, 1(&,33;!+*(!WLT HAYE!($'()+.<!(#


*&.#!(#,(!1, +!/$!
4*+$G!)(!)'!IH5,;!'6$1)2)1,33;!*2!(#$!',15*)3),1!P*)+('!/$1,&'$!(#)'!4)'$,'$!1,&'$'!)+23,00,()*+!,(!(#)'!P*)+!U(#$!ND!P*)+(V!,+4!
&3()0,($3;!2&')*+7!"#$'$!6,()$+('!,3'*!'&22$5!25*0!@$5($/5,3!P*)+(!2&')*+!U:/,0/**!'6)+$=V<!/&(!,.,)+<!(#$!.*34!'(,+4,54!
2*5!4),.+*')'!)'!',15*)3),1!P*)+(!IH5,;7!T+*(#$5!/&[[>*54!,''*1),($4!>)(#!(#)'!4)'$,'$!)'!$+(#$')()'<!>#)1#!)'!)+23,00,()*+!
,(!(#$!6*)+(!*2!($+4*+!,((,1#0$+(!(*!/*+$7!!e,()$+('!>)(#!6'*5),()1!,5(#5)()'!1,+!,3'*!4$0*+'(5,($!$+(#$')()'7!!
!
Take home point:!"#$!.*34!'(,+4,54!(*!4),.+*'$!,+-;3*')+.!'6*+4;3)()'!)'!I H5,;!*2!(#$!',15*)3),1!P*)+('7! !

28. 17yo boy with 8-kg weight gain during the past year. No medical illness. BMI is 32. He asks, "Do you think that my
weight gain is inherited from my father?" Appropriate response?
- "Yes, your weight gain can be caused by genes and environment combined."

Why it’s right: "#)'!)'!(#$!:/$'(=!,+'>$5!1#*)1$7!d$!-+*>!(#,(!>$).#(!,+4!(;6$!Y!4),/$($'!#,'!,!'(5*+.!.$+$()1!
1*++$1()*+<!,+4!6,()$+('!>#*!#,@$!,!2,0)3;!#)'(*5;!*2!* /$')(;!*5!(;6$!Y!4),/$($'!#,@$!,!.$+$()1!65$4)'6*')()*+!(*!(#$'$!
4)'$,'$'7!D(!)'!)06*5(,+(<!(#*&.#<!+*(!(*!,((5)/&($!>$).#(!.,)+!1*063$($3;!(*!.$+$()1'! ?!)(!)'!,3'*!4&$!(*!$+@)5*+0$+(,3!
1,&'$'<!)7$7!$I$51)'$<!2**4!1#*)1$'<!$(17!N*!(#$!,665*65),($!,+'>$5!>* &34!/$<!:;$'!,+4!+*m!)(!)'!,!1*0/)+,()*+!*2!.$+$()1'!
25*0!;*&5!2,(#$5<!/&(!,3'*!(#$!1#*)1$'!;*&!0,-$!)+!($50'!*2!4)$(!,+4!$I$51)'$7=!g*>!(#,(!>*&34!/$!(#$!/$'(!,+'>$5<!/&(!
(#$!MNOLJ! >)33!\T\JLn!.)@$ !'&1#!,!13$,5
H1&(!,+'>$5!(*!,+!$(#)1'!%&$'()*+7!!
!
Take home point:!8/$')(;!,+4!(;6$!Y!4),/$($'! ,5$!4&$!(*!/*(#!.$+$()1!,+4!$+@)5*+0$+(,3! 2,1(*5'7!!
!!

29. 52yo M neighbors have reported that he has been confused and not taking care of himself. 4-mo Hx of diarrhea. PE
shows extreme muscle wasting, stomatitis, and a diffuse rash that is worse in sun-exposed areas. Diagnosis?
- Pellagra

Why it’s right: "#)'!6,()$+(!)'!+),1)+H4$2)1)$+(!U@)(,0)+!AZV7!g),1)+!)'!2*&+4!)+!'$@$5,3!2**4'!U0$,('<!'$,2**4'<!,+4!'6)1$'V
,'!>$33!,'!)+!65*1$''$4!2**4'!(#,( !,5$!2*5()2)$4
!>)(#!AZ7!e,()$+('!>#*!,5$!4$2)1)$+(!)+!+),1)+!,5$!3)-$3;!(*!$)(#$5!/$!TV!+*(!
$,()+.!2**4!*5!AV!#,@$!'*0$!'*5(!*2!0,3,/'*56()*+!65*1$''7!g),1)+!4$2)1)$+1;!5$'&3('!)+!(#$!(#5$$!b9'm!4),55#$,<!
4$50,()()'<!,+4!4$0$+(),!U,+4!$@$+(&,33;!4$,(#V7!"#$!4$50,()()'!,''*1),($4! >)(#!+),1)+!4$2)1)$+1;!)'!'6$1)2)1,33;!65$'$+(!
*+!'&+H$I6*'$4!, 5$,' 7!"#)'!6,()$+(!2&32)33'!(#$!(#5$$!b9'!
?!1*+2&'$4!,+4!+*(!(,-)+.!1,5$!*2!#)0'$32!U4$0$+(),V<!4),55#$,!Ui!
0*+(#'ffV<!,+4!4$50,()()' 7!!
!
T!2&+!>,;!(*!,'-!,!%&$'()*+!,/*&(!+),1)+!4$2)1)$+1;!)'!( *!65$'$+(!'*0$*+$!>)(#!1,51)+*)4!';+45*0$!>#*!,3'*!65$'$+('!
>)(#!,33!(#$'$!').+'!*2!+),1)+!4$2)1)$+1;<!,+4!(*!,'-!>#,(9'!(#$!4)'$,'$!0$1#,+)'0 7!"#$!,+'>$5!)'!3*''!*2!(5;6(*6#,+!
/$1,&'$!(5;6(*6#,+!)'!/*(#!,!65$1&5'*5!*2!+),1)+!,'!>$33!,'!'$5*(*+)+!U,+4! ,3'* <!0$3,(*+)+V7!D+!1,51)+*)4!';+45*0$<!(#$5$!
)'!,+!*@$565*4&1()*+!*2!'$5*(*+)+!U1,&')+.!23&'#)+.!,+4!4),55#$,V <!>#)1#!4$63$($'!(#$!(5;6(*6#,+!'(*5$'!?!,+4!(#$+!
(#$5$9'!+*!(5;6(*6#,+!(*!'&663;!+),1)+! ! !(#$!(#5$$!b9'f!
!
Take home point:!g),1)+!U@)(,0)+!AZV!4$2) 1)$+1;!)'!1#,5,1($5)[$4!/;!(#$!(#5$$!b9'!?!4$0$+(), <!4),55#$,<!,+4!4$50,()()'!
U1#,5,1($5)'()1,33;!65$'$+(!*+! '&+H$I6*'$4!,5$,'V !r!,+4!4$,(#<!$@$+(&,33;7!!

30. 80yo F with suspected temporal arteritis (TA). ESR is 100, Pretest probability for TA is 50% in this patient. In the
evaluation of TA, ESR has a sensitivity of 99% and specificity of 60%. Based on the results of the ESR testing, most
appropriate next step?
- Additional testing to confirm the diagnosis of TA

12 ©Test Pirates, LLC. All Rights Reserved.


Step 1 Practice Test #17 Explanations


Why it’s right: D2
!;*&!5$0$0/$5!+*(#)+.!,/* &(!#*>!(*!1,31&3,($! '$+')()@)(;!,+4!'6$1)2)1)(;<!5$0$0/$5!Ng8M"!,+4!NeDg7!
W).#!'$+')()@)(;!5&3$'!,!4)'$,'$!8M"7!T+4!#).#!'6$1)2)1)(;!5&3$'!,!4)'$,'$!Dg7!N*!,!@$5;!'$+')()@$!($'(!U3)-$!jjSV<!5&3$'!,
4)'$,'$!8M"!*+3;!)2!(#$!5$'&3(!)+!+$.,()@$7!W$5$<!(#$!JN\! )'!`FF<!>#)1#!)'!#).#<!'*!(#)'!)'!6*')()@$!5$'&3(!,+4!,!'$+')()@)(;!*2!
jjS!4*$'!+*(!5&3$!(#$!4)'$,'$!*&(7!"#$!'6$1)2)1)(;<!(#*&.#<!)'!+*(!@$5;!#).#!U*+3;!aFSV7!D2!(#$!JN\!>,'!jjS!'6$1)2)1!($'(<!
(#$+!>$!>*&34!*22)1),33;!4),.+*'$!(#)'!6, ()$+(!>)(#!($06*5,3!,5($5)()'!/$1,&'$!(#$!#).#3;!'6$1)2)1!($'(!>*&34!5&3$!(#$!
4)'$,'$!Dg7!A&(<!&+2*5(&+,($3;<!(#$!'6$1)2)1)(;!)'!3*><!'*!>$!1,++*(!5&3$! )+!(#$!4)'$,'$7!"#$!'($6!Y!_c!%&$'()*+Q!?!d#,(!
>*&34!/$!(#$!+$I(!/$'( !($'(Q!L)-$3;!
($06*5,3!,5($5;!/)*6'; 7!W).#!63,($3$('!,5$ !,+*(#$5!').+!*2!($06*5,3!,5($5)()'7!
"$06*5,3!,5($5)()'!)'!5$,33;!,!13)+)1,3!4),.+*')'7!
D+!($50'!*2!(5$,(0$+(<!'($5*)4'!'#*&34!g8"!/$!4$3,;$4!>#)3$!>,)()+.!2*5!
(#$!($06*5,3!,5($5;!/)*6';7!!
!
Take home point:!\$0$0/$5!Ng8M"!,+4!NeDg!^!#).#3;!'$+')()@$!($'(!5 &3$'!,!4)'$,'$!8M"!U)2!(#$!5$'&3(!)'!+$.,()@$V<!,+4!,!
#).#3;!'6$1)2)1!($'(!5&3$'!,!4)'$,'$!Dg!U)2!(#$!5$'&3(!)'!6*')()@$V7!M'&,33;!@$5;!'$+')()@$!($'('!,5$!6$52*50$4!2)5'(!,+4!(
+!
'6$1)2)1!($'('!,2($5!(*!1*+2)50!,!4),.+*')'7!"#)'!)'!(#$!/,')'!/$#)+4!WDX!($'()+.m!(#$!JLDNT!U#).#3;!'$+')()@$!($'(V!)'!
6$52*50$4!2)5'(<!,+4!)2!6*')()@$<!(#$!d$'($5+!/3*(!U#).#3;!'6$1)2)1!($'(V!)'!6$52*50$4!(*!0,-$!(#$!1*+2)50$4!4),.+*')'7! !

31. Sequence surrounding the first two exons of the human beta-globin gene shown, with exons in bold. Translation start
codon is underlined. A mutation from G-->A at position 355 is most likely to lead to beta-thalassemmia by which
mechanism?
- Disruption of normal splicing by creation of a new 3' splice site

Why it’s right: N63)1$!')($'!,5$!


>#$5$!(#$!$I*+'!,+4!)+(5*+'!0$$(7!L$(9'!.$(!,!2$>!(#)+.'!'(5,).#(!2)5'(7!bgT!1,+!/$!
replicated!(*!0,-$!+$>!bgT7!bgT!1,+!,3'*!/$! transcribed!(*!0,-$!\gT7!"#)'!\gT!)'!(#$+!:65*1$''$4=!)+(*!0\gT<!
/$2*5$!)(!1,+!/$!(5,+'3,($47!e*'(H65*1$''$4!\gT!)'!(#$+!translated!(*!65*($)+'!/&)3(!/;!,0)+*!,1)4'7!"#$!65*1$'')+.!6,5(!
)'!>#$+!\gT!)'!'63)1$4<!0$,+)+.!)+(5*+'!,5$!5$0*@$4!(*!*+3;!)+13&4$'!$I*+' !U'*&+4'!1*&+($5)+(&)()@$<!/&(!4*+9(!+,0$!
0$!2*5!(#$!+*0$+13,(&5$V 7!T(!(#)'!6*)+(<!*+1$!,33!$I*+'!,5$!,'')0)3,($4!6*'(
H)+(5*+!5$0*@,3<!(#$!0\gT!$I)('!(#$!+&13$&'!
(*!/$!(5,+'3,($47!e#$>f!
!
N*<!(#)'!%&$'()*+!)'!,'-)+.<!)2!,!6*)+(!0&(,()*+!U*+$!/,'$!6,)5!1#,+.$'!25*0!C!(*!TV!,(!(#$!')($!*2!,!'63)1$!')($!U*5!P&+1()*
+!
*2!)+(5*+!,+4!$I*+V!*11&5'<!>#,(!)'!(#$!5$'&3(Q!d$33<!+*50,3!'63)1)+.!>)33!/$!4)'5&6($4!,+4 !,!+$>!'63)1$!')($!15$,($47!D2!
;*&!P&'(!0$0*5)[$4!(#$!3)+$!)+!B)5'(!T)4!(#,(!4$'15)/$'!,!1,&'$!*2!/$(,H(#,3,''$0),!,'!,!6*)+(!0&(,()*+!)+!'63)1$!')($'!,+4!
65*0*($5!'$%&$+1$'<!(#$+!;*&!>*&34!'()33!.$(!(#)'!5).#(7!_*+.5,('f !
!
Take home point:!A$(,H(#,3,''$0),!)'!1,&'$4!/;!,!6*)+(!0&(,()*+!)+! , !'63)1$!')($!*5!65*0*($5!'$%&$+1$!(#,(!5$'&3('!)+!
(#$!2*50,()*+!*2!,!+$>!65*($)+7! !
!

32. 42yo M in ED because of a 10-day history of progressive fever, SOB, and nonproductive cough. 20-kg weight loss.
Immigrated to USA from the Ivory Coast 4 years ago. T 38C. Lungs clear, CXR shows diffuse interstitial infiltrate. Silver
stain obtained via bronchoscopy shows Pneumocystis jiroveci (formerly P. carinii). High dose prednisone and
trimethoprim-sulfamethoxazole is initiated, and workup for HIV infection is done. HIV ELISA positive, HIV western blot
positive, CD4 22; HIV viral load <50. Explanation?
- Infection with HIV-2

Why it’s right: T33!(#$'$!').+'<!';06(*0'<!,+4!4),.+*'()1!($'('!6*)+(!(*>,54'!WDX7!" #)'!6,()$+(!#,'!e_e!6+$&0*+),<!>#)1#!
)'!,+!TDbNH4$2)+)+.!4)'$,'$7!"#$!_bi!1*&+(!)'!,3'*!3*>!UuRFV7! "#$!*+3;!5$,'*+!2*5!,!3*>!WDX!@)5,3!3*,4<!)'!(#,(!(#$!WDX!($'(!
2*5!@)5,3!3*,4!*+3;!4$($1('!WDX
H̀!,+4!+*(!WDX HY7!WDXHY!)'!3$''!1*00*+!,+4!3$''!6,(#*.$+)1!U0$, +)+.!)(!typically!4*$'+9(!

13 ©Test Pirates, LLC. All Rights Reserved.


Step 1 Practice Test #17 Explanations

4),0$($5!U3,5.$5!(#,+!6$+1)3!()6!$5,'$5V<!,+4!$@*3&()*+!U1#,+.)+.!*@$5!()0$ V7!"#$'$!,5$!,33!>,5+)+.!').+'!(#,(!>,55,+(!
/)*6';!)+!(#$!65*6$5!13)+)1,3!1*+($I(7!D2!(#$!3$')*+!)'!3,5.$5!(#,+!,!6$+1)3!()6!$5,'$5<!/&(!)'!,3'*!';00$(5)1!>)(#!5$.&3,5!
/*54$5'<!*+$!1*3*5!U$@$+!)2!@$5;!4,5-!*5!/3&$V<!,+4!'(,/3$!)+!')[$K0*56#*3*.;!*@$5!(#$!6 ,'(!R!;$,5'<!(#$+!(#)'!3)-$3;!4*$'!
g8"!+$$4!(*!/$!/)*6')$47!W*>$@$5<!)2! 0*5$!*2!(#$'$!15)($5),!,5$!0,(1#$4!(#$+!+*(<!/)*6';f!J@$+!)+!;*&+.!6,()$+('!
/$1,&'$!(,++$5'!,+4!(,++)+. H/**(#!&'$5'!,5$!65*+$!(*!0$3,+*0,!$@$+!,'!($$+,.$5'!)2!(#$;!(,+!$+*&.#!,+4!,5$!2,)5H
'-)++$47!D+!(#)'!%&$'()*+<!(#$!1#,+.$!)+!')[$!,3*+$!*@$5!,!'#*5(!6$5)*4!UP&'(!i!0*+(#'V!)'!5$,'*+!$+*&.#!(*!/)*6';!,+4!
4),.+*'$!,!0,3).+,+(!65*1$''7!W)'(*3*.;!)'!1*+')'($+(!>)(#!,!0,3).+,+(!65*1$''! ?!63$*0*56#)'0<!#;6$51#5*0,()1!
1#,+.$'<!$I($+4)+.!6,'( !(#$!$6)4$50)'!)+(*!(#$!4$50)'f! W;6$51#5*0),!0$,+'!0*5$!/3&$!U/,')1,33;V! ?!0*5$!/3&$!1*3*5!*+!
#)'(*3*.;!0$,+'!3*('!*2!bgT!0,($5),3<!3*('!*2!bgT!5$63)1,()*+<!/).!+&13$)!&+4$5.*)+.!5$63)1,()*+<!$(1! ?!0*5$!/3&$!(#,+!
6)+-7!"#)'!6,()$+(!+$$4'!,!>*5-&6!2*5!0$(,' (,')'!,(!(#)'!6*)+(7!"5$,(0$+(!)'!$I1)')*+!>)(#!dDbJ!0,5.)+'!U3)-$!Y10V !,+4!,!
0$(,'(,()1!>*5-&67!n*&!'#*&34!/$!,3'*!,/3$!(*!)4$+()2;!0$3,+*0,!*+!#)'(*3*.;!)2!,!6)1(&5$!)'!65$'$+($47! !
!
Take home point:!\$0$0/$5!(#$!TA_bJ9'!*2!0$3,+*0,ff!^!,';00$(5;<!/*54$5' !U)55$.&3,5V<!1*3*5!U0&3()63$!1*3*5'V<!
4),0$($5!U3,5.$5!(#,+!6$+1)3!()6!$5,'$5V<!,+4!$@*3&()*+!U1#,+.)+.!*@$5!()0$V7 !

36. 42yo farmer has a 7-mm red scaly plaque on helical rim of left ear. Photomicrograph shown. Dx?
- Actinic keratosis

Why it’s right: "# )'!%&$'()*+!)'!,!2,@*5)($!*+!(#$!MNOLJ!N($6!`!$I,07!D2!;*&!.$(!(*!(#$!$+4!*2!(#$!$I,0!,+4!;*&!+$@$5!
6)1-$4!,1()+)1!-$5,(*')'!,'!,!4),.+*')'<!(#$+!;*&!>)33!+*(!/$!.$(()+.!,!`FFSf!"#)'!)'!,!2,@*5)($<!,3'*<!2*5!6)1(&5$'7!c+*>!
>#,(!)(!3**-'!3)-$!?!)(!3**-'!3)-$!,!3&06K/&06!>)(#!,!1$+($5!*2!'1,3;!1#,+.$'!*5!-$5,()+!U>#)($<!45;!1#,+.$'! >)(#)+!(#$!
3$')*+V7!D(!)'!,30*'(!,3>,;'!*+!,!'&+H$I6*'$4!5$.)*+!3)-$!(#$!#$3)I!*2!(#$!$,5!*5!'1,36!*2!,!0,+!>)(#!3)((3$!#,)57!T1()+)1!
,1(&,33;!)063)$'!'&+7!T1()+)1!1#,+.$'!)+!(#$!$ ;$!2*5!$I,063$!,5$!4&$!(*!'&+H$I6*'&5$<!,+4!6($5;.),!U')+.&3,5!6($5;.)&0V!
1,+!#,@$!,1()+)1!1#,+.$'!*+!#)'(*3*.;!')+1$!'&+!)'!,!#&.$!5)'-!2,1(*5!2*5!6($5;.),7!"#$!*(#$5!4$5)@,()*+!*2!,'-)+.!(#)'!
%&$'()*+!)'<!>#,(!)'!(#)'!6$5'*+!65$4)'6*'$4!(*Q!T+'>$5m!'%&,0 *&'!1$33!1,51)+*0,7!g*(!/,',3!1$33<!,+4!+*(!0$3,+*0,7!
T1()+)1!-$5,(*')'!)'!g8"!1,+1$5<!/&(! )(!)'!,!65$1,+1$5*&'!3$')*+7!
!
!
Take home point:!T1()+)1!-$5,(*')'!)'!4&$!(*!'&+!$I6*'&5$!,+4!)'!,!2,@*5)($!*2!/*,54!$I,0)+$5'!(*!65$'$+(!)+!6)1(&5$!
2*50,(7!c+*>!>#,( !)(!3**-'!3)-$f!T1()+)1!0$,+'!'&+H1#,+.$'7!!
!

37. 30yo F training for a marathon, running 20 min/day. Fasting glucose is 60. After her glucose stores have been
depleted, which organ, in addition to liver, releases glucose?
- Kidney

Why it’s right: D+!(#$!2,'()+.!'(,($<! (#$!3)@$5!5$3$,'$'!.3&1*'$!65)0,5)3;!/;!/5$,-)+.!4*>+!.3;1*.$+!'(*5$'!4&5)+.!
.3;1*.$+*3;')'7!"#$!3)@$5!,3'*!0,-$'!.3&1*'$!(#5*&.#!.3&1*+$*.$+$')'!/$1,&'$!(#$!3)@$5!#,'!(#$!$+[;0$!.3&1*'$ HaH
6#*'6#,(,'$<!>#)1#!)'!+$1$'',5;!)+!.3&1*'$!65*4&1()*+7!"#$!*(#$5!*5.,+'!(#,(!,3'*!#,@$!(#)'!$+[;0$!,+4!6,5()1)6,($!)+!
.3&1*'$!65*4&1()*+!)+13&4$!(#$!-)4+$;!,+4!)+($'()+,3!$6)(#$3)&07!T+*(#$5!'&/'(,+1$!(#,(!1,+!,3'*!5$'&3(!)+!.3&1*'$!
65*4&1()*+!,5$!*44 H1#,)+!U+*(!$@$+ H1#,)+V!2,((;!,1)4'!5$3$,'$4!/;!,4)6*'$7!"#$'$!$+($5!(#$!"_T!,'!'&11)+;3
H_*T!,+4!
2*33*>!(#$!6,(#!/,1-!(*!.3&1*'$!(#5*&.#!.3&1*+$*.$+$')'7!! !
!
Take home point:!"#$!0,P*5)(;!*2!.3&1*'$!)+!(#$!2,'()+.!'(,($!)'!4$5)@$4!25*0!.3;1*.$+!25*0!(#$!3)@$5!@),!.3;1*.$+*3;')'7!
8(#$5!'&663)$'!*2!.3&1*'$!)+13&4$!. 3&1*+$*.$+$')'!/;!(#$!3)@$5<!(#$!-)4+$;'<!,+4!(#$!)+($'()+,3!$6)(#$3)&07!T4)6*'$!,3'*!
)+15$,'$'!/3**4 !.3&1*'$!@),!(#$!5$3$,'$!*2!*44 H1#,)+!2,((;!,1)4'7!
!

15 ©Test Pirates, LLC. All Rights Reserved.


Step 1 Practice Test #17 Explanations

38. Male newborn at 28 weeks'. Given ventilator support with up to 80% oxygen for the next 72 hours, but dies of resp.
failure. Cause is inadequate secretion from which labeled cell types?
- Type II pneumocyte (produces surfactant)

Why it’s right: n*&!0&'(!-+*>!(#$!(#5$$!(;6$'!*2!1$33'!)+@*3@$4!)+!(#$!5$'6)5,(*5;!(5$$!
?!(;6$!D!6+$&0*1;($'<!(;6$!DD!
6+$&0*1;($'<!,+4!13,5,!1$33'7!";6$!DD!6+$&0*1;($'!,5$!5$'6*+')/3$!2*5!'$15$()+.!6&30*+,5;!'&52,1(,+(7!D+!,!+$*+,($<!
6&30*+,5;!'&52,1(,+(!65*4&1()*+!/$.)+'!,5*&+4!>$$-!Ya<!/&(!0,(&5$!3$@$3'!,5$!*+3;!5$,1#$4!,(!>$$-!ZR7!N*!,!/,/;!/*5+!
65)*5!(*!ZR!>$$-'!)'!,(!5)'-!2
*5!5$'6)5,(*5;!4)'(5$''!';+45*0$!4&$!(*!3*>!3$@$3'!*2!'&52,1(,+(7!T+*(#$5!2,1(!(*!-+*>!,/*&(!
(;6$!DD!6+$&0*1;($'!)'!(#,(!(#$;!,5$!65$1&5'*5'!(*!(;6$!D!6+$&0*1;($'!,+4!(#$;!65*3)2$5,($!4&5)+.
!3&+.!4,0,.$7!
l&$'()*+'!1,+! /$!,'-$4!,/*&(!(#$'$!3$''!1*00*+3; H($' ($4!2$,(&5$'!*2!(;6$!DD!6+$&0*1;($'7!
!
!
Take home point:!";6$!DD!6$+&0*1;($'!,5$!5$'6*+')/3$!2*5!'&52,1(,+(!65*4&1()*+<!65*3)2$5,()+.!4&5)+.!3&+.!4,0,.$<!,+4!
,5$!65$1&5'*5'!(*!(;6$!D!6+$&0*1;($'7! !

39. 9yo girl with poor growth during the past year. < 3rd percentile for height and at 10th percentile for weight. PE
normal. Visual field testing shows bitemporal hemianoia. Labs show GH deficiency. MRI shows calcified cystic mass in
suprasellar region. Tumor derived from?
- Diverticulum of the roof of the embryonic oral cavity

Why it’s right: "#$!4),.+*')'!)'!,!15,+)*6#,5;+.)*0,<!>#)1#!)'!,!/$+).+!(&0*5!*2!1#)34#**47!D(!1,+!/$!0,4$!*2!/*(#!1;'()1!
,+4!'*3)4!1*06*+$+('!,3*+.!>)(#!1#*3$'($5*3!15;'(,3'!,+4!1,31)&0<!,!1#,5,1($5)'()1!2)+4)+.!*+!O\D7!T+($5)*5!$I($+')*+!*2!
(#$!(&0*5!*+(*!(#$!*6()1!1#),'0!15$,($'!,!/)($06*5,3!#$0),+*6),7!\,(#-$9'!6*&1#!)'!,+!$0/5;*3*.)1,3!'(5&1(&5$!(#,(!
2*50'!25*0!(#$!)+@,.)+,()*+!U,-,!4)@$5()1&3&0!6)+1#$'!*22V!*2!(#$!*5,3!$1(*4$507!b&5)+.!2*50,()*+<!\,(#-$9'!6*&1#!
1*+()+&$'!(*!4$@$3*6!)+ (*!(#$!,+($5)*5!6)(&)(,5;!U,-,!(#$!,4$+*#;6*6#;')'<!@$5'&'!(#$!+$&5*#;6*6#;')'!*5!6*'($5)*5!
6)(&)(,5;V7!D+!(#)'!6,()$+(<!+*(!*+3;!,5$!(#$5$!1*065$'')@$!2)+4)+.'!*2!(#$!(&0*5!U/)($06*5,3!#$0),+*6),V<!/&(!,3'*!').+'!
*2!,!0,32&+1()*+)+.!,+($5)*5!6)(&)(,5;!.3, +4!5$'&3()+.!)+!.5*>(#!#*50*+$!4$2)1)$+1;7!"#$!6,()$+(!)'!)+!(#$!@$5;!3*>!
6$51$+()3$!2*5!#$).#(!,+4!>$).#(!4&$!(*!3,1-!*2!.5*>(#!#*50*+$7!! !
!
Take home point:!T!15,+)*6#,5;+.)*0,!)'!,!/$+).+<!0)I$4!(&0*5!*2!1#)34#**47!b&5)+.!$0/5;*.$+$')'<!(#$!*5,3!
$1(*4$50!)+@,.)+,($'!(*!2*50!\,(#-$9'!6*&1#!(#,(!.*$'!*+!(*!/$1*0$!(#$!,+($5)*5!6)(&)(,5;!.3,+4!U,-,!
,4$+*#;6*6#;')'V7! !
!

40. 28yo F G1P1 with 2-day history of a painful mass in her right breast. Delivered healthy female newborn 3 weeks ago,
and breastfeeding since. T 37C, PE shows 3-cm tender mass surrounded by an area of erythema beneath the right areola.
Causal org?
- Staphylococcus aureus

Why it’s right: "#$!4),.+*')'!)'!,1&($!0,'()()'<!>#)1#!)'!,!/5$,'(!,/'1$''7!"#$!6#;')1,3!2)+4)+.'!,5$!1*+')'($+(!>)(#!,+!
,/' 1$''!>)(#)+!(#$!'-)+!,+4!'&/1&(,+$*&'!()''&$! ?!6,)+<!'>$33)+.<!,+4!$5;(#$0,7!T+*(#$5!(;6)1,3!2)+4)+.!1*+')'($+(!>)(#!
,+!,/'1$''!)'!:23&1(&,()*+<=!*5!,!>,33$4 H*22K4$0,51,($4!,5$,!*2!'>$33)+.!(#,(!23&1(&,($'!*5!0*@$'!'3).#(3;!>#$+!6,36,($47!
T+!,/'1$''!)'!)+! $''$+1$!,!>,33$4H*22!)+2$1()*+!2&33!*2!)+23,00,(*5;!1$33'!(#,(!,'!,!>#*3$!0,-$!&6!6&'7!"#$!*(#$5!@$5')*+!
*2!(#)'!%&$'()*+!)'!.)@)+.!(#$!4),.+*')'!,+4!,'-)+.!(#$!+$I(!/$'(!'($6Q!T+'>$5!?!1*+()+&$!/5$,'(2$$4)+.!')+1$!(#$! /,1($5),!
,5$!+*(!#,502&3!(*!/,/; 7!!
!
Take home point:!T1&($!0,'()()'!65$'$+('!,'!,!6,)+2&3<!$5;(#$0,(*&'!0,''!4&5)+.!/5$,'(2$$4)+.!,+4!)'!1,&'$4!/;!N7!
,&5$&'7!!
16 ©Test Pirates, LLC. All Rights Reserved.
Step 1 Practice Test #17 Explanations

41. 3yo boy with bacterial colitis caused by Salmonella enterica serovar arizonae. Which factor accounts for recruitment
of PMNS to inflammatory site by intestinal epithelial cells?
- Interleukin-8 (IL-8)

Why it’s right: "#$5$!,5$!(#5$$!0,)+!1;(*-)+$'K1#$0*(,1()1!2,1(*5'!
(*!5$0$0/$5! (#,(!5$15&)(!+$&(5*6#)3'm
!DLHh<!_R,<!,+4!
L$&-*(5)$+$!Ai7!"#$'$!2,1(*5'!,5$!65*4&1$4!(*!,((5,1(!+$&(5*6#)3
'!,+4!13$,5!,+!)+2$1()*+7!DL
Hh!)'!'$15$($4!/;!

17 ©Test Pirates, LLC. All Rights Reserved.


Step 1 Practice Test #17 Explanations

Why it’s right: "#)'!6,()$+(!#,'!,+!$I($5+,3!#$0*55#*)4!/$1,&'$!#$!#,'!6,)+7!C$+$5,33;<!$I($5+,3!#$0*55#*)4'!1,&'$!6,)+!


>#)3$!)+($5+,3!*+$'!,5$!6,)+3$''7!A*(#!1,+!/$!'$$+!$I($5+, 33;<!#*>$@$5<!>#$+!)+($5+,3!#$0*55#*)4'!65*3,6'$7!
W$0*55#*)4'!5$'&3(!25*0!6**3)+.!*2!/3**4!)+!,!4)3,($4!*5!0,32&+1()*+)+.!@$)+7!d#$+!/3**4!13*('!U4&$!(*!'(,')'V<!)(!15$,($
,!(#5*0/*')'<!,+4!(#)'!(#5*0/*')'!)'!>#,(!1,&'$'!6,)+7!T+,(*0)1,33;<!$I($5+,3!#$0*5 5#*)4'!1*0$!25*0!(#$!)+2$5)*5!5$1(,3!
@$)+!>#)1#!#,'!,+,'(*0*'$'!>)(#!(#$!6*5(,3!@$+*&'!';'($0!U(#$!'&6$5)*5!5$1(,3!@$)+V7!D+($5+,3!#$0*55#*)4'!1*0$!25*0!
(#$!'&6$5)*5!5$1(,3!@$)+7!"#$5$!,5$!(#5$$!0,)+!6*5(,3 H';'($0)1!,+,'(*0*'$'!(*!-+*>!2*5!(#$!MNOLJm!3$2(!.,'(5)1!U6*5(,3V!
(*!(#$!$'*6#,.$,3!U';'($0)1V<!6,5,&0/)3)1,3!(*!(#$!'&6$52)1),3!,+4!)+2$5)*5!$6).,'(5)1!/$3*>!(#$!&0/)3)1&'!,+4!(#$!
'&6$5)*5!$6).,'(5)1!,+4!3,($5,3!(#*5,1)1!,/*@$!(#$!&0/)3)1&'<!,+4!(#$!'&6$5)*5!5$1(,3!>)(#!(#$!0)443$!,+4!)+2$5)*5!5$1(,
@$ )+'7!!
!
Take home point:!JI($5+,3!#$0*55#*)4'!1,&'$!6,)+!,+4!,5$!4&$!(*!(#5*0/*')'!)+!(#$!)+2$5)*5!5$1(,3!@$)+<!>#)3$!)+($5+,3!
#$0*55#*)4'!,5$!6,)+3$''!,+4!1*0$!25*0!(#$!'&6$5)*5!5$1(,3!@$)+7! !
!!

45. 6-week-old girl with 6-day hx of vomiting small amount of milk 2 to 3 times daily. 50 %ile for length and weight.
Cause?
- Immature lower esophageal sphincter (LES)

Why it’s right: "#$5$!,5$!,!2$>!(#)+.'!(#,(!1,+!1,&'$!@*0)()+.!)+!,!+$>/*5+!,2($5!2$$4)+.7!"#$!*+$'! (#$!MNOLJ!3)-$'! (*!
,'-!,/*&(!)+13&4$!6;3*5)1!'($+*')'!,+4!4&*4$+,3!,(5$'), 7!T+!,++&3,5!6,+15$,'!1,+!,3'*!1&(!*22!(#$!4&*4$+&0!,+4!1,&'$!
@*0)()+.!,2($5!2$$4)+.7!T33!*2!(#$'$!1,&'$'!,5$!'$@$5$<!5$%&)5$!'&5.$5;<!,+4!>*&34!5$'&3(!)+!,!L8d!>$).#(!,+4! #$).#(!H!
u`F!6$51$+()3$7!d#$+!(#$!MNOLJ!.)@$'!;*&!,!+*50,3 !2)+4)+.!3)-$!(#$!RF!6$51$+()3$!2*5!#$).#(!,+4!>$).#(<!)(!'#*&34+9(!/$!
).+*5$4!/$1,&'$!(#$;!,5$!4)5$1()+.!;*&!(*>,54'!(#$!,+'>$5!(#,(!)+4)1,($'!,!0)34$5!4)'$,'$!65*1$''7!T!+$>/*5+!>)(#!
(#
6;3*5)1!'($+*')'!>)33!g8"!/$!)+!(#$!RF !6$51$+()3$!2*5!#$).#(!,+4!>$).
#(<!'*!(#)'!1*&34!+*(!/$!(#$!,+'>$57!L$''!
1*+'$%&$+(),3K#,502&3 !1,&'$'!*2!@*0)()+.!)+13&4$!)00,(&5)(;!*2!(#$!LJN!*5!0)3-H65*($)+!,33$5.;7!B*5!(#$!3,(($5<!,!6,()$+(!
>*&34!,3'*!3)-$3;!65$'
$+(!>)(#!,33$5.)1!'-)+!1#,+.$'<!'*!(#$!6,()$+(!65$'$+($4!)+!(#$!%&$'()
*+!3)-$3;!#,'!)00,(&5)(;!*2!(#$!
LJN7 !
!
Take home point:!D00,(&5)(;!*2!(#$!LJN!1,&'$'!@*0)()+.!)+!+$>/*5+'!(;6)1,33;!>)(#*&(!1#,+.$'!)+!#$).#(!*5!>$).#(7!D2!
(#$5$!)'!@*0)()+.!63&'!3*>!>$).#(!,+4!#$).#(<!'&'6$1(!,!0*5$!'$@$5$! 4)'$,'$ !3)-$!6;3*5)1!'($+*')'!*5!
4&*4$+,3!,(5$'),7!!

46. 37yo M with 4-day hx of diarrhea and abdominal pain, worse in past 24h, with watery-brown stools. Completed a 10-
day course of amoxicillin for a sinus infection 5 days ago. Stool shows: Fecal fat negative; ova and parasites negative; Cx
for infection negative; C. Diff toxin positive. Which pathologic findings present in intestinal tract?
- Pseudomemnbranes of fibrin and inflammatory debris

Why it’s right: _3*'(5)4)&0!4)2)1)3$!)'!,!.5,0!6*')()@$!/,1($5),!(#,(!1,&'$'!'$@$5$!4),55#$,!
, +4!6'$&4*0$0/5,+*&'!1*3)()'!
,2($5!,+()/)*()1!&'$<!$'6$1),33;!,06)1)33)+!,+4!13)+4,0;1)+7!
"#)'!6,()$+(!>,'!(5$,($4!>)(#!,0*I)1)33)+!2*5!,!')+&'!)+2$1()*+!
,+4!'&/'$%&$+(3;!4$@$3*6$4!4),55#$,!R!4,;'!,2($57!"#$!/,1($5),!'$15$($'!Y!(;6$'!*2!(*I)+'m!"*I)+!T<!$+( $5*(*I)+<!(#,(!
/)+4'!(*!(#$!/5&'#!/*54$5!*2!(#$!.&(G!,+4!(*I)+!A<!1;(*(*I)+<!(#,(!4$'(5*;'!(#$!1;(*'-$3$(,3!'(5&1(&5$!*2!$+($5*1;($'7!"#$!
5$'&3(!)'!(#$!2*50,()*+!*2!2)/5)+*&'<!63,%&$
H3)-$!0$0/5,+$'!2)33$4!>)(#!+$15*')'<!2)/5)+<!$I&4,($'<!,+4!3$&-*1;($'!&'&,33;!
)+!(#$!1*3*+7!b),.+*')'!)'!0,4$!/;!(#$!4$($1()*+!*2!1H4)2!(*I)+7!B)5'(!,+4!'$1*+4H3)+$!(5$,(0$+(!)'!0$(5*+)4,[*3$G!)2!
)+2$1()*+!6$5')'('<!*5,3!@,+1*0;1 )+!'#*&34!/$!&'$4!U+*(!DX!@,+1*0;1)+ V7!
!
!
Take home point:!_3*'(5)4)&0!4)2)1)3$!1,&'$'!'$@$5$!4),55#$ ,!,+4!6'$&4*0$0/5,+*&'!1*3)()'!,2($5!,+()/)*()1!&'$! ?!
(;6)1,33;!,06)1)33)+!*5!13)+4,0;1)+7!"#$!6'$&4*0$0/5,+$'!,5$!63,%&$ H3)-$!0$0/5,+$'!2)33$4!>)(#!+$15*')'<!2)/5)+<!,+4!
$I&4,($7!!

18 ©Test Pirates, LLC. All Rights Reserved.


Step 1 Practice Test #17 Explanations

47. 28yo M in ED 30 minutes after SOB. 3-year hx of cocaine abuse. T 38.1, P 100/min, BP 150/45. PE: diminished pulses
in left upper extremity. Crackles heard over all lung fields. 2/6 diastolic murmur at left sternal border. CXR shows a
widened aortic arch. Dx?
- Dissecting aneurysm

Why it’s right: _*1,)+$!)'!,!'()0&3,+(<!0$,+)+.!)(!)+15$,'$'!#$,5(!5,($<!/3**4!65$''&5$<!,+4!,3$5(+$''7!
_#5*+)1!1*1,)+$!&'$!
3$,4'!(*!1,54),1!,55#;(#0),'<!0;*1,54),3!)+2,51()*+<!,+4!4)3,($4!1,54)*0;*6,(#)$'<!(*!+,0$!,!2$>7!"#$!'(5$''!65*4&1$4!
/;!$3$@,($4!W\!,+4!Ae!3$,4'!(*!4)3,()*+!*2!/3**4!@$''$3'<!,+4!)+!(#)'!6,()$+(<!*11&55$4!,(!(#$!,*5()1!5**(!,+4!,'1$+4)+.!
,*5(,7!"#$!>$,-+$''!*2!(#$!3,;$5'!0,-)+.!&6!(#$!4)3,($4!,*5()1!>,33!,1&($3;!3$4!(*!,!4)''$1()*+7!"#$!4)''$1()*+!)+!(#)'!
1,'$ !
'(,5('!,2($5!(#$!'$1*+4!0,)+!/5,+1#!*2!(#$!,*5()1!,51#7!\$0$0/$5!(#$!,*5()1!/5,+1#$'!)+!*54$5m!/5,1#)*1$6#,3)1!,5($5;<!
3$2(!1*00*+!1,5*()4!,5($5;<!3$2(!'&/13,@),+!,5($5;7!W$5$<!(#$!2,3'$!3&0$+!'(,5('!/$2*5$!(#$!3$2(!/5,1#),3<!,+4!(#$5$2*5$!
2$$4'!(#$!3$2(!'&/13,@),+!,5($5;!>)(#!,!0&1#!3*>$5!65$''&5$<!5$'&3()+.!)+!4)0)+)'#$4!6&3'$'!)+!(#$!&66$5!$I(5$0)(;!*+!(#$!
3$2(7!D2!(#$5$!>$5$!4)0)+)'#$4!6&3'$'!)+!/*(#!,50'<!(#$+!(#$!4)''$1()*+!'(,5($4!
$,53)$5!
/$2*5$!(#$!2)5'(!/5,+1#!*2!(#$!,*5(,7!
"#$!0&50&5!(#,(!)'!#$,54!)'!,*5()1!5$.&5.)(,()*+!U4),'(*3)1!0&50&5!#$,54!/$'(!,(!(#$!3$2(!'(
$5+,3!/*54$5 <!>)4$+$4!6&3'$!
65$''&5$V<!,+4!(#,(9'!4&$!(*!)+@*3@$0$+(!*2!(#$!,+$&5;'0!,+4!4)''$1()*+!,(!(#$!,*5()1!5**(7!"#$!T\!>,'!3)-$3;!3*+. H
'(,+4)+.!U4&$!(*!(#$!,+$&5;'0!65)*5!(*!(#$!,1&($!4)''$1()*+V!,+4!+*>!(#$!6,()$+(!#,'!6&30*+,5;!$4$0,!'$1*+4,5;!(*!
(#$!,*5()1!5$.&5.)(,()*+!,+4!,1&($!4)''$1()*+7!
B)+,33;<!>)4$+$4!,*5()1!,51#!*5!>)4$+$4!0$4),'()+&0!)+!.$+$5,3!*+!_q\!
)+4)1,($'!(#$!65$'$+1$!*2!,!4)''$1()*+7!!
!
Take home point:!T1&($!,*5()1!4)''$1()*+!1,+!*11&5!,(!(#$!')($!*2!,+!,*5()1!,+$&5;'0<!,+4!65$'$+ ('!>)(#!';06(*0'!*2!

19 ©Test Pirates, LLC. All Rights Reserved.


Step 1 Practice Test #17 Explanations

50. 55yo M with chronic bronchitis in ED after being unresponsive. Found bottles of albuterol, ampicillin, codeine, and
theophylline bedside. T 37.2 C, p 112/min, respirations are 6/min, BP 95/60. Acute Rx should include?
- Naloxone

Why it’s right: "#)'!6,()$+(!#,'!,+!$3$@,($4!#$,5(!5,($<!3*>! /3**4!65$''&5$<!/&(!0*'(! +*(,/3; <!5$'6)5,(*5;!4$65$'')*+7!D+!
'&1#!'$@$5$!2*50'!*2!5$'6)5,(*5; !4$65$'')*+<!,3>,;'!'&'6$1(! *6)*)4!*@$54*'$7!"#)'!6,()$+(!*@$54*'$4!*+!'$@$5,3!
0$4)1,()*+'<!,+4!(#$!*+$!(#,(!>*&34!1,&'$!'$@$5$!5$'6)5,(*5;!4$65$'')*+!)'!1*4$)+$<!,+!*6)*)47!86)*)4!(*I)1)(;!)'!(5$,($4!
>)(#!+,3*I*+$<!,!+*+H'$3$1()@$!,+4!1*06$()()@$!*6)*)4!5 $1$6(*5!,+(,.*+)'(7!T40)+)'(5,()*+!*2!+,3*I*+$!5$@$5'$'!(#$!
$22$1('!*2!*6)*)4'!,+4!,3'*!>)33!%&)1-3;!0,-$!,!6,()$+(!.*!)+(*!*6)*)4!>)(#45,>,37! "#)'!0$4)1,()*+!>*&34!/$!,40)+)'($5$4!
DX!)+!(#$!Jb<!/&(!>#$+!()0$!)'!*2!(#$!$''$+1$!,+4!)2!(#)'!6,()$+(!>,'!2*&+4 !,(!#*0$<!)+(5,0&'1&3,5!+,3*I*+$!'#*&34!/$!
.)@$+7!
"#$*6#;33)+$!#,'!,!+,55*>!(#$5,6$&()1!)+4$I !U)+!*(#$5!>*54'<!)(9'!$,';!(*!1,&'$!')4$!$22$1('!>)(#!*+3;!,!'3).#(3;!
#).#$5!4*'$!(#,+!+*50,3V<!/&(!)(!0*5$!3)-$3;!(*!1,&'$!1,54)*(*I)1)(;!,+4!+$&5*(*I)1)(;!,1&($ 3;7!
!
!
Take home point:!"#$!#,330,5-!*2!*6)*)4!*@$54*'$!)'!5$'6)5,(*5;!4$65$'')*+7!"#$!,+()4*($!)'!+,3*I*+$ !.)@$+!DX!*5!DO7!!

20 ©Test Pirates, LLC. All Rights Reserved.
















Practice Test Explanations
Step 1 Practice Test # 17 – Block 2
Question Total: 50

Step 1 Practice Test #17 Explanations


Block 2

1. 50yo man has persistent cough for 2 months. He has had a 5 kg (11 lb) weight loss. He is a farmer and on itraconazole
4 weeks for histoplasmosis from chicken coops. Medications: hydrochlorothiazide, enalapril, atenolol, omeprazole, and
metoclopramide for hypertension and gastroesophageal reflux. Thin. Physical exam otherwise normal. Itraconazole
interacts with which drug and account for lack of effect of itraconazole?
- Omeprazole

Why it’s right: This is really a trick question because when you read it you may begin to think about P450 inducer and
inhibitors. However, both itraconazole (part of family with ketoconazole) and omeprazole are P450 inhibitors, meaning
the concentration of the drug remains in the body longer because it isn’t broken down by the P450 system in the liver.
But this patient is still suffering from histoplasmosis based on persistent cough and body habitus. Omeprazole, though a
P450 inhibitor, also inhibits the absorption of medications through the gut because it buffers stomach acid and prevents
breakdown of some drugs that is required for their absorption.

Take home point: Omeprazole changes the pH of the stomach and can prevent the absorption of certain drugs that
require acidic breakdown to be absorbed. Omeprazole is also a P450 inhibitor, increasing the effect and concentration of
other drugs.

2. 20yo man with 6-hour difficulty breathing and vomiting. 10-year history of type 1 diabetes on insulin. Pulse 90,
respirations 30 and deep, bp 90/60. Physical shows dehydration. Labs: Na+ 130, K+ 6.5, HCO3 5, glucose 500, pH 7.2,
pCO2 25 mm Hg. Which compound stimulated hormone-sensitive lipase in adipocytes causing accumulation of
metabolites causing acidosis?
- Epinephrine

Why it’s right: This patient has diabetic ketoacidosis (DKA), more common in type 1 diabetics (type 2 are more likely to
demonstrate hyperosmolar hyperglycemia when glucose is elevated). Signs of DKA include Kussmaul breathing (deep
and rapid breathing, causing a sensation of shortness of breath), nausea and vomiting, abdominal pain, delirium, fruity-
smelling breath due to excess ketones. The underlying pathophysiology of DKA is due to the fact that the body cannot
utilize glucose (due to lack of insulin), and enters starvation mode by breaking down fat stores for energy, releasing free
fatty acids (causes an acidosis) and ketones. Adipose cells release energy via hormone-sensitive lipase. Adrenaline
including epinephrine stimulates lipase in adipose cells.

Take home point: In DKA, epinephrine (adrenaline) stimulates hormone-sensitive lipase to breakdown fat stores in the
body, releasing ketones and free fatty acids.

3. Physician sad to inform patient of progression of carcinoma to the terminal phase. Physician's face makes patient cry
and ask, "It's bad news, isn't it?" Which is best response?
- "Yes, it is."
2 ©Test Pirates, LLC. All Rights Reserved.
Step 1 Practice Test #17 Explanations


Why it’s right: There will almost always be a question about how to deliver bad news to patients on the USMLE, if not
this one, then next USMLE. Giving patients bad news is a skill just like any other part of medicine and requires a few
main points to remember. The first is be straightforward. This patient is asking, is it bad news? There is no other answer
than being direct and saying yes, because a “no” would be lying, and yes, but the cure is around the corner, would also
be lying in part. That brings up another point – don’t give false hope or expectations. Research regarding a cure for
terminal diseases could be still in the bench-work phase, not even in human/clinical trial yet, requiring years before such
a cure is made available. A final key point is to not use too much medical jargon. Medical and scientific terminology can
be confusing for someone who isn’t listening to bad news, let alone a patient who may be in shock over the terminal
diagnosis.

Take home point: Delivering bad news requires the physician to be direct, to not give false hope about treatments not
readily available, and to use limited medical jargon.

4. 75yo man 2-year history of decreased force of urinary stream, urinate several times throughout night. BUN 55, Cr 5.0.
Ultrasound shows bilateral hydronephrosis and dilated ureter. Mechanism of renal failure?
- Increased hydrostatic pressure in Bowman space

Why it’s right: The diagnosis is urinary retention likely due to an enlarged prostate – benign prostatic hyperplasia. The
urine has been building up over the last 2 years, and therefore chronically, has led to hydronephrosis and dilated
ureters. Renal failure (evidenced by elevated BUN and creatinine) is due to postrenal azotemia, which only develops
with bilateral ureter obstruction. Other causes of obstruction include kidney stones, neoplasia, and congenital
anomalies. The urine builds up all the way into the renal pelvis and into the nephron subunit of the kidney. The specific
place that increases in pressure and is upstream of urinary output through the renal pelvis is Bowman’s space, where
blood is filtered and the substance that is filtered out of blood enters Bowman’s à proximal tubule à loop of Henle à
distal convoluted tubule à collecting tubule à renal pelvis.

Take home point: Postrenal azotemia (a form of renal failure) leads to renal failure via increased hydrostatic pressure in
Bowman’s space.

5. 4yo boy two bacterial urinary tract infections past year. Physical exam normal. Radiologic studies show dilation of left
ureter and renal pelvis, minimal left-sided renal function. Left nephrectomy done. Photo: dilated renal pelvis and ureter.
Which is cause of renal disease?
- Congenital ureteral obstruction

Why it’s right: This patient has congenital stenosis of one of the ureters likely due to failure of ureter canalization in
utero. Repeated urinary tract infections are common in these patients due to urinary stasis, which promotes bacterial
colonization. The urine then builds up before the point of stenosis, into the urinary pelvis, and into the kidney leading to
kidney failure from post-renal azotemia. Refer to above question explanation for more detailed information.

Take home point: Congenital ureteral obstruction is a cause of repeated urinary tract infections in young patients and
renal failure due to post-renal azotemia.

6. 45yo man poorly controlled type 2 diabetes 1-month low-grade fever. Getting hemodialysis for end-stage renal
disease. T 37 C (98.6 F), pulse 72, bp 144/92. Physical subclavian catheter below right clavicle. Lungs clear. Cardiac exam
no murmurs. Blood cultures grow nonhemolytic, catalase-negative, gram-positive cocci in pairs and chains. Which
3 ©Test Pirates, LLC. All Rights Reserved.
Step 1 Practice Test #17 Explanations

organism?
- Enterococcus faecalis

Why it’s right: Asymptomatic bacteremia is not uncommon in patients with chronic blood catheters, and risk factors for
infection are compounded in poorly controlled diabetics, like in this patient. Though the patient doesn’t have signs of
sepsis (his BP, HR and temperature are all normal), his blood cultures are positive likely from inoculation at the site of
the subclavian catheter. Another sign of bacteremia is a low-grade fever that waxes and wanes. The gram-positive cocci
bug described as nonhemolytic, catalase-negative, and in pairs and chains is most likely enterococcus faecalis.

Take home point: Indwelling blood catheters put patients at risk for bacteremia and sepsis. Enterococcus faecalis can
cause catheter-associated infections and is a nonhemolytic, catalase-negative, gram positive cocci found in pairs and
chains.

7. 3yo boy sickle cell disease with fever and pain over left foot 3 weeks. Hematocrit stable. Leukocyte count 15,000
predominance of neutrophils. Which is most likely explanation for findings?
- Osteomyelitis

Why it’s right: In terms of bone pathology, sickle cell patients are at increased risk for both osteonecrosis and
osteomyelitis. Distinguishing between the two can prove to be a challenge. In this patient, the patient has signs of
infection including fever and elevated WBC with a neutrophil predominance indicating acute infection. The most
common cause of osteomyelitis is staph aureus, but in sickle cell patients it’s salmonella.

Take home point: Signs of osteomyelitis include elevated WBC (predominance of neutrophils), fever, and bone pain. In
sickle cell patients, salmonella is the most common cause.

8. 10yo girl well-child examination. No menstrual period. 50th percentile height and weight. Physical exam absence of
breast bud development and no pubic or axillary hair. It is most appropriate to tell mother that first objective sign of
puberty will be which?
- Breast bud development

Why it’s right: There will almost always be a question on the Tanner stages of development in boys and/or girls on the
USMLE step 1. It is very high yield. For both boys and girls, the stages must be memorized. The higher yield ones are the
first and last stages for boys and girls. The first sign of puberty in a girl is breast bud development. It is actually tanner
stage 2 (breast buds) since stage 1 for boys and girls is pre-pubescence. In boys, the first sign of puberty is enlargement
of the testes and increased pigmentation of the scrotal sac. The final stages of development in both boys and girls is an
adult distribution of pubic hair. If you just memorize these above facts, you have a pretty good chance of answering
most of the questions on the Tanner stages of development.

Take home point: Memorize the Tanner stages of puberty! – or just the highest yield ones: The first sign (the second
stage) of puberty in girls is breast buds; the first sign (second stage) of puberty in boys is testicular enlargement and
scrotal skin pigmentation.

9. 45yo woman follow-up exam after 8 weeks’ tamoxifen therapy for estrogen- and progesterone-positive invasive ductal
carcinoma of breast. 50yo sister also hormone-sensitive breast cancer. Physical exam normal. Serum decreased
concentrations of endoxifen, active metabolite of prodrug tamoxifen. Genetic analysis shows homozygous cytochrome

4 ©Test Pirates, LLC. All Rights Reserved.


Step 1 Practice Test #17 Explanations

P450 2D6*4 alleles. Likelihood patient's sisters has same alleles?


- 25%

Why it’s right: The question is asking about inheritance of an autosomal recessive genetic mutation and putting the
question in a clinically-relevant scenario. You don’t have to understand the scenario to answer the question, however it
is helpful. Basically, the patient doesn’t metabolize tamoxifen as fast as others because of a mutation in the P450
system. Therefore, there is less of the active metabolite, which is relevant because tamoxifen is a prodrug and in order
to become active, it needs to be metabolized. Now for the Punnett square analysis: this patient has the phenotype by a
homozygous genotype mutation – since it is homozygous for the mutant, it must be autosomal recessively inherited.
Because if it was heterozygous for the mutation and the patient still had the phenotype, then it would be of autosomal
dominant inheritance. A Punnett square for an autosomal recessive disease yields the following, A=wildtype, a=mutant:

A a
A AA Aa
a Aa aa

For the sister to have the disease, she must also have the genotype aa. So the chances of that, as seen in the table, are 1
out of 4, or 25%.

Take home point: The chances of inheriting an autosomal recessive disease are 25% if each parent carries a mutant
allele but is not affected by the mutation.

10. 60yo man progressive shortness of breath past 3 months. Worked in shipyard. Respirations 25. Bilateral basilar
crepitus and crackles. Xray chest reticulonodular pulmonary infiltrates consistent with interstitial fibrosis. Picture: sputum
sample of elongate structures (ferruginous body). Fibrosis initiated by interaction of structures with which cell?
- Alveolar macrophage

Why it’s right: On the real test, they are much less likely to give you the history of shipyard work, since that fact just
gives this question away. The diagnosis is asbestosis and other jobs associated with this disease are roofing and
plumbing. Findings aside from this patient’s occupational history, though, point towards asbestosis – bilateral lower lobe
involvement (characteristic of asbestosis), a chronic history of shortness of breath, and the description/picture of the
ferruginous body, or a golden-brown fusiform rod shaped like a dumbbell. They are also known as asbestosis bodies and
they represent asbestosis fibers laden with hemosiderin. The initiating cell in the lungs that “cleans up” and identifies
foreign bodies, pathogens, etc., is the alveolar macrophage, which also leaves fibrosis in its wake. Type II pneumocytes
proliferate in lung damage and clara cells degrade toxins, but the initiation of fibrosis is the job of the macrophage.

Take home point: Ferruginous bodies are found in asbestosis and induce lung fibrosis initiated by alveolar macrophages.

11. 30yo woman 20 weeks’ gestation, uncomplicated pregnancy. Fundal height is greater than expected for gestational
age. Ultrasound shows increased amniotic fluid. Which abnormality is cause?
- Tracheoesophageal atresia

Why it’s right: Think about amniotic fluid like a cycle of in’s versus out’s – meaning, the fetus swallows amniotic fluid,
which passes through the fetus, and exists through the urinary tract. If the esophagus or other parts of the GI tract are
not working, fluid builds up. If the kidneys are not working or there is a block in the urinary tract, there is a lack of fluid
leaving the fetus. When the fetus is unable to swallow fluid due to T-E atresia, polyhydramnios results, increasing fundus
5 ©Test Pirates, LLC. All Rights Reserved.
Step 1 Practice Test #17 Explanations

height for which an ultrasound can be done to confirm excess amniotic fluid. When the kidneys are malfunctioning,
there is a lack of fluid to cushion the baby and a Potter’s syndrome follows (due to oligohydramnios).

Take home point: Tracheoesophageal atresia results in polyhydramnios, or excess amniotic fluid, while oligohydramnios
can cause Potter’s syndrome.

12. 43yo woman for health maintenance exam. Physician 30 minutes late for appointment because of an emergency,
when enters, patient checks watch. Which is best initial response?
- "I'm sorry I got delayed. I hope I haven't made you late somewhere else."

Why it’s right: Patients hate waiting, and it’s no wonder that the USMLE wants to train future physicians early on how to
handle patient expectations when it comes to wait times. The best way to handle a patient when the doctor is late is to
apologize, don’t ignore the fact that you’re late, and be empathetic – it was a great follow-up sentence to say, “I hope I
haven’t made you late somewhere else,” because likely, the patient does have multiple visits in one day and/or also has
places to be. Patients dislike it when a doctor thinks the doctor’s schedule is more important than the patient’s. It would
also not have been bad in this scenario to say that the physician was coming from an emergency.

Take home point: When a physician is late to a clinic appointment due to an emergency, the physician should respond
with an apology and empathetic statement. Giving the patient a reason for tardiness is optional if appropriate.

13. 27yo woman 12-hour history of fever and abdominal pain. History of recurrent urinary tract infections. Temperature
is 39 C (102.2 F). Physical exam tenderness of right flank. Abdominal xrays bilateral staghorn renal calculi. Urinalysis pH
8, many RBCs, WBC, bacteria. Calculi composed of?
- Struvite

Why it’s right: Also known as ammonium-magnesium-phosphate stones. All the hints for this type of kidney stone are
given – staghorn calculus = likely struvite; UTI = struvite, since the stone is the nidus for infection. Now struvite, stones
are not synonymous with staghorn calcli – other stones can form staghorn calculi, and struvite stones are not always the
shape of staghorn calculi. The bacteria common in these stones are the urease-positive bacteria: Proteus mirabilis,
Staphylococcus, and Klebsiella. Finally, these stones along with calcium phosphate stones form at a basic pH (pH>7.4).

Take home point: The most common causes of staghorn calculus stones are struvite stones, which also form in the
presence of urease-positive bacteria and can present along with a UTI.

14. 48yo nulligravid woman with excessive uterine bleeding for 3 months. Bleeding during menses and at irregular
intervals. Menses were regular before. BMI 27. Pelvic exam: adnexae are nonpalpable. Endometrial curettage shows
abundant tissue. Which is cause of symptoms?
- Endometrial hyperplasia

Why it’s right: The diagnosis of endometrial disorders and pathology can be narrowed down to one or two diagnoses
based on two main symptoms and findings on physical exam – pain only during menses (or present in between menses)
and an enlarged or normal sized uterus. For instance, if this patient had pain with menses and a normal sized uterus, the
diagnosis would more likely be endometriosis. If the uterus was enlarged, the diagnosis would be adenomyosis – and
this disease also causes pain between menses. Another important piece of the history is the patient’s age – 48yo which
is perimenopausal. If this patient has endometriosis, it would have likely presented prior to 48yo. Around menopause,
6 ©Test Pirates, LLC. All Rights Reserved.
Step 1 Practice Test #17 Explanations

hormone levels fluctuate and estrogen can actually increase especially if ovulation has stopped – leading to endometrial
gland proliferation and increased menses. Risk factors for endometrial hyperplasia include anovulatory cycles, hormone
replacement therapy, polycystic ovarian syndrome, and granulosa cell tumor. Endometrial hyperplasia does increase the
risk for endometrial carcinoma.

Take home point: An endometrial disorder that causes painless irregular menses around menopause is likely
endometrial hyperplasia.

15. 65yo woman surgical repair of aneurysm right internal carotid artery in cavernous sinus. Three days later, physical
exam right pupil larger than left pupil. Weakness of eye movement. Diagram of coronal section through middle cranial
fossa shown. Which nerve is damaged?
- A

Why it’s right: The aneurysm is compressing cranial nerve III (oculomotor nerve). CN III travels through the cavernous
sinus and can be affected by conditions that cause vascular compromise like uncontrolled diabetes or compressive
conditions like a tumor or aneurysm. The outer fibers of the nerve carry parasympathetic fivers that control pupillary
constriction and lens accommodation. The inner fibers of the nerve carry motor function responsible for eye movements
(including eyelid opening). Therefore, vascular compromise is more likely to affect the inner part of the nerve (due to
ischemia since the nerve is profused from the outside to the inside), resulting in motor defects, while compressive
phenomena cause pupillary malfunction (blown, nonreactive pupil). However, a compressive lesion large enough (like a
larger aneurysm) can affect first pupillary function and then motor function, like in this patient. Therefore, symptoms
are not mutually exclusive, meaning, a life-threatening aneurysm can still cause vascular-like changes, so it’s very
important to check pupils and lift a droopy eyelid!

The treatment here is immediate neurosurgery with aneurysm coiling.

Take home point: Cranial nerve III is responsible for pupillary constriction, lens accommodation, and eye movement and
eyelid elevation. A brain aneurysm pushes on the outer parts of the nerve, first inhibiting pupillary constriction and then
limiting eye movement and eyelid lift.

16. 55yo woman 6-week history low energy, irritability, crying spells, difficulty falling asleep, wakes up during night,
cannot focus. Taking lorazepam for 15y for generalized anxiety disorder. Taking estrogen replacement therapy for
postmenopausal symptoms. Vital signs normal. Physical exam normal. Mental status shows constricted affect. Mood is
"testy." Speech is slowed. No psychosis. Wishes she wouldn't wake up but doesn't plan to harm herself. Cause of
symptoms?
- Major depressive disorder

Why it’s right: This patient wishes she wouldn’t wake up, but doesn’t plan on harming herself – this is a prime example
of anhedonia and one of the hallmarks of major depressive disorder. At least 5 of the SIGECAPS must be present for 2 or
more weeks, and 1 of them must be depressed mood or anhedonia. In our patient: Sleep disturbance, Energy (loss of),
Interest (loss of, or anhedonia), Concentration (loss of), and Psychomotor retardation or agitation (slow speech). The
presence of a co-morbid mood disorder is not uncommon, and this patient also has generalized anxiety disorder. She is
also going through menopause, which results in hormonal changes, and mood disorders may become more apparent.

Take home point: To diagnose major depressive disorder, 5 of the SIGECAPS must be present, and one of them must be
either anhedonia or depressed mood.

7 ©Test Pirates, LLC. All Rights Reserved.


Step 1 Practice Test #17 Explanations

17. Ten years after total gastrectomy, 60yo man difficulty walking. Diffuse spasticity in arms and legs, impaired
proprioception in his feet, increased muscle stretch reflexes in arms and knees, absence of muscle stretch reflexes in
ankles, bilateral extensor plantar responses. Which vitamin is deficient?
- B12 (cobalamin)

Why it’s right: Loss of the stomach in a total gastrectomy = loss of parietal cells, which produce intrinsic factor needed
for B12 absorption at the terminal ileum. Therefore, this patient is B12 deficient evidenced by symptomatology
supportive of B12 deficiency – spasticity and extensor plantar responses consistent with an upper motor neuron lesion
(lateral corticospinal tracts) and impaired proprioception consistent with dorsal column lesions.

Take home point: History of a gastrectomy or terminal ileum resection/damage increases the risk for B12 deficiency due
to low levels of intrinsic factor needed for B12 absorption.

18. 80yo man type 2 diabetes 2-month history severe constipation. Laxatives haven't relieved symptoms. Abdominal
exam shows distention. Colonoscopy shows no abnormalities. Patient has dysfunction of which nerve?
- Pelvic splanchnic

Why it’s right: The diagnosis is neurogenic bowel dysfunction in the setting of (likely) uncontrolled diabetes. Patients
with diabetes have small-vessel disease, and this is the basis of their three main problems – retinopathy, nephropathy,
and neuropathy. Nerves need a blood supply too, and because nerves are small, they are fed by small blood vessels.
Unfortunately, these are the first vessels (the small ones) to go in diabetes (why the retinal blood vessels are also at risk,
because of their small caliber!). In terms of neuropathy, diabetics not only develop peripheral neuropathy and
decreased sensation in distal extremities, like their toes, but also have damage to the nerves that supply the GI tract.
Diabetes results in gastroparesis and, as a result, increased fullness after meals due to delayed gastric emptying. They
get constipation, like in this patient, due to damage to the nerves that supply the colon and rectum. The colon is
supplied by the vagus nerve up to the splenic flexure, while the pelvic splanchnic nerves supply the colon from the
splenic flexure all the way to the rectum.

Take home point: The colon is supplied by the vagus nerve up until the splenic flexure. The descending colon and
rectum are supplied by the pelvic splanchnic nerves.

19. 34yo man lightheaded after running 12 miles of marathon in the setting of a hot day. Pulse 130 bp 80/60. Which
changes to autonomic nervous system occurred?
- Sympathetic efferent activity increased, parasympathetic efferent activity decreased

Why it’s right: This patient is experiencing the beginning phases of shock due to volume loss evidenced by elevated
heart rate and low blood pressure. A hot day and running 12 miles for a marathon is a perfect storm for dehydration if
this patient did not properly hydrate prior to the race. Even before the blood pressure drops, the first sign of volume
loss/shock is elevated heart rate. Vasoconstriction is also occurring, which masks a low blood pressure due to low blood
volume. With increased volume loss, the blood pressure begins to drop. The stimulus for all these processes (increased
heart rate, vasoconstriction) is increased sympathetic output (the efferents) and decreased parasympathetics since
parasympathetics do the opposite and decrease heart rate.

Take home point: During volume loss or shock (i.e. in dehydration, hemorrhage), the sympathetic efferents increase
output to elevate heart rate and raise blood pressure while the parasympathetics efferents decrease signalling.

8 ©Test Pirates, LLC. All Rights Reserved.
Step 1 Practice Test #17 Explanations

20. 6-Mercaptopurine (6-MP) used to treat acute lymphoblastic leukemia (ALL). 6-MP acted on by enzymes to make 6-
thioguanine nucleotides (6-TGN). Efficacy and toxicity of 6-MP correlated with 6-TGN. 6-MP acted on by xanthine oxidase
(XO), thiopurine methyltransferase (TPMT), leading to inactive metabolites. Which are clinical consequences of ALL
patients homozygous for low-activity alleles of TPMT?
- They should be given lower doses of 6-MP

Why it’s right: This question tells us that an active metabolite of 6-MP is 6-TGN, and too much of 6-TGN can lead to
toxicity. 6-TGN is inactivated by two enzymes – XO and TPMT. With less activity of one of these enzymes, 6-TGN
increases, and the risk for toxicity increases. A homozygous allele for low activity of TPMT, means that 6-TGN will
increase to dangerous levels, therefore, patients with a mutation in either XO or TPMT should be given a lower dose of
6-MP. Heterozygous alleles may be given a normal dose, but caution for side effects should be exercised. You are
expected to know that XO and TPMT are enzymes that breakdown and inactivate substances.

Take home point: XO and TPMT are responsible for inactivating active metabolites of chemotherapeutic drug, 6-MP.
Patients with a mutation in the gene that code for these enzymes should be given lower doses of 6-MP.

21. Ten human subjects given new oral drug to monitor drug effect and toxicity. Blood analyzed for human
pharmacokinetics of drug for first time. Which trial type?
- Phase 1

Why it’s right: Know the phases of clinical trials! They will come up on Step 1, 2 and 3! Preclinical trials are animal
studies. Phase 1 trials are the first ones with humans (hence, numero uno), and are done with only a few patients to test
for efficacy, toxicity, and pharmacokinetics. A way to remember this is that if the drug is toxic, we are only testing it on a
few people (not ethical, but how else are we to test the drug?). Phase 1 is also done on healthy volunteers. Phase 2 is
with patients who have the disease to test efficacy, dosing, and adverse effects. Phase 3 is the largest pre-marketing trial
and compares the new treatment to the standard of care or a placebo. Phase 4 is the easiest to remember – it occurs
after the drug is already on the market – and is key because results from this phase can pull a harmful drug from the
market.

Take home point: Phase 1 clinical trials test just a few healthy people with the new drug/therapy to assess safety,
efficacy, and pharmacokinetics.

22. 15yo girl health maintenance exam. Mother dx squamous cell carcinoma face, maternal grandfather died of
metastatic melanoma. In patient this age, which factor most predicts compliance with photoprotection?
- Use of sunscreen by her peers

Why it’s right: This patient is at high risk of skin cancer, and her compliance with sunscreen will surely be influenced by
her family history, but even more, and sadly, by what her peers are doing. Use of sunscreen by peers will influence her in
a positive way and increase her protection from the sun. The most protective, and recommended sunscreen in this
patient, is a barrier type of sunscreen (i.e. one with zinc oxide in it) – this is another common version of the sunscreen
question. Now a sunscreen with zinc plus a color or tint in it meant to be used as makeup or foundation is a real million-
dollar idea!

Take home point: Patients at high risk for skin cancer should wear sunscreen at all times and limit sun exposure.
Sunscreens with zinc in them provide the greatest exposure because they provide barrier protection.

9 ©Test Pirates, LLC. All Rights Reserved.


Step 1 Practice Test #17 Explanations

23. 34yo AIDS patient with pulmonary tuberculosis. No CD4+ T lymphocytes in peripheral blood. Which cellular
components most likely to have deficient function in tuberculous lesions in lungs?
- Macrophages

Why it’s right: Tuberculosis is a mycobacterium that causes an immune reaction mediated by CD4+ T cells and
macrophages. On histopathology, tuberculosis results in caseating granulomas and the key players of the granulomas
are giant cells and macrophages. Because AIDS patients have limited helper T cells (or as in this patient, no detectable T
cells), they have a deficiency in T cell-mediated immune reactions that result from certain bacteria, viruses, and
mycobacteria. Because CD4+ T cells activate macrophages involved in the reaction to TB, there is a deficiency in the
activation of macrophages by defective T cells.

Take home point: CD4+ helper T cells activate macrophages in response to infection, particularly infection by
mycobacteria, viruses, and fungi. Patients with AIDS have limited immune response to TB due to decreased T cell
activation of macrophages.

24. 44yo woman follow-up after two Pap smears showing atypical squamous cells of undetermined significant. Test
shows viral E6 protein of human papillomavirus. This protein promotes cell growth and malignancy by causing cellular
p53 protein degradation. This degradation beings when p53 protein targeted to which type of cell enzymes?
- Ubiquitin ligase

Why it’s right: It is key to memorize the tumor suppressor genes and the proto-oncogenes (or the tumor “enhancer”
genes). Rb and p53 are the highest-yield and both tumor suppressor genes. Therefore, decreasing the genetic products
of these genes (their respective proteins) via a mutation, will lead to cancer (basically). This question is asking, which is
the way by which the protein product of p53 is degraded by cancer-promoting viral product of HPV? There are a few
ways in which proteins are degraded in general – through ubiquitin-tagged proteasome degradation, lysosomal
degradation, and calcium-dependent enzymes. For p53 protein, the primary way it is degraded is via ubiquitin ligase.

Take home point: p53 is a tumor suppressor gene. In proto-oncogenic states, p53 products are destroyed by ubiquitin
ligase.

25. 40yo woman hx of 6-month episodic sinusitis with 2-week intermittent headaches, fatigue, and generalized joint pain,
worsening cough productive of blood-tinged sputum. Failed antibiotics, decongestants and nasal corticosteroids. Physical
exam: erythema nasal mucosa, two small ulcerations. Lungs scattered crackles. Xray patchy bilateral opacicites. Labs:
Hemoglobin 13, ESR 70, leukocyte 10,500, anti-neutrophil cytoplasmic antibody increased. Dx?
- Wegener granulomatosis

Why it’s right: On the real test, they are less likely to use the eponym “Wegener granulomatosis” or “Nazi
granulomatosis” and more like to use the terms, granulomatosis with polyangiitis. This is an autoimmune disease of the
small vessels that results in a necrotizing vasculitis. The disease is high-yield in terms of renal disease and lung disease. It
also affects the upper airways (sinuses), unlike Good Pasture’s, which also results in hemoptysis and nephropathy but
does NOT affect the upper airways. The nephropathy of Wegener’s is characterized by a rapidly progressive (also known
as crescentic) glomerulonephritis. Ulcers within the upper airways are not uncommon and sufficient ulceration can lead
to collapse of the nasal septum with development of a “saddle nose.” Labs typically demonstrate c-ANCA positivity (c-
ANCA = PR3-ANCA).

10 ©Test Pirates, LLC. All Rights Reserved.


Step 1 Practice Test #17 Explanations

Take home point: Wegener granulomatosis, or granulomatosis with polyangiitis, affects the kidneys, upper AND lower
airways (unlike Good Pasture’s, which just affects the lower airways), and is associated with c-ANCA positivity.

26. 25yo woman 6-month history of joint pain poorly responsive to aspirin. Physical exam: bilateral swelling of proximal
interphalangeal joints, metacarpophalangeal joints, and wrists; weakness of grasp. Small nodules palpated beneath skin
around joints of fingers. Dx?
- Rheumatoid arthritis

Why it’s right: These are classic signs of rheumatoid arthritis (RA). Age of presentation is bimodal – 20s-30s and then
50s-60s. It affects the PIP and MCP joints (unlike osteoarthritis (OA) which affects DIP and PIP joints), and other small
joints including wrists and ankles (less like to affect knees, hips, and shoulders like in osteoarthritis). The small nodules
under the skin are subcutaneous rheumatoid nodules, which are small buildups of fibrinoid necrosis. Remember, -oid
means resembles or “is like.” For example, this practice test is USMLE-oid, or it’s like the USMLE, but not the USMLE.
Therefore, fibrinoid means it’s made of necrosis composed of cellular matter that is like fibrin.

Take home point: RA characteristically affects small joints including the PIPs (not DIP), MCPs, wrists and ankles vs. OA
which tends to affects larger joints like knees, hips, and shoulders.

27. 50yo woman 1-year hx of hot flashes and irregular menses. Decreased bone mineral density. Alendronate prescribed.
Mechanism of drug?
- Inhibition of osteoclast-mediated bone resorption

Why it’s right: Menopause is caused by ovarian “burn-out,” characterized by lack of ovulation and decreased estrogen
production by the ovaries. This estrogen loss results in a cascade of effects including hormonal changes (notably a rise in
LH and FSH), osteoporosis, hot flashes, irregular menses, vaginal atrophy, hirsutism, and coronary artery disease. These
patients have bone loss that can be mitigated by use of bisphosphonates, including alendronate. Alendronate is a
pyrophosphate analog that inhibits osteoclasts, which breakdown bone. Side effects of bisphosphates are another hot
topic – corrosive esophagitis and osteonecrosis of the jaw.

Take home point: The mechanism of action of alendronate is osteoclast inhibition, decreasing bone resorption. Side
effects include corrosive esophagitis and osteonecrosis of the jaw.

28. Female newborn at 36 weeks’ gestation has respiratory distress. Apgar 3 and 5 at 1 and 5 minutes. Physical shows
cyanosis. Endotracheal and NG tubes placed. Xray shows nasogastric tube in left hemithorax, mediastinum displacement
to right, absence bowel gas in abdomen. Which embryological event led to these findings?
- Incomplete formation of pleuroperitoneal membrane

Why it’s right: The diagnosis is congenital diaphragmatic hernia due to, the answer to the question, incomplete
formation of the pleuroperitoneal membrane, which is the embryonic precursor to the diaphragm. The reason for
herniation on the left side of the chest is because the liver blocks the intestines on the right. Therefore, the weaker point
is the left chest cavity, and the stomach and/or intestines can herniate into the chest. Typical findings include imaging of
the NG tube in the left chest cavity/hemithorax with bowel gas/fluid levels in the chest, absent bowel sounds on the left
abdomen, and mediastinal displacement rightward.

11 ©Test Pirates, LLC. All Rights Reserved.


Step 1 Practice Test #17 Explanations

Take home point: Congenital diaphragmatic hernias are due to incomplete formation of the pleuroperitoneal
membrane, which eventually forms the diaphragm.

29. 65yo women progressive vulvar itching past 2 months; miconazole for yeast infections ineffective. Exam: atrophy of
labia minora and thin, parchment-like skin over vulva and anus. Dx?
- Lichen sclerosus

Why it’s right: This is a USMLE Step 2 question, but here it is! Basically, menopause is the withdrawal from estrogen and
this estrogen loss results in vaginal atrophy. Continued vaginal atrophy can eventually result in lichen sclerosis, which is
more a skin condition of the vagina resulting in chronic itching, often misdiagnosed as a yeast infection, and creates fine,
lacey, white or parchment-like skin changes over the vulva. Histology shows hyperkeratosis and hyperplasia – a chronic
inflammatory component may also be present (i.e. lymphocytic infiltrate). Treatment is supportive therapy and
corticosteroids.

The last way they can trip you up on this question, and more likely to be on Step 2, is that this can also affect men; if
lacey, white skin changes are described on a male’s penis, this would also be the diagnosis.

Take home point: Lichen sclerosus is a pruritic skin condition that typically affects older women and appears as a lacey,
white, parchment-like rash on the vulva and anus.

30. 75yo man 2-day ear ringing, nausea, fatigue. Temp 37 C (98.6 F), pulse 100, respirations 24, bp 140/85. Physical: mild
epigastric tenderness. ABG pH 7.42 pCO2 30 pO2 95 HCO3 19. Dx?
- Salicylate poisoning

Why it’s right: This patient took too much aspirin, a salicylate. Toxicity from salicylates first results in a respiratory
alkalosis early on, and chronically, results in an anion gap metabolic acidosis. This ABG represents a respiratory alkalosis
with a concurrent and compensatory metabolic acidosis. The pH is alkalotic, but lower than would be expected for a
pCO2 of 30, meaning this is a compensated respiratory alkalosis. Aspirin first causes a respiratory alkalosis because it
directly stimulates the respiratory center in the brain, resulting in hyperventilation. Ear ringing is another hint because
salicylates are toxic to hearing cilia.

Take home point: Aspirin or salicylate toxicity causes a respiratory alkalosis early and a metabolic acidosis late. It also
causes tinnitus.

31. 15yo girl 1-day hx redness and painful skin following sunbathing. She used sunblock. No medications. Physical exam:
severe erythema of back and extremities, no blisters. Dx?
- First-degree burn

Why it’s right: Believe it or not, a sunburn of this magnitude actually qualifies as a first-degree burn. If there were
blisters, it would bump up to a second or third-degree burn. Just like a chemical burn or burn from boiling water/fluid,
etc., the sun can cause a similar type of burns, and the one described in this question – severe erythema on sun-exposed
areas – is a first-degree burn. Even if sunblock is utilized, UVA/B rays can still result in a burn. The ultimate block from
sun via a sunscreen is a barrier cream – i.e. zinc oxide.

Take home point: A sunburn that causes erythema and pain is a first-degree burn. +blisters = third-degree burn.

12 ©Test Pirates, LLC. All Rights Reserved.


Step 1 Practice Test #17 Explanations

32. 58yo man supraventricular tachyarrhythmia refractory to pharmacotherapy gets ablation of accessory excitatory
pathway in atrial endocardium. Which area should be avoided to leave sinoatrial (pacemaker) node intact?
- The junction of the superior vena cava and the right atrium

Why it’s right: The SA node is the pacemaker of the heart and is located at the junction of the superior vena cava and
the right atrium. This question is asked in a way to make is clinically applicable, but really they just want to know if you
know where the SA node is located in the heart. In ablative therapy for a heart arrhythmia, the excitatory heart tissue
where the premature beat is coming from is ablated. Usually the source of arrhythmia is supraventricular, or above the
ventricles. So, care should be taken to avoid the SA node when performing an ablation.

Another version of this topic/question that is frequently asked, is what becomes the pacemaker of the heart when the
SA node fails? What if the SA node was accidentally ablated and another pacemaker had to take over? Answer: AV node.
What if the AV node fails? Bundle of His/Purkinje fibers/ventricles.

Take home point: The SA node is located at the junction of the superior vena cava and the right atrium.

33. 81yo woman massive pulmonary embolism from deep venous thrombosis. Platelet count 160,000. Appropriate
pharmacotherapy is started. One week later, platelets 55,000. Thrombocytopenia most likely caused by a drug with
which of the following mechanism of action?
- Potentiates the action of antithrombin III

Why it’s right: This patient has heparin-induced thrombocytopenia (HIT) due to… heparin. Heparin acts as a cofactor for
antithrombin III and increases its action. Antithrombin inhibits thrombin, stopping the coagulation cascade, leading to
anticoagulation. Antithrombin III also inhibits factors VII, IX, X, XI, XII. This patient was appropriately started on heparin
for a DVT, but developed HIT 1 week later, which is the typical timeframe for HIT to occur after heparin initiation. HIT
results in thrombosis (either one or multiple thromboses) and thrombocytopenia. The next best question would be,
what do you do now? Start treatment with a heparin alternative: lepirudin or bivalirudin.

Take home point: The mechanism of action of heparin is activation of antithrombin III, leading to inhibition of thrombin
and anti-coagulation.

34. 22yo woman, g1p1, 2-day hx of fever, severe vaginal bleeding. 4 days ago delivered healthy male newborn. Temp
38.1 C (100.6 F). Pelvic exam: open cervix, heavy vaginal bleeding. US shows uterus with no placental tissue or thrombi. If
operation required to control bleeding, ligation of branch of which artery required?
- Internal iliac

Why it’s right: One of the most common causes of bleeding minutes to days after delivery is retained placental tissue,
which increases the risk for infection. This patient likely has retained placental tissue not seen on ultrasound and/or
postpartum endometritis, especially in the setting of a low-grade fever. If bleeding cannot be controlled (and this also
applies to ways to control bleeding minutes after delivery), then the blood supply to the uterus must be cut off, which
comes from the uterine artery, which comes from… the internal iliac artery.

Another anatomy question they like to ask regarding surgery and the female reproductive system is, which vessels and
ligaments are cut and can cause injury to the ureters? In a hysterectomy, severance of the cardinal ligament (which runs
with the uterine vessels) can cause ureter injury (because of its proximity). In an oophorectomy, severance of the
suspensory ligament of the ovaries (which runs with the ovarian vessels) can cause ureter injury.
13 ©Test Pirates, LLC. All Rights Reserved.
Step 1 Practice Test #17 Explanations


Take home point: The uterine artery is a branch of the internal iliac artery.

35. 35yo woman abnormal Pap smear. Cervical biopsy shows microinvasive cervical carcinoma. Which microscopic
features led to dx?
- Neoplastic cells in sub-basement membrane connective tissue

Why it’s right: This question describes the correct answer as long as you know that carcinoma in situ means the
neoplastic cells on histology have not broken through the basement membrane. Once the cells are through the
basement membrane, it is considered invasive carcinoma. I can guarantee that “microinvasive carcinoma” is nowhere is
your basic study materials. As such, this question is about being able to apply basic pathology knowledge to a situation
described. Neoplastic cells that are microinvasive have just broken through the basement membrane and are sub-
basement membrane, or right under the basement membrane.

Take home point: Carcinoma in situ is contained by the basement membrane, microinvasive carcinoma has just broken
through the basement membrane and is just below it, and invasive carcinoma is fully through the basement membrane.

36. 17yo girl 1-day shortness of breath, weakness and muscle tenderness. Did triathlon previous day. BMI 19.
Temperature 38 C (100.4 F), respirations 20, bp 150/90. Physical bilateral crackles lower lobes, muscle tenderness.
Creatinine 4. Urinalysis 3+ protein, 4+ hemoglobin. Patient's condition is from increased release of which substance?
- Myoglobin

Why it’s right: The diagnosis is rhabdomyolysis with release of myoglobin and the subsequent toxicity and damage of
myoglobin to the kidneys. This patient is in acute kidney failure evidenced by elevated creatinine and protein and
hemoglobin in the urine. The hemoglobin is actually the myoglobin because myoglobin is detected as hemoglobin. The
patient’s muscle tenderness is due to muscle breakdown from prolonged use – since she just completed a triathlon.
Patients with prolonged seizure activity or even struggling against restraints placed for safety as an inpatient can lead to
rhabdomyolysis. Finally, the lower lobe crackles are from dependent pulmonary edema due to protein leakage in the
urine – loss of colloid pressure in the vessels leads to fluid extravasation into the tissues.

Take home point: Rhabdomyolysis is due to muscle breakdown and release of myoglobin. It can cause kidney injury and
failure, which results in an elevated hemoglobin value on urine analysis.

37. 16yo boy with no signs of puberty. Sex development Tanner stage 2. Physical exam: circumcised penis, soft small tests
5 mL, prostate firm, nontender, no discharge or lesions. Testosterone low. Which hormone is cause of decreased serum
testosterone and lack of pituitary hormone stimulus?
- Luteinizing hormone

Why it’s right: This patient has delayed onset puberty. Most of the changes in puberty are the result of increased
testosterone. Testosterone increases penis and testicular size, libido, body habitus (height and muscle), and deepens the
voice. Luteinizing hormone (LH) is released by the anterior pituitary and stimulates the synthesis of testosterone by
Leydig cells. In the setting of low testosterone and high LH, testicular insufficiency is likely the cause of this patient’s
delay in puberty.

14 ©Test Pirates, LLC. All Rights Reserved.


Step 1 Practice Test #17 Explanations

Take home point: Low testosterone levels result in delayed puberty and can occur in testicular insufficiency with high
levels of luteinizing hormone.

38. 28yo woman wants to lose weight. She binges on high-carbohydrate foods 2 to 3 times a week, forcing herself to
vomit after. BMI 23. Which physical finding is likely?
- Parotid gland enlargement

Why it’s right: This patient suffers from bulimia nervosa. Excess vomiting has resulted in parotid gland enlargement and
inflammation from overuse. These patients can also have tooth enamel loss from stomach acid in the mouth, imprints of
teeth on the backs of knuckles (Russell’s sign) from fingers down the throat used to induce vomiting, and electrolyte
disturbances – hypokalemic hypochloremic metabolic acidosis, specifically. These patients are ultimately distinguished
from anorexic patients based on a normal BMI (anorexia is characterized by a low BMI or body weight <85% of ideal
body weight). These patients are also typically fully aware that their behavior is abnormal (more so than anorexic
patients). Treatment is with SSRIs.

Take home point: Bulimia nervosa can cause parotid gland enlargement and inflammation, tooth enamel loss, and a
hypokalemic hypochloremic metabolic acidosis.

39. 15yo girl emigrated from India and with several lesions on neck for 2 weeks. Physical exam shows hypopigmented,
hypoesthetic area on left side of forehead and 4-cm lesions on neck. Biopsy shows acid-fast bacilli. Best explanation why
the organism results in dermal rather than visceral infections?
- Temperature sensitivity

Why it’s right: The causative organism is mycobacterium leprae (leprosy), a bug that likes cool temperatures and infects
the skin and superficial nerves because of its penchant for the cold. It causes sensation loss in a glove and stocking
pattern on hands and feet, respectively. It can present diffusely over the skin (lepromatous form) or limited to a few
hypoestehtic skin plaques (tuberculoid form). It is treated with dapsone, rifampin, and clofazimine for 2-5 years for the
lepromatous form, and with dapsone and rifampin for 12 months for the tuberculoid form. A favorite question is
presenting signs of hemolysis and anemia, and the question asks which drug was used to treat leprosy? Answer is
dapsone which caused a hemolytic anemia in a patient with G6PD deficiency.

Take home point: Mycobacterium leprae causes leprosy and infects skin and superficial nerves because it likes cool
temperatures.

40. 59yo man has total thyroidectomy for 4-cm follicular carcinoma of thyroid. Twelve hours after procedure, has
paresthesias of hands and feet. Vitals stable, carpal spasm on inflammation of bp cuff. Lab findings?
- Decreased parathyroid hormone, decreased calcium

Why it’s right: The parathyroid glands are very close to the thyroid gland, and as such, can be injured in a total
thyroidectomy. This patient has signs of low calcium, specifically, Trousseau sign of latent tetany, which is flexion of the
wrist and MCP joints and extension of the DIP and PIP joints during compression of the brachial artery (compressed by
the bp cuff). Another sign of hypocalcemia is Chvostek sign, which is contraction of the lips and/or nose when the facial
nerve is tapped.

15 ©Test Pirates, LLC. All Rights Reserved.


Step 1 Practice Test #17 Explanations

Take home point: Spasm of muscles in the wrist and fingers during compression of the brachial artery is secondary to
hypocalcemia = Trousseau sign of latent tetany.

41. 41yo man with asthma and allergy to grass pollen wheezes and difficulty breathing 10 min after mowing lawn. Drug
for immediate relief of acute symptoms?
- Albuterol

Why it’s right: Commonly known as a rescue inhaler, albuterol is a beta2-agonist used in an acute exacerbation of
asthma. It results in smooth muscle relaxation. Long-acting beta2-agonists used for prophylaxis include salmeterol and
formoterol. Corticosteroids are generally added in chronic asthma and include beclomethasone and fluticasone.

Take home point: Albuterol is a beta2-agonist used to treat acute asthma and asthma attacks.

42. 14yo boy daily headaches for 2 months. Headaches are bilateral aching in temples. "Has not been himself" for
months. Confused, forgetting names, dates, places, clumsy, frequent falls. School performance declined. Physical exam:
broad-based ataxic gait. Slow to answer questions. Chronic abuse of which substance?
- Inhaled glue

Why it’s right: The toxic chemical here is toluene, which is in glue, nail polish remover, and paint thinner among other
things. It can cause dizziness, nystagmus, incoordination, slurred speech, unsteady gait (broad-based gait due to
cerebellar dysfunction), depressed reflexes, tremor, muscle weakness, euphoria, diplopia, and coma. Chronic use leads
to multi-organ damage including brain atrophy evidenced by forgetfulness, dementia, and irreversible encephalopathy.
It can also cause heart damage in the form of myocarditis and congestive heart failure as well as kidney damage and a
renal tubular acidosis.

Take home point: Inhalation and abuse of glue, paint thinner, and nail polish remover (all contain toluene) can cause
permanent neurological, cardiac and kidney damage. Neurological damage includes dementia and cerebellar damage
evidenced by changes in balance and gait.

43. 40yo African American woman 2-week hx fever, malaise, dyspnea. Temperature 36.7 C (98 F), respirations 20.
Physical exam: erythema nodosum, parotid enlargement, hepatosplenomegaly. Calcium 16. CT chest bilateral hilar
adenopathy. Increased in which in serum?
- 1,25-Dihydroxycholecalciferol

Why it’s right: The diagnosis is sarcoidosis based on findings of erythema nodosum, hepatosplenomegaly, bilateral hilar
adenopathy, and hypercalcemia. The pathophysiology of elevated calcium in sarcoidosis starts with the epithelioid
macrophages, part of the granulomas of sarcoid. The macrophages have 1-alpha-hydroxyase, an enzyme found in the
kidneys that converts 25-vitamin D (D3) to the active form 1,25-dihydroxylcholecalciferol. Active vitamin D increases the
intestinal absorption of calcium and phosphorous, pathologically leading to hypercalcemia in sarcoid.

Take home point: Sarcoidosis results in hypercalcemia due to elevated levels of 1,25-dihydroxylcholecalciferol mediated
by 1-alpha-hydroxylase within epithelioid macrophages.

16 ©Test Pirates, LLC. All Rights Reserved.


Step 1 Practice Test #17 Explanations

44. 56yo woman with restrictive cardiomyopathy, proteinuria, renal failure. 35-year history of rheumatoid arthritis. Renal
biopsy shows glomerular deposition of eosinophilic hyaline material. Congo red statin: birefringent pattern under
polarized light. Structure of material?
- beta-pleated sheet structure

Why it’s right: This patient has AA amyloidosis in the setting of rheumatoid arthritis (RA). Amyloid represents folded
protein that is prone to buildup in the body. Many proteins can form amyloid as long as they adopt the folded
conformation known as the beta-pleated sheet. In the case of RA (and other autoimmune conditions like IBD and
ankylosing spondylosis) acute phase reactant proteins buildup and can cause amyloidosis. Renal failure due to
amyloidosis in RA is rare, but can occur in the setting of long-standing RA from amyloid buildup in the glomeruli – the
eosinophilic hyaline material seen on histology is amyloid, and staining demonstrates a birefringent pattern
characteristic of amyloid under polarized light. In this patient, amyloid has also built up in the heart and led to a
restrictive cardiomyopathy.

Take home point: Secondary AA amyloidosis can result in renal failure in the setting of long-standing RA. Amyloid forms
characteristic beta-pleated sheets.

45. 63yo man 3-month hx difficulty sleeping. Sleeps better upright. HR 90, bp 110/60. Physical exam: increased jugular
venous pressure, mild ankle edema. Cause of edema?
- Increased capillary hydrostatic pressure

Why it’s right: This patient has congestive heart failure evidenced by difficulty sleeping on his back (due to pulmonary
edema), increased JVD, and ankle edema. His resting heart rate is high because the heart is trying to continue pumping,
but doesn’t have much force – blood pressure is borderline low 110/60 for a such a high heart rate. The result is buildup
of blood on the venous side because of pump failure, and this buildup of fluid results in elevated hydrostatic pressure
within the blood vessels and leakage of fluid à pitting edema in the extremities and pulmonary edema in the lungs.
Different permutations of osmotic and hydrostatic pressure are favorite questions of the USMLE.

Take home point: In congestive heart failure, edema of extremities and lungs results from elevated hydrostatic pressure
in blood vessels, causing extravasation of fluid.

46. 14yo boy come to ER 1 hour after colliding with teammate playing soccer. Physical exam: edematous tissues of left
eye, mild depression of left zygomatic bone. Skin between eye and upper lip numb. Double vision look upward. Nerve
damaged causing sensory loss?
- Maxillary division of trigeminal nerve

Why it’s right: Orbital floor fractures (fracture of the maxillary bone) are the most common fractures in orbital trauma
because of the thinness of the orbital floor and susceptibility to blunt trauma. Within the orbital floor travels the inferior
orbital nerve, a branch of the maxillary division of CN V (the trigeminal nerve), exiting via the inferior orbital foramen.
This nerve is responsible for skin sensation of the cheek and of the upper molars (patients often complain of upper teeth
numbness). Double vision is due to swelling of the orbit. More likely in this case however, the double vision is due to
inferior rectus entrapment, preventing the affected eye from looking up because it is tethering the eyeball down by the
inferior rectus (even though the inferior rectus is responsible for downgaze). Treatment in the case of muscle
entrapment requires surgery with muscle release and repair of the orbital floor bones, potentially with a porous implant.

17 ©Test Pirates, LLC. All Rights Reserved.


Step 1 Practice Test #17 Explanations

Take home point: The inferior orbital nerve (a branch of V2) is commonly injured in orbital injuries and fractures
because the maxillary bone that makes up the orbital floor is most commonly fractured in blunt orbit trauma. This
results in decreased sensation within the V2 distribution of the face.

47. 35yo man increasingly depressed, impulsive and difficult over past year. Grimaces intermittently with rapid, jerking,
purposeless movements of fingers. Historical factor relevant in dx?
- Family history of a similar illness

Why it’s right: This is Huntington’s disease, an autosomal dominant disease that results from degeneration of the
caudate nucleus. It can first present as a mood disorder with symptoms of depression, aggression, and dementia. It can
even be first mistaken for substance-abuse disorder. Chorea (dance-like) and uncontrolled movements that are fast,
jerky and purposeless are characteristic of Huntington’s. Family history is key because this is autosomal dominant and
presents earlier in life with each subsequent degeneration (known as “anticipation”).

Take home point: Huntington’s disease is autosomal dominant and may first present with depression and aggressive
behavior. A positive family history is necessary for the diagnosis because of the mode of inheritance.

48. 18yo man Crohn disease 1-day hx severe abdominal pain and intermittent bloody diarrhea. Temperature 38 C (100.4
F), pulse 98, respirations 18. Physical exam: draining anal fistula. Treatment with antibiotics and prednisone over next 3
weeks recovers. Mechanism of pharmacotherapy is suppression of which?
- T-lymphocyte function

Why it’s right: Crohn’s is an inflammatory bowel disease that can affect the GI tract from the mouth to the anus (as
opposed to ulcerative colitis, which just affects the large intestine). On histology, it results in noncaseating granulomas
and lymphoid aggregates mediated by helper Th1 cells – versus colitis, which is mediated by helper Th2 cells. The key
immune player in Crohn’s is the T cell, so therapy targets immune suppression and suppression of T cell function.

Take home point: Both inflammatory bowel diseases, Crohn’s and ulcerative colitis, are mediated by T lymphocytes –
Th1 and Th2, respectively.

49. 34yo woman with pyelonephritis treated with bactericidal antibiotic 4 days no improvement. Antibiotic added that
inhibits binding to 30S ribosome, blocking protein synthesis intracellularly. Antibiotic?
- Gentamicin

Why it’s right: No improvement after 4 days of antibiotic therapy means that the antibiotic needs to be changed
because it is not working – either due to resistance or ineffectiveness. Second-line treatment is a stronger antibiotic, like
an aminoglycoside, which is good a second-line drug in pyelonephritis, a life-threatening complication of UTIs. The
mechanism of action of aminoglycosides is to bind to the 30s subunit of the ribosome and inhibit formation of the
initiation complex that begins reading mRNA. Without reading mRNA, protein synthesis cannot begin and the bacteria
cannot reproduce. For this reason, aminoglycosides are bacteriocidal.

Take home point: The mechanism of aminoglycosides is inhibiting the formation of the initiation complex by binding to
the 30s ribosomal subunit to prevent mRNA translation.

18 ©Test Pirates, LLC. All Rights Reserved.


Step 1 Practice Test #17 Explanations

50. 3yo boy and his 5yo brother with recurrent hemarthroses. Both parents healthy, but mother with two younger
brothers with same sx and maternal uncle who died at 8 of mild head trauma. Partial thromboplastin time is prolonged.
Defect?
- Factor VIII (antihemophilic factor)

Why it’s right: Hemophilia A and B are both inherited in an X-linked recessive fashion. The inheritance pattern described
in the question stem is X-linked recessive because it affects males on the mother’s side of the family, therefore the
mother of these two brothers is a carrier. One of the characteristic (and high-yield!) symptoms of hemophilia A and B is
hemarthrosis or bleeding into a joint after minimal joint trauma. Oftentimes, the hemarthroses will just be described as
“painful joints” and with this family history in the setting of easy bruising, hemophilia can be diagnosed. Hemophilia A is
due to a deficiency in factor 8 (increases PTT) while B is due to a deficiency in factor 9 (also increases PTT). Hemophilia A
is more common than B.

Take home point: Hemophilia A and B are X-linked recessive bleeding disorders that result in recurrent bruising and
hemarthroses, or bleeding into joints after minimal joint trauma.

19 ©Test Pirates, LLC. All Rights Reserved.
















Practice Test Explanations
Step 1 Practice Test # 17 – Block 3
Question Total: 50

Step 1 Practice Test #17 Explanations


Block 3

1. 6yo boy from Russia with unstable gait and incoordination for 2 weeks. Pale, bulky stools for 4 years and two episodes
of bacterial pneumonia and chronic cough since age 1 year. 3%ile for height/weight. Neuro exam shows ataxia, no DTRs,
loss of proprioception. Stool shows increased fat concentration. Vitamin deficient?
- Vitamin E

Why it’s right: This patient has cystic fibrosis with pancreatic insufficiency resulting in an inability to absorb fat-soluble
vitamins – A, K, D, and E. Vitamin E deficiency causes similar neurological changes to that of B12 deficiency – subacute
combined degeneration of the spinal cord with dorsal column and spinocerebellar tract demyelination. The result is
ataxia, loss of proprioception, etc. Vitamin E deficiency also results in a hemolytic anemia because vitamin E protects
erythrocytes from free radical damage.

Take home point: Cystic fibrosis results in fat malabsorption and malabsorption of fat-soluble vitamins, A, K, D, and E,
due to pancreatic insufficiency.

2. 54yo F 1 week after sudden loss of vision in left eye, returned within 1 day. 3-month hx of progressive SOB with
exertion. Echocardiography shows mass in the left atrium of the heart. Lesion is resected, photomicrograph of it is shown.
Which describes the lesion?
- Myxoma

Why it’s right: There are several causes of thromboembolic phenomena – which occurred in this patient in the form of
amaurosis fugax, a risk factor for both central retinal artery occlusion (which would result in permanent loss of vision)
and stroke. These causes include atrial fibrillation, valvular disease, endocarditis, and atrial myxoma. Females are at
increased risk for atrial myxoma, and this patient is fittingly female. Atrial myxoma can be asymptomatic, but in this
patient it was symptomatic and the cause of her SOB. Pieces of the tumor break off and result in stroke.

Take home point: Atrial myxoma is a cause of thromboembolism including amaurosis fugax, central retinal artery
occlusion, and stroke.

3. 27yo primigravid woman at 34 weeks' with nausea and vomiting, and abdominal pain for 12 hours. Everything's been
normal. BP is now 164/102, and right upper quadrant tenderness. Labs show Hb 7.4, HCT 24%, Platelets 72k, Cr 1.2, total
bili 2.3, AST 112, ALT 126. Peripheral blood smear will show?
- Schistocytes

Why it’s right: The diagnosis here is two-fold: preeclampsia (pregnancy-induce hypertension) and HELLP syndrome
(hemolysis, elevated liver enzymes, low platelets). The most common time preeclampsia presents is at 34 weeks. Blood
pressure is >140/90 with proteinuria or evidence of kidney damage. The cause is placental ischemia resulting in
increased vascular tone – a snowball effect. Hemolysis of erythrocytes results from platelet aggregation (also resulting in
2 ©Test Pirates, LLC. All Rights Reserved.
Step 1 Practice Test #17 Explanations

thrombocytopenia) and creates schistocytes seen on blood smear. Mortality in these patients is due to cerebral
hemorrhage and acute respiratory distress syndrome.

Take home point: HELLP is associated with preeclampsia and presents with hypertension and proteinuria or kidney
damage (preeclampsia), abdominal pain, hemolysis (schistocytes on blood smear), elevated liver enzymes, and low
platelets.

4. 60yo F 3 hours after sudden onset ankle pain. 4-year Hx of increasing serum creatinine concentrations. Began
furosemide 1 month ago, also takes glipizide. P 120/min, resp 25/min, BP 150/100. Joint fluid shows negatively
birefringent crystals. Increased risk of which complications of underlying process causing joint findings?
- Nephrolithiasis

Why it’s right: Sudden onset joint pain with fluid tap demontrated negative birefringent crystals are symptoms and
findings of an acute gouty arthritis attack. Patients who experience gout attacks have higher levels of uric acid and are
also at increased risk for uric acid kidney stones – nephrolithiasis. The attack was likely also precipitated by the addition
of furosemide, a diuretic that decreases uric acid excretion. Diuretics in general can cause dehydration, which is another
cause of kidney stones. Of note, the pulse is elevated because the patient is in pain. Side note: the most common cause
of urinary retention after surgery is pain.

Take home point: Elevated serum uric acid levels increases the risk for gout attacks and uric acid kidney stones.
Furosemide increases serum uric acid levels and can precipitate an attack.

5. 56yo F follow-up 8 weeks after recovering from pneumococcal pneumonia. Chest X-rays normal. Which allowed this
resolution to occur?
- Maintenance of basement membrane integrity

Why it’s right: For the chest X-ray to be clear, the basement membrane had to be intact during the pneumonia and
during the repair process once the pneumonia resolved. The basement membrane the question stem is describing is
that between the lung parenchyma and blood vessels that allows gas exchange to occur. In pneumonia, it is a filling
process of the alveoli that occurs with infection and inflammation. When there is no necrosis or permanent destruction
of respiratory air cells and blood vessels, the infection is cleared and lung parenchyma assumes its physiological
structure and function. If there was destruction of tissue without complete resolution (i.e. if granulation tissue formed, if
fibrosis occurs or new blood vessels were produced) it would be seen on X-ray. Also, if type I pneumocytes are lost or
damaged during the infectious process, type II pneumocytes (part of the parenchyma) can differentiate to reproduce
type I pneumocytes.

Take home point: Maintenance of the basement membrane during a pneumonia allows complete resolution of infection
and inflammation because if it is not destroyed and remains intact, the chest X-ray will appear clear.

6. 29yo F with 5-week hx of fatigue and 4-day hx of heart palpitations and anxiety. Has primary hypothyroidism Rx with
triiodothyronine, but she has doubled the dose because of fatigue. TFT will show which?
- TSH DECREASED, Free thyroxine DECREASED, Free triiodothyronine INCREASED

Why it’s right: There exist two treatments for primary hypothyroidism – levothyroxine, which is T4, and
triiodothyronine, which is T3. If this patient is on triiodothyronine, then her levels of T3 will be increased because she

3 ©Test Pirates, LLC. All Rights Reserved.


Step 1 Practice Test #17 Explanations

started taking more of it. Native T3 will decrease via negative feedback. More specifically, T3 produces negative
feedback at the level of the anterior pituitary by decreasing sensitivity to TRH, lowering TSH. With lower levels of TSH,
natural T4 and T3 levels decrease.

Take home point: Supplementation with triiodothyronine increases T3 levels, and decreases TSH and T4 through
negative feedback at the level of the anterior pituitary.

7. 20yo F has multiple neurofibromas. Mom, uncle, and brothers with similar lesions. Mode of inheritance?
- Autosomal dominant

Why it’s right: This is a straightforward, one-step question regarding inheritance mode. Even if you didn’t know that
neurofibromatosis is an autosomal dominant disorder, the question stems gives a big hint – it’s very prevalent in the
family, as mom, uncle and brothers have similar phenotypic findings. Both neurofibromatosis type 1 and 2 (NF-1 and 2)
are autosomal dominant diseases.

Take home point: NF-1 and NF-2 are both autosomal dominant disease. Know the major autosomal dominant diseases
as these questions are high-yield questions.

8. 45yo M with yellow skin. Drinks eight to ten 12-ounce cans of beers daily for 10 days. Liver is tender. Serum: total bili
5.9, Alk Phos 210, AST 110, ALT 69, gamma-glutamyltransferase 25 (n = 0-30). Liver biopsy will show?
- Mallory hyaline

Why it’s right: The diagnosis is alcoholic hepatitis secondary to long-standing alcohol consumption. Jaundice is described
as yellow skin and due to bilirubin buildup. His liver enzymes are elevated and fit the 2:1 ratio of AST:ALT in alcoholic
liver disease. Viral hepatitis, in contrast, causes a ratio reversal, or ALT:AST of >2. GTT is also usually elevated in alcoholic
liver disease, but does not need to be to make the diagnosis. Liver biopsy classically will demonstrate swollen and
necrotic hepatocytes with intracytoplasmic eosinophilic (red) inclusions called Mallory bodies, which are hyaline
congregates. Neutrophils will also be present. With continued alcohol consumption, cirrhosis results and liver enzymes
will actually decrease. The characteristic finding on pathology is sclerosis around the central vein.

Take home point: Biopsy of alcoholic hepatitis will demonstrate Mallory bodies, or hyaline, which are eosinophilic,
intracytoplasmic inclusions in swollen and necrotic hepatocytes due to liver damage.

9. 50yo M smoked 2 packs per day for 34 years, with SOB on exertion, chronic cough, and wheezing. Increased AP
diameter, diminished breath sounds, scattered rhonchi. Which Lab abnormalities is expected?
- Increased blood HCO3

Why it’s right: The diagnosis is COPD and this patient is a chronic blue bloater based on his history and findings of
increased AP (anterior-posterior) diameter. COPD leads to loss of elastic fibers and decreased recoil ability of alveoli and
respiratory passageways. CO2 is retained instead of exhaled, and a respiratory acidosis results. The chronicity of the
acidosis causes a metabolic alkalosis that attempts to compensate for the acidosis. The kidneys compensate and
generate a metabolic alkalosis by retaining HCO3, increasing HCO3 blood content.

Take home point: COPD results in a compensated respiratory acidosis due to a concurrent metabolic alkalosis that
increases HCO3 blood content.

4 ©Test Pirates, LLC. All Rights Reserved.


Step 1 Practice Test #17 Explanations

10. 35yo M uses crack cocaine daily, with 2-hour Hx of substernal chest pain. T 37C, P 110/min, BP 160/100. Most
appropriate next step?
- Admit the patient to the hospital for possible myocardial ischemia

Why it’s right: Cocaine use is a risk factor for heart problems, especially myocardial infarction due to vasoconstriction of
the coronary arteries secondary to effects of cocaine. Chronic cocaine use also leads to cardiomyopathy, which is a
separate risk factor for MI. If this patient uses cocaine daily and has chest pain, a myocardial infarction is the diagnosis
until proven otherwise. He also has tachycardia and elevated blood pressure. Tachycardia is one of the first signs of an
MI on ECG, and in someone with risk factors, an MI must be ruled out. The next most important step is emergency
treatment, MI workup, and cardio consult!

Take home point: Cocaine users are at a high risk for myocardial infarction due to vasoconstriction caused by the effects
of cocaine.

11. 54yo F with hypertension and bilateral renal artery stenosis starts taking NSAIDS for back pain. Her Cr concentration
increases from 1.0 to 5.0. Cause is due to inhibiting which?
- Vasodilating prostaglandins at the afferent arteriole

Why it’s right: Renal stenosis occurs at the level of the afferent arteriole, or arteriole leading into the kidney (versus
away from the kidney – the efferent arteriole). Stenosis decreases blood flow to the kidney, starving the kidney of
oxygen, and leading to afferent arteriole dilation, a response by the kidney via prostaglandins to increase blood flow
(and the glomerular filtration rate [GFR]) for oxygen delivery. In bilateral stenosis, both afferent arterioles are dilated to
maintain GFR). NSAIDs inhibit renal production of prostaglandins, and can lead to acute kidney failure due to the loss of
afferent arteriole dilation. For the reason, the creatinine jumped from 1.0 to 5.0. Think of it like the emergency backup
water supply is the only supply left, it is running at max, and then it is suddenly closed when NSAIDs are used.

Take home point: NSAIDs can precipitate acute kidney failure due to inhibition of prostaglandin dilation of the afferent
arteriole.

12. 83yo M brought to ED after being found at home bedridden and confused. No meds. P 100/min, BP 85/50. BP
unchanged after 1L IV saline. Pulmonary artery catheter shows: Cardiac output (CO) high, PCWP low, systemic vascular
resistance low. Cause of hypotension?
- Early septic shock

Why it’s right: Types of shock are high-yield for all step exams (1-3) and the shelf exam for surgery. Septic shock is
always easy to identify because conceptually it leads to a unique set of findings in terms of CO, PCWP, and systemic
vascular resistance. The infection that causes sepsis leads to a toxin release and inflammatory response that results in
loss of intravascular proteins and fluid into the extravascular space and diffuse vasodilation à low systemic vascular
resistance. Administration of fluids does not improve shock because the fluid continues to leak from the intravascular
space (the treatment, therefore, is norepinephrine to raise blood pressure via vasoconstriction). CO is high because the
heart is responding to hypotension and trying to compensate, while PCWP is low because the cause of hypotension is
not cardiogenic (if that were the case, PCWP would be high due to the buildup of blood in a poorly pumping heart).

Take home point: In septic shock, cardiac output is high, PCWP is low, and systemic vascular resistance is low.
Hypotension typically does not respond to fluid administration.

5 ©Test Pirates, LLC. All Rights Reserved.


Step 1 Practice Test #17 Explanations

13. 32yo M with 3-month hx of swelling and breast tenderness. Receiving thyroid hormone and steroid replacements
since removal of pituitary adenoma 2 years ago. Started hCG injections 4 months ago. Most likely binding site of hCG
causing gynecomastia?
- Tissue: Testicle; Effect: estradiol production

Why it’s right: This question can be figured out by simply knowing that estrogen (and its various forms including
estradiol, estrone, and estriol) increases breast size. The physiology, however, is a bit more complicated, but can be
explained starting with beta-hCG. Beta-hCG maintains the corpus luteum during the first trimester of pregnancy by
acting like luteinizing hormone (LH). In men, LH is secreted by the anterior pituitary and stimulates testosterone
synthesis in Leydig cells. Therefore, hCG injections in men act like LH and stimulate testosterone secretion from the
testicle. Testosterone is then converted to estrogen in both adipose tissue and Leydig cells via aromatase.

Take home point: hCG acts like LH and causes estrogen release in women and can cause testosterone and estrogen
release in men via action on the Leydig cells in the testicles.

14. 38yo F with 3-day hx of sore throat. Photo shown of throat. Which nerves is tested by saying "ah," elevating area at
tip of the arrow?
- Vagus

Why it’s right: This is a straight recall question. Cranial nerve X (Vagus nerve) is responsible for palatal elevation. It also
carries fibers for taste in the epiglottic region, swallowing, maintaining the uvula in the midline, coughing,
thoracoabdominal viscera, and the chemo- and baroreceptors of the aortic arch.

Take home point: CNX (Vagus nerve) is responsible for palatal elevation.

15. 8yo boy with disruptive behavior, interrupts, always moving, trouble completing tasks. Drug with which mechanism is
appropriate?
- Increased release of dopamine and norepinephrine

Why it’s right: Attention-deficit hyperactivity disorder (ADHD) is treated with methylphenidate, a CNS stimulant that
works by increasing catecholamines in the synaptic cleft, especially dopamine and norepinephrine. Other CNS stimulants
include dextroamphetamine and methamphetamine. Atomoxetine is also used to treat ADHD and works by inhibiting
the reuptake of norepinephrine.

Take home point: The mechanism of action of medications like methylphenidate used to treat ADHD is increased
release of dopamine and norepinephrine.

16. 62yo M with alcohol-induced liver disease develops ascites. Infection ruled out. Most appropriate diuretic, in addition
to loops, is which?
- Spironolactone

Why it’s right: It is important that information regarding an infection was given, because ascites is alcoholic cirrhosis can
lead to spontaneous bacterial peritonitis and the additional treatment would therefore be antibiotics. Since there is no
infection, the addition of another diuretic, specifically a potassium-sparing one like spironolactone, is appropriate

6 ©Test Pirates, LLC. All Rights Reserved.


Step 1 Practice Test #17 Explanations

therapy. Patients with cirrhosis should also be supplemented with potassium to promote the production of coagulation
factors, which are decreased typically in cirrhosis.

Take home point: Ascites secondary to alcohol-induced cirrhosis should be treated with loop diuretics and
spironolactone, as well as antibiotics if an infection is present.

17. 72yo M with weakness and fatigue. Hemoglobin concentration is 9.2, WBC 5400, platelets 350k. Peripheral blood
smear is shown. Cause?
- Gastrointestinal blood loss

Why it’s right: The most common cause of anemia in an elderly patient is a GI bleed until proven otherwise. If you didn’t
know that, then exam of the blood smear would show microcytosis and hypochromia, or pale RBCs with a central
clearing that is larger than normal RBCs. These findings are characteristic of iron-deficiency anemia secondary to a GI
bleed. Other causes of iron-deficiency are due to lack of iron intake (i.e. malnutrition/absorption disorders) or increased
demand (i.e. pregnancy).

Take home point: Iron deficiency anemia (commonly caused by GI bleeding in elderly) causes microcytosis and
hypochromia of RBCs demonstrated on blood smear.

18. 65yo F with widely metastatic breast cancer unresponsive to chemo. No family. "close friend," at all her visits and
now she is moved to inpatient hospice after she decides she wants no further curative therapy. Hospital rules state
visitors cannot stay after visiting hours. Says, "We can't bear to be apart. It would be cruel to separate us now.". Which
response is appropriate?
- "The two of you seem to have a very important relationship. Of course you may stay together."

Why it’s right: This is definitely a difficult ethics question. Are rules allowed to be broken for terminally ill patients? In
this case, the question stem gives a lot of background regarding her history without any family and with a terminal
disease, encouraging the test-taker to choose a more sympathetic route and allow the patient and her friend to stay -
especially when the patient says, “it would be cruel to separate us now.” The answer doesn’t imply that the friend is
moving into the room with her; it only means that the rules are allowed to be broken when circumstances reflect a
situation similar to what this patient’s, and the patient’s friend should have the option of staying.

Take home point: Ethical questions that are not straightforward have clues in the stem to support one answer over the
others. Read carefully!

19. 46yo M treated with oral cyclosporine after cardiac transplant. Cyclosporine decreases likelihood of rejected by which
actions?
- Suppressing the early response of T lymphocytes to activation

Why it’s right: Cyclosporine is a common medication utilized in transplant rejection prevention. It works by binding to
cyclophilins, blocking the differentiation and activation of T cells by inhibiting calcineurin. IL-2 production via T cells is
then blocked. Tacrolimus is also a calcineurin inhibitor but binds to FK-binding protein.

Take home point: Cyclosporine binds to cyclophilins, blocking the differentiation and activation of T cells by inhibiting
calcineurin.

7 ©Test Pirates, LLC. All Rights Reserved.


Step 1 Practice Test #17 Explanations

20. 30yo M in ED 15 minutes after found unconscious. Comatose, pupils 4 mm in diameter, not reactive to light. CT head
shown. Cause of coma is bleeding from which structures?
- Middle meningeal artery

Why it’s right: This patient is obtunded due to an epidural hematoma, which on imaging, shows a biconvex, hyperdense
opacity, representing blood. The bleeding classically does not cross sutures lines, meaning the blood stays in its own
area, but it can cross the falx cerebri (so cross from one hemisphere to the other if the bleeding is located superiorly).
Conversely, a subdural hematoma can cross suture lines (so the bleeding is over a larger part of the hemisphere of the
brain), but not the falx. An epidural hematoma comes from the middle meningeal artery, which is a branch of the
maxillary artery, and it travels through the foramen spinosum.

Take home point: Epidural hematoma is due to bleeding from the middle meningeal artery, a branch of the maxillary
artery.

21. 68yo M with 6-month hx of erectile dysfunction. PE and labs normal. If pharmacotherapy is indicated, drug with
which MOA?
- Inhibition of phosphodiesterase

Why it’s right: The medications used to treat erectile dysfunction (ED) include sildenafil and vardenafil, which are
phosphodiesterase 5-inhibitors. Phosphodiesterase 5 breaks down cGMP, and without the breakdown of cGMP and
increased levels of it, smooth muscle relaxes and vasodilation results (via the release of nitric oxide). When vasodilation
occurs in the corpus cavernosum of the penis, blood flow increases, leading to an erection. It is key to note that
parasympathetic innervation must be intact for these medications to work because innervation initiates the release of
cGMP and the medication keeps the cGMP level high.

Take home point: Sildenafil and vardenafil are phosphodiesterase 5-inhibitors, leading to increased levels of cGMP,
increased blood flow in the penis, and an erection. Hooray.

22. 35yo M with recurrent sinusitis and bronchitis. Cardiac examination shows PMI at fourth intercostal space within the
midclavicular line on the right. Hepatic margin is palpable on the left. Endoscopy shows nasal polyps. Biopsy shows
thickened, ciliated, pseudostratified epithelium with small patches of squamous metaplasia and mild lymphoid
infiltration. Which structure most likely to be absent on electron microscopic exam of epithelium?
- Dynein arms

Why it’s right: The diagnosis is Kartagener’s syndrome (or primary ciliary dyskinesia). This disease is due to a defect in
dynein arms of cilia, resulting in a ciliopathy. Cilia are made of microtubules in a 9+2 arrangement, and dynein arms are
the links between the pairs that allow the cilia to move as a single unit. The array of problems that results from dynein
defects all require the proper functioning of cilia to work. These patients develop bronchiectasis and recurrent sinusitis
evidenced by nasal polyps as a result of cilia in the respiratory bronchioles inability to sweep away mucus and particles.
Males with Kartagener’s also suffer from sterility due to sperm immobility, while females have decreased fertility since
the fallopian tubes use ciliary processes to sweep the oocyte towards the uterus. Patients also have another
characteristic finding related to embryogenesis – sinus inversus, when organs are on the opposite sides of their usual
locations – i.e. the liver on the left and heart on the right (also known as dextrocardia).

8 ©Test Pirates, LLC. All Rights Reserved.


Step 1 Practice Test #17 Explanations

Take home point: Kartagener’s syndrome is a ciliopathy that results from defective dynein arms, or links between the
microtubules that make up cilia.

23. 30yo primi at 22 weeks' gestation with 1-day hx of fever, chills, and muscle aches. T 39.4, P 114/min, resp 15/min, BP
104/72. PE shows uterus consistent with 22-week gestation. Fetal heart sounds are heard. WBC 12K, Blood cultures grow
gram-positive rods. Causal organism?
- Listeria monocytogenes

Why it’s right: This question demonstrates why pregnant women should avoid ingestion of unpasteurized milk/cheese
and deli meats during pregnancy – due to an increased risk to the mother and fetus for Listeria infection. Listeria also
causes severe infections in elderly and immunocompromised patients. This patient has sepsis (elevated heart rate,
decreased blood pressure, elevated WBC, and positive blood cultures). The organism causes fetal infection by crossing
the placenta, and eventually causing intrauterine demise or newborn meningitis/sepsis. A true giveaway for this
question would be if they described this gram-positive rod as exhibiting “tumbling motility,” a unique characteristic of
Listeria.

Take home point: Listeria is a gram-positive rod that can cause septicemia in pregnant women, neonates,
immunocompromised patients, and the elderly.

24. 42yo M with multiple lesions over his body. PE shows flaccid bullous erosions involving upper and lower extremities
and torso. Biopsy shows extensive epidermal acantholysis resulting in the formation of intraepidermal blister. Intact basal
layer of keratinocytes adherent to basement membrane is identified. Which is related to pathogenesis?
- Development of autoantibodies against desmosomal proteins

Why it’s right: The diagnosis is pemphigus vulgaris based on the presence of flaccid bullous erosions (in bullous
pemphigoid (BP) they are tense bullous erosions) that on histology demonstrate acantholysis (or breakage of cellular
adhesions) in an intraepidermal pattern (versus BP, which is along the basement membrane). The disease is due to
autoantibodies against desmosomes that link keratinocytes together in the epidermis (versus hemidesmosomes that link
the epidermis to the basement membrane). Another key differentiator between pemphigus and bullous pemphigoid is
that pemphigus can involve the skin and oral mucosa, whereas BP only involves the skin and spares the oral mucosa.

Take home point: Pemphigus vulgaris results from autoantibodies against desmosomes that link keratinocytes together
in the epidermis.

25. 50yo M 3 days after his first generalized tonic-clonic seizure. 1-month hx of frequent episodes of pins-and-needles
sensation around the mouth, hands, and feet, involuntary contraction of muscles. Neuro exam shows mild, diffuse
hyperreflexia. Which serum electrolyte is abnormal?
- Calcium

Why it’s right: Hypocalcemia results in severe systemic findings including tetany (intermittent, involuntary muscle
spasms), seizures (in severe cases or long-standing hypocalcemia), hyperreflexia, and tingling. Other more specific signs
include Chvostek’s and Trousseau’s signs, for which the USMLE will often simply describe because they do not use
eponyms. Hypercalcemia results in stones, bones, groans and psychiatric moans. Stones = kidney stones; bones = bone
pain; groans = stomach pain; and moans = anxiety or altered mental status. Importantly, with symptoms of
hypercalcemia, patients do not always have calcium in their urine.
9 ©Test Pirates, LLC. All Rights Reserved.
Step 1 Practice Test #17 Explanations


Take home point: Hypocalcemia can result in tetany, or intermittent, involuntary muscle spasms, a tingling sensation,
and seizures.

26. 52yo F with hot flashes. Menses have been irregular for the past 6 months. Physiologic cause?
- Failure of the ovaries to secrete 17beta-estradiol

Why it’s right: This patient is beginning menopause with decreased levels of estrogen (the cause of hot flashes and
irregular menses) as a result of ovarian failure. Estrogen levels increase as a result of 17beta-estradiol release from the
ovaries, estriol release from the placenta (when there is one present), and estrone via aromatization from adipose
tissues. FSH and LH interact to stimulate the ovary to increase estrogen levels, and for this reason, during menopause,
FSH and LH levels are high due to the lack of negative feedback from the non-functioning ovaries.

Take home point: The ovaries release 17beta-estradiol, which increases estrogen. In menopause, the ovaries begin to
burnout, resulting in decreased levels of estrogen, hot flashes, and irregular menses.

27. 32yo F G2P1 at 7 weeks' gestation with vaginal bleeding for 3 days and increasingly severe left abdominal pain for 18
hours. Direct and rebound tenderness with guarding in left lower quadrant. Cervical os is closed. serum beta-hCG is 6000.
U/S shows empty uterus. Dx?
- Ectopic pregnancy

Why it’s right: Pain in the early stages of pregnancy may indicate either ectopic pregnancy or spontaneous abortion.
This patient has signs of an acute abdomen (direct and rebound tenderness with guarding), meaning there is likely
already rupture of the fallopian tube where the growing embryo was located. The level of beta-hCG is typically low, and
ultrasound showing an empty uterus confirms the diagnosis. This question could have been made more difficult by
asking what would biopsy of the endometrium show: a decidualized endometrium without chorionic villi (because they
only develop in an intrauterine pregnancy). That question would take this from a one-step question (what’s the
diagnosis?) to a two-step question (what’s the diagnosis? Now, what’s the histopathology?)

Take home point: Suspect ectopic pregnancy during early stages of pregnancy that present with pain and findings of an
acute abdomen on exam.

28. 26yo F 5 weeks after birth of first child. Worries constantly that the infant is ill and wakes up to make sure he is well.
Washes her hands 30 times per day. Worried about people braking into her house, checks lock 3-4 times a night. Not
breast feeding. Rx?
- Sertraline

Why it’s right: The diagnosis is obsessive-compulsive disorder based on this patient’s constant anxiety, repeated
compulsions to alleviate her anxiety, and the problems these actions are causing that limit her daily functioning. Her
constant worry represents intrusive thoughts and obsessions, while the washing of her hands and re-checking the locks
are compulsions. The treatment is either an SSRI or clomipramine. Almost all medications end up in breastmilk, and
since SSRIs are a Category C, it may be a relative contraindication during breastfeeding.

Take home point: Treatment for obsessive-compulsive disorder includes an SSRI or clomipramine.

10 ©Test Pirates, LLC. All Rights Reserved.


Step 1 Practice Test #17 Explanations

29. 10yo boy has had anemia since birth. Spleen is five times normal. Splenectomy is indicated if anemia is caused by
which?
- Hereditary spherocytosis

Why it’s right: Hereditary spherocytosis is one of the few diseases for which a splenectomy is a treatment, especially for
the purposes of Step 1. The deficient protein in the RBC’s is part of the plasma membrane and cytoskeleton. Without a
perfect plasma membrane structure, the spleen perceives these RBCs, which are spheres, as normal and subsequently
removes them, even though they are still functioning as normal RBCs. As a result, anemia requires the splenectomy.
Other diseases that cause abnormal blood cells (i.e. other hemoglobinopathies, lymphoma, platelet disorders) may also
benefit from splenectomy.

Take home point: The treatment of choice for anemia caused by hereditary spherocytosis is splenectomy.

30. 62yo F in ED for 2-day hx of fever, abdominal tenderness, and painful urination. Agitated. T 38.8C, Labs show WBC
14k. Admitted to hospital, nurses note she has torn up four breakfast menus because she is confused. Cause?
- Delirium

Why it’s right: This patient has delirium secondary to an untreated urinary tract infection evidenced by fever, elevated
white blood cell count, and painful urination. This patient is at risk of developing sepsis, which would perpetuate the
delirious state. Delirium is different from dementia, and it’s important to distinguish between the two. Whenever there
is an underlying, identifiable cause, the diagnosis is likely delirium > dementia. Delirium is also more common to
precipitate in the acute inpatient setting shortly after admission. If hallucinations are present, it is almost always also
delirium (however, but don’t forget about Lewy body dementia!). The treatment would be to treat the underlying cause
– the infection.

Take home point: Delirium is usually secondary to an underlying cause such as an infection, trauma, substance abuse or
withdrawal, and/or a CNS disorder.

31. 60yo F with 3-year Hx of hyperlipidemia. Low-cholesterol diet and exercise program ineffective after 1 year.
Lovastatin initiated, but unable to tolerate greater than 20 mg daily. Additional drug is added that inhibits transport of
cholesterol through intestinal wall. Which drug?
- Ezetimibe

Why it’s right: First-line after a trial of diet and exercise to treat hyperlipidemia is a statin. If a patient cannot tolerate
statins, either due to muscle aches or pains from myositis, another medication should be initiated. A medication that has
a low likelihood of causing myositis is ezetimibe, a cholesterol absorption blocker that works at the level of the small
intestinal brush border. Another possible option is a bile acid resin like cholestyramine, which also decreases LDL. A
fibrate would be more appropriate in the setting of elevated triglycerides. Niacin is okay at lowering LDL, but it indicated
when HDL is low.

Take home point: Ezetimibe blocks cholesterol absorption at the small intestinal brush border.

32. 48yo M with bronzing of his skin, weakness, and fatigue during the past 3 months. PE shows hepatomegaly, and
small testes. Serum: AST INCREASED, ALT INCREASED, iron INCREASED, transferrin sat INCREASED, ferritin INCREASED,

11 ©Test Pirates, LLC. All Rights Reserved.


Step 1 Practice Test #17 Explanations

testosterone DECREASED, LH DECREASED, FSH DECREASED. Explanation?


- Increased intestinal iron absorption

Why it’s right: This patient has hemochromatosis. The most common sites for iron deposition are the liver, pancreas,
heart, and pituitary gland. The deposition of iron in the anterior pituitary gland leads to a decrease in LH, FSH, testicular
size = = hypogonadism. Severe liver damage results (indicated by elevated LFTs) and all measures of iron level are
increased. The only lab result that is decreased is TIBC, which increases when iron levels are low and decreases when
they are high. The pathophysiology of the disease is secondary to a genetic mutation (in the HFE gene) that results in
increased iron absorption at the level of the intestine.

Take home point: Hemochromatosis causes liver deposition throughout the body due to a genetic mutation that causes
increased iron absorption at the level of the intestine.

33. 45yo M with intermittent bloody diarrhea and abd pain. Sigmoidoscopy and rectal biopsy show IBD. Monoclonal
antibody is begun, which is directed against what components?
- Tumor necrosis factor

Why it’s right: Treatment for IBD which utilizes a biologic agent is typically infliximab or adalimumab, both of which
target TNF-alpha. These medications are also used when a biologic is indicated for the treatment of rheumatoid arthritis,
psoriatic arthritis, and ankylosing spondylitis. The underlying pathophysiology of IBD hinges on the action of TNF-alpha
in the body’s inflammatory response. With regard to IBD, it is high-yield to know the differences between Crohns
disease and ulcerative colitis. Also high-yield, IBD vs. IBS.

Take home point: Infliximab and adalimumab both target TNF-alpha and are used to treat IBD.

34. 41yo F with increasingly severe headaches for 6 weeks. BP 160/100. Bruit over left costovertebral angle. U/A normal.
Angiogram of left renal artery shows alternating areas of stenosis and aneurysmal dilatation ("string of beads" sign). Dx?
- Fibromuscular dysplasia

Why it’s right: These symptoms and findings are all typical of fibromuscular dysplasia, a tissue disorder that targets the
arterial walls of the kidney, leading to the formation of aneurysms à stenosis à decreased blood flow. Over time,
hypertension results due to perpetual RAAS activation because the kidneys are starved of blood/oxygen. Angiogram is
the diagnostic test of choice, which will show a “string of beads appearance” due to the presence of small aneurysms
and strictures along the afferent arteriole.

Take home point: Fibromuscular dysplasia results in the formation of aneurysms along the afferent arteriole of the
kidneys, leading to hypertension.

35. 55yo M with sepsis. Appears anxious and confused. Rx with vancomycin and ceftriaxone initiated in ED. T 39.8, P
132/min, BP 85/48. PE shows warm, flushed skin. No edema. Administer which solutions?
- 0.9% Saline

Why it’s right: Although septic shock does not typically respond to fluid administration, fluids can still be initiated
because the patient is “third-spacing” fluid due to the infection, and normal saline (0.9%) can be initiated to replace lost
fluid and increased blood pressure. Epinephrine should also be given to vasoconstrict vessels and increase blood
12 ©Test Pirates, LLC. All Rights Reserved.
Step 1 Practice Test #17 Explanations

pressure. In septic shock, the blood vessels vasodilate and blood pools in the periphery (which is the reason for warm,
flushed extremities/skin).

Take home point: All forms of shock should be treated with normal saline. Septic shock should also be treated with a
vasopressor such as epinephrine.

36. 36yo M undergoes elective liposuction under general anesthesia. Operation is terminated when patient develops
hyperthermia, tachycardia, and marked muscle rigidity. MOA of drug that should be administered?
- Decreases release of Ca from the sarcoplasmic reticulum

Why it’s right: This patient developed malignant hyperthermia, a potentially lethal side effect of inhaled anesthetics and
succinylcholine. It is marked by muscle rigidity, hyperthermia, and tachycardia (or a general destabilization of vital signs).
There is an inherited predisposition for malignancy hyperthermia (RYR1 gene), so it’s important to ask a patient prior to
undergoing a procedure requiring anesthesia if any family members have had complications with anesthesia. Treatment
is to stop the inciting agent and give dantrolene. The mechanism of action of dantrolene is that it prevents the release of
calcium from the sarcoplasmic reticulum in skeletal muscle, halting muscle excitation and contraction.

Take home point: Malignant hyperthermia is treated with dantrolene, which prevents the release of calcium from the
sarcoplasmic reticulum in skeletal muscle.

37. In a survey of 100 households (average three residents per household), 45 with asthma are detected. Prevalence?
- 15%

Why it’s right: Prevalence is the percentage of the population who currently has the disease – whereas incidence is the
percentage of the population who developed the disease (new cases) during a period of time. In this case, simple math
comes in handy – if there are 3 persons per household, and there are 100 households in the population, 3*100 = 300,
and 45 have asthma: 45/300 = 0.15 or 15%

Take home point: Prevalence is the percentage of the population who currently have the disease.

38. While lifting weights, 24yo M swelling in right inguinal region. Photograph shown of small intestine resected. Dx?
- Strangulation

Why it’s right: The photo is of necrotic bowel due to strangulation of intestine during an inguinal hernia that
precipitated while lifting weights. The difference between a direct and indirect inguinal hernia is high-yield. This patient
had a direct inguinal hernia, which is protrusion of the intestine through Hesselbach’s triangle, or the zone medial to the
inferior epigastric artery. The layers of tissue it prolapsed through squeezed off the blood supply to the intestine and
caused necrosis. They usually occur in older men or patients who weightlift. Indirect inguinal hernias are more common
in infants due to the failure of the processus vaginalis to close and are less likely to result in strangulation. Femoral
hernias are more common in women.

Take home point: Direct inguinal hernias are more common in older men and patients who lift weights. They are more
prone to strangulation (causing bowel necrosis) than indirect inguinal hernias.

13 ©Test Pirates, LLC. All Rights Reserved.


Step 1 Practice Test #17 Explanations

39. 24yo with second-degree burn. Two weeks after, tissue shows increased fibroblast migration and proliferation,
increased collagen and fibronectin, and decreased metalloproteinases. Caused by production of which?
- Transformation growth factor-beta

Why it’s right: TGF-beta has several roles including T-cell recruitment, induction of apoptosis, and cell cycle regulation. It
is also responsible for fibroblast recruitment during the wound healing process and promotes fibrosis. MMPs are
released from fibroblasts and breakdown collagen. TGF-beta “wants” to perpetuate fibrosis and collagen regeneration,
therefore it inhibits MMPs. TGF-alpha on the other hand, is involved more in the maintenance of granulomas and other
inflammatory states (i.e. Crohn’s disease).

Take home point: TGF-beta is involved in wound healing by recruiting fibroblasts and inhibiting MMPs.

40. Protein found in brown adipose tissue of mice causes leak of H ions inward across inner mitochondrial membrane.
Effect of this protein on oxidative phosphorylation and energy metabolism?
- Increased ratio of oxygen consumption to ATP generation

Why it’s right: At the start of the electron transport chain, NADH and FADH2 pass off their hydrogens from the inner
mitochondrial matrix to the intermembranous space (between the outer and inner mitochondria, since it is a double-
membraned organelle). A gradient of H+ builds in the intermembranous space, and when a hydrogen ion passes through
Complex V membrane protein back into the inner mitochondria (mito. matrix) it generates ATP + H2O. In this scenario, if
a protein allows the leakage of H+ back into the matrix without passage through the coupled transporter (Complex V),
less ATP is generated. Therefore, the ratio of oxygen consumption needs to be higher for the same amount of ATP to be
generated; in other words, the ratio of oxygen consumption to ATP production increases.

Take home point: The generation of ATP via the electron transport chain relies on the gradient of hydrogen ions –
specifically, H+ concentration in the intermembranous space should be higher than the inner mitochondria (AKA the
mitochondrial matrix), so that the flow through of H+ though an intermembranous protein transporter generates ATP.

41. 57yo M with alcoholism has distended abdomen with shifting dullness, fluid wave, caput medusae, palmar erythema,
spider angiomata. Additional finding?
- Gynecomastia

Why it’s right: These are the clinical sequelae associated with alcoholic cirrhosis, specifically end-stage. Several of these
findings are the result of increased levels of estrogen due to the inability of the liver to degrade estrogen. These include
spider nevi (or spider angiomata), testicular atrophy, palmar erythema, and gynecomastia, or enlarged breasts.

Take home point: Cirrhosis leads to increased estrogen levels due to the liver’s inability to degrade estrogen. High levels
of estrogen cause spider angiomata, testicular atrophy, palmar erythema, and gynecomastia.

42. 16yo girl with 2-year hx of fainting; increased in frequency during past 6 months. BP 110/80 supine and 60/40
standing. Neuro exam normal. Plasma shows undetectable noreipinephrine and marked increase in dopamine
concentration when standing. Deficiency of which?
- Dopamine beta-hydroxylase

14 ©Test Pirates, LLC. All Rights Reserved.


Step 1 Practice Test #17 Explanations

Why it’s right: This is a very specific question about the steps of catecholamine synthesis, which begins with
phenylalanine. The more common disorder due to a defect in this pathway is phenylketonuria, from a decrease in
phenylalanine hydroxylase, the first enzyme involved in the pathway. Later on, dopamine à norepinephrine via enzyme
dopamine beta-hydroxylase. NEà epinephrine via enzyme phenylethanolamine N-methyltransferase. Therefore, a
defect in dopamine beta-hydroxylase will cause increased dopamine upstream and decrease NE and epinephrine
downstream. Vitamin C is also needed for the dopamine à NE step.

Take home point: Defective or deficient dopamine beta-hydroxylase results in increased dopamine and decrease NE and
epinephrine (see catecholamine synthesis pathway).

43. 20yo M with suspected appendicitis has periumbilical burning and discomfort, localizes to RLQ 5 cm superomedial to
anterosuperior ilica spine. This is because periumbilical region and appendix are both supplied by afferent fibers in dorsal
root ganglia of which levels?
- T10

Why it’s right: The high-yield dermatomes are (besides all of them) T4 at the nipple line and T10 at the umbilicus. If you
memorized that, then this is an easy question and the dermatome Q you were hoping to get! In appendicitis, the fibers
first stimulated are visceral fibers that carry afferent innervation from both the tissue surrounding the appendix and the
periumbilical region – AKA referred pain. Later, the parietal peritoneum is inflamed, and the fibers that carry pain from
this region are not shared with any other region, localizing the pain to the inciting region – the RLQ, home to appendix.

Take home point: The T10 dermatome carries fibers from the periumbilical region. In appendicitis, visceral pain presents
first, followed by parietal pain.

44. 60yo M from china to USA with 1-month hx of confusion and swelling of abdomen and legs. Peanut farmer in china.
Smoked 1 pack per day for 40 years. No alcohol. PE shows ascites and ankle edema. Liver palpable, spleen isn't.
Serum: Albumin 4, total bili 2.5, Alk Phos 200, AST 45, ALT 60, IgG anti Hep A positive; Anti hep B negative, anti hep C
negative.
U/S shows 10 cm lesion. Biopsy shows dysplastic hepatocytes in small clusters and rows with no normal architecture.
Exposure to which?
- Aflatoxin

Why it’s right: This patient has a hepatic malignant process (likely hepatocellular carcinoma [HCC]) due to aflatoxin
exposure from peanuts. Though it’s not in First Aid (but may in future editions), aflatoxin exposure has been shown to
be associated with HCC. The labs show normal albumin, elevated bilirubin, elevated alkaline phosphatase, and the upper
limit of normal ranges for both AST and ALT. Based on these labs alone, there is evidence of a cholestatic process that is
blocking the biliary ducts at the hepatocellular levels – bili and alk phos are elevated, while AST and ALT are normal-high
values, very telling of a cholestatic process. The liver is also palpable and possibly enlarged due to the lesion, which
would not be the case in alcoholic cirrhosis which results in a smaller or nonpalpable liver. Biopsy shows dysplasia with
loss of normal architecture consistent with a malignant process.

Take home point: Aflatoxin is associated with hepatocellular carcinoma. Labs consistent with a cholestatic process can
result from a hepatic malignancy.

15 ©Test Pirates, LLC. All Rights Reserved.


Step 1 Practice Test #17 Explanations

45. 42yo M in ED for 5-hour hx of fever, chills, and severe pain and swelling of his left arm. Scratched his arm on a nail
yesterday. Appears confused, T 40C, BP 71/38. Labs show Hb 14, HCt 42%, WBC 15K (35% PMNs, 40% bands, 25%
lymphos), Platelets 50K, Serum BUN 28, Cr 2.8. Symptoms due to systematic release of which cytokines?
- IL-1 and tumor necrosis factor (TNF)-alpha

Why it’s right: It is important to recognize that this question isn’t asking, what’s the diagnosis? It’s asking, based on your
understanding of basic microbiology principles, what is the underlying disease process resulting in a systemic
inflammatory response and disseminated intravascular coagulation (DIC)? The inciting factor in this case is an endotoxin
from a gram-negative bacteria contracted after scratching his arm with a nail. The endotoxin has activated
macrophages, which release IL-1 and TNF-alpha, while other tissue factors are also activated by the endotoxin to initiate
DIC, the reason for the decreased platelet count and kidney damage.

Take home point: Endotoxin of gram-negative bacteria results in the activation of macrophages and tissue factor that
releases TNF-alpha and IL-1, resulting in DIC.

46. 62yo M dies suddenly while playing tennis. No cardiac risk factors, no hx of CAD. Autopsy, cardiac valve defect and
concentric LVH. Which valve abnormalities is most likely involved?
- Aortic stenosis

Why it’s right: Ventricular hypertrophy will typically result from the heart pumping against some form of resistance –
such as arterial resistance in patients with hypertension or a valvular resistance in patients with aortic stenosis; right
ventricular hypertrophy would be from pulmonary vasculature resistance like in patients with pulmonary fibrosis or
from pulmonic valve stenosis. The most likely cause in a patient with no previous risk factors (i.e. hypertension) is likely
aortic stenosis.

Take home point: Long-standing aortic stenosis leads to left ventricular hypertrophy.

47. 67yo M has urinary urgency after placement of urinary bladder catheter during transurethral resection of the
prostate. Most appropriate Rx has which MOA?
- Inhibition of muscarinic receptors

Why it’s right: The bladder has both muscarinic receptors (mostly M2, but M1-3 are present) and beta-receptors.
Activation of M2 leads to bladder contraction while activation of beta-receptors leads to detrusor relaxation. Therefore,
to treat bladder spasms (or overactive bladder) we must inhibit muscarinic receptors – i.e. oxybutynin. Other receptors
in the GU tract to be aware of are the alpha-receptors. The alpha-receptors are located in the urethra (and the trigone of
the bladder), and stimulation of these leads to decreased urinary output. Inhibition of these receptors is used to treated
BPH by causing smooth muscle relaxation and increased urinary output.

Take home point: Treat bladder spasms and incontinence by inhibiting muscarinic receptors of the bladder (mostly M2).

48. Mouse embryos are produced with two pronuclei, both of same parental origin. When the pronuclei are maternal,
products have poorly developed extraembryonic structures. When both pronuclei are paternal, products have poorly
developed embryonic tissue. Which genetic mechanisms?
- Imprinting

16 ©Test Pirates, LLC. All Rights Reserved.


Step 1 Practice Test #17 Explanations


Why it’s right: The answer here is not uniparental disomy, which is when offspring receive 2 copies of a chromosome
from one parent (instead of one copy from each). The embryos in this question are receiving genes from one parent,
true, but this does not always lead to defective offspring. The reason there is a defect, is because specific alleles of a
gene are imprinted, or inactivated by methylation, and since both copies were inherited from one parent, both copies
were inactivated/imprinted/methylated. To be normally developed, both a copy of DNA is needed from the dad and a
copy from the mom to have a complete set of DNA and develop normally. When both pronuclei were from the mom,
the extraembryonic structures were abnormal because genes that control for this development were imprinted on the
mom’s side, and the embryo received both from mom; the father’s DNA is imprinted for embryonic tissue. For
reference, please refer to the results from an experiment done in the 1980s upon which this question is based.1

1.McGrath J, Solter D. Complettion of Mouse Embryogenesis Requires Both the Maternal and Paternal Genomes. Cell 1984;37:179-83.

Take home point: Imprinting is the inactivation of an allele so that only one allele at that loci is expressed. Uniparental
disomy is when offspring receives 2 copies of an allele from one parent, which is part of the process but the not ultimate
cause of abnormal offspring.

49. 18-yo F with sepsis after an abortion. Within 24 hours she becomes dyspneic, oliguric, and develops petechiae,
ecchymoses, and bleeding from venipuncture sites. Which lab finding?
- Decreased plasma fibrinogen concentration

Why it’s right: This patient has sepsis secondary to an infection after an abortion. Sepsis has led to DIC evidenced by
oliguria, dyspnea, petechiae, and bleeding from all sites – a highly characteristic sign of DIC. Know the labs and expected
coagulation values. If you understand the pathophysiology of DIC, it is easier to memorize the labs: Basically, DIC is a
paradox of both clotting and bleeding. Clotting factors, fibrinogen, and platelets are being used up to create clots, which
perpetuates bleeding because all the factors are used up. Moreover, the clots are also being broken down, leading to an
increase in fibrin split products (D-dimers). Thus, fibrinogen is decreased, platelets are decreased, while bleeding time,
PT and PTT are all increased. Schistocytes are also found on blood smear.

Take home point: In DIC, fibrin split products are increased while plasma fibrinogen concentration is decreased.

50. 63yo M with 6-month hx of exertional chest pain relieved by rest. smoked for 45 years. Mild HTN, no meds. Which
lesion in LAD is most likely cause?
- Calcified 80% stenosis

Why it’s right: With risk factors that include age, smoking history, and untreated hypertension, this patient likely has
some degree of coronary artery disease as a cause of his stable angina – or chest pain relieved by rest. 100% stenosis
would be a myocardial infarction, so that’s not the correct answer. And typically, more than 60% of the artery needs to
be stenosed for a patient to be symptomatic, therefore 80% represents the best choice.

Take home point: Near complete stenosis of a coronary artery causes symptomatic angina. Complete stenosis causes an
MI.

17 ©Test Pirates, LLC. All Rights Reserved.
















Practice Test Explanations
Step 1 Practice Test # 17 – Block 4
Question Total: 50


Step 1 Practice Test #17 Explanations


Block 4

1. 65yo F with ovarian cancer treated with cyclophosphamide and other chemotherapeutic agents. Cyclophosphamide
affects which target?
- DNA replication

Why it’s right: Cyclophosphamide is an alkylating agent and creates crosslinks between strands of DNA at guanine N-7
positions. This prevents DNA replication. Cyclophosphamide must be bioactivated by the liver because its metabolite is
what forms the crosslinks. It is used to treat solid tumors, leukemia, lymphoma, SLE (especially lupus nephritis), and
polyarteritis nodosa. Also high-yield about this medication is that along with myelosuppression, it can cause
hemorrhagic cystitis, which is partially prevented with the drug mesna.

Take home point: The mechanism of action of cyclophosphamide is crosslinking DNA at guanine N-7 to interfere with
DNA replication.

2. Pharma company trying to develop a long-acting weight-loss agent that mimics activity of a naturally occurring
peptide originates in adipose tissue, signals brain about stored fat, and suppresses appetite by its action in the CNS.
Which chemical mediator?
- Leptin

Why it’s right: The main hormone made by adipose cells that helps in regulating hunger is leptin. It acts on receptors in
the hypothalamus: within the lateral area of the hypothalamus, leptin acts on receptors to inhibit hunger since the
lateral area is responsible for increasing hunger, and destruction of the lateral area results in anorexia. Conversely,
within the ventromedial area of the hypothalamus, leptin acts on receptors to stimulate them and increase satiety
(decreasing hunger as well). Therefore, overall, leptin suppresses appetite.

Take home point: Leptin is a hormone secreted by adipose cells and suppresses appetite.

3. 45yo F has thyroidectomy because of asymmetric enlargement of thyroid noticed 6 weeks ago. Underwent
adrenalectomy for pheochromocytoma 3 years ago. Bilateral thyroid lesions with spindle cells arranged in small clusters.
Amyloid deposits b/w neoplastic cells. C-cell hyperplasia, malignant. Which marker to monitor?
- Calcitonin

Why it’s right: The diagnosis is multiple endocrine neoplasia 2A (MEN 2A) since the patient has had a
pheochromocytoma, and now has medullary thyroid carcinoma, which secretes calcitonin. Therefore, the marker to
monitor to measure disease activity is calcitonin, which would be pathologically high. Other MEN 2A manifests include
cutaneous lichen amyloidosis. The diagnosis is less likely MEN 2B because, though this disease is also characterized by

2 ©Test Pirates, LLC. All Rights Reserved.


Step 1 Practice Test #17 Explanations

medullary thyroid carcinoma and pheochromocytoma, patients with MEN 2B also have oral/intestinal
ganglioneuromatosis and a marfinoid habitus.

Take home point: In MEN 2A and 2B, calcitonin is a known tumor marker and can be monitored to measure disease
activity because it is secreted by medullary thyroid carcinoma associated with both diseases.

4. 45yo F intubated, mechanically ventilated with fungemia with Candida albicans. Rx with caspofungin is started.
Feature of causal organism targeted by this drug?
- Beta-Glucan carbohydrates in the cell wall

Why it’s right: The question is asking the mechanism of action of caspofungin, which is inhibition of cell wall synthesis by
inhibiting the synthesis of beta-glucan, a key component of the fungal cell wall. Micafungin also shares the same
mechanism of action. Both can be used to treat invasive aspergillosis and Candida. Side effects include GI upset and
flushing secondary to the release of histamine.

Take home point: The mechanism of action of capsofungin is inhibition of beta-glucan synthesis, a key component of the
fungal cell wall.

5. 56yo M 4 hours after sudden onset of uncontrollable irregular movements of the left side of the body. PE shows flailing
movements of the proximal appendicular muscles on the left. Nuclei damaged?
- Right subthalamic

Why it’s right: Hemiballismus describes uncontrolled, flailing movements of one half of the body (hence the “hemi-“). It
is secondary to a lesion at the contralateral subthalamic nucleus, a lesion such as a lacunar stroke of sudden onset like in
this patient. The subthalamic nucleus controls movement inhibition on the contralateral side, so injury
results in uncontrolled movement on the side opposite the lesion.

Take home point: Hemiballismus is secondary to a lesion/injury to the contralateral subthalamic nucleus.

6. 60yo M 1-month Hx of progressive SOB with exertion.


Breath sounds: Dec on right lung base, normal on left lung base
Percussion note dull on right lung base, nml on left lung base
Tactile fremitus decreased on right lung base, nml on left lung base
Adventitious sounds none on right lung base, crackles on left lung base
Dx?
- Pleural effusion

Why it’s right: There are only a few things that can cause all these specific right lung findings – fluid. Typically, fluid
surrounding the lung, like in a pleural effusion, will cause “dulling” of everything, or a muting of all findings on physical
exam. If it was a filling process such as an infectious pneumonia, sounds and exam findings would be increased – like
tactile fremitus. In the case of a collapsed lung, at least one side would be tympanic and not dull to percussion. Fibrosis
may also cause a decrease in sounds, but fremitus may still be normal to increased. Finally, in this case, the crackles on
the left lung base are likely due to pulmonary edema or fluid spillover from the underlying disease process causing the
pleural effusion in the first place.

3 ©Test Pirates, LLC. All Rights Reserved.
Step 1 Practice Test #17 Explanations

Take home point: A pleural effusion will cause a “dulling” of all findings: decreased breath sounds, dullness to
percussion, and decreased tactile fremitus.

7. 68yo M with difficulty swallowing solids for 2 months. Hx of dilated cardiomyopathy. X-rays of esophagus w/ barium
contrast show indentation and posterior displacement of the esophagus. Enlargement of what caused dysphagia?
- Left atrium

Why it’s right: This is an anatomy question, basically asking which structure is anterior to the esophagus at the level of
obstruction. This patient has a dilated cardiomyopathy which causes an enlargement of all chambers, including the left
atrium, which is anterior to the esophagus and the cause of mechanical obstruction causing dysphagia.

Take home point: The left atrium is the heart chamber that is closest to the esophagus and anterior to it. Enlargement
of the left atrium can result in mechanical obstruction of the esophagus.

8. 25yo F with 3-year hx of irregular menses. Menarche was at age of 14 years. BP 116/62. PE shows increased hair
growth on the face and chest. Pelvic exam shows clitoromegaly and a normal-appearing uterus. Serum shows increased
17-hydroxyprogesterone and androstenedione. Deficiency of ?
- 21-hydroxylase

Why it’s right: The most common form of congenital bilateral adrenal hyperplasia is 21-hydroxylase deficiency. In
females, this deficiency causes masculinization leading to pseudohermaphroditism. The enzyme that is deficient is
responsible for converting progesterone to aldosterone (so hypotension results), hydroxyprogesterone to cortisol (also
resulting in hypotension) and a shunting of the upstream hormones to testosterone, DHT and estradiol (leading to
masculinization in males). Evidence of sexual changes is typically not present in males with this enzyme deficiency.

Take home point: A deficiency in enzyme 21-hydroxylase results in masculinization in females and hypotension in both
males and females due to an increase in sex hormones and a decrease in aldosterone and cortisol.

9. 60yo M in ED for sudden onset of acute abdominal pain and tenderness, nausea, vomiting, and bloody diarrhea 2
hours ago. He has a Hx of cirrhosis and hepatocellular carcinoma. BP 99/50. Loss of bowel sounds. Surgery shows small
intestine with dark purple-red hemorrhagic appearance. Cause?
- Mesenteric venous thrombosis

Why it’s right: In a patient with a history of cirrhosis and hepatocellular carcinoma, there is a buildup of blood
throughout the body due to the liver’s inability to drain the portal circulation, which results in findings like hemorrhoids,
caput medusa, and esophageal varices. Stasis leads to thrombosis, and on gross pathology, dark, hemorrhagic (==
necrosis!) bowel is diagnostic of ischemia secondary to thrombosis. In this case, the small intestine is necrotic, and the
small intestine is drained by the superior mesenteric vein. Another association with cirrhosis is pylephlebitis, which is
thrombophlebitis of the portal vein caused by infection (low-yield point).

Take home point: Necrotic and hemorrhagic small bowel in the setting of cirrhosis is secondary to thrombosis of the
mesenteric vein (specifically the SMV).

4 ©Test Pirates, LLC. All Rights Reserved.


Step 1 Practice Test #17 Explanations

10. 38yo M in ED 30 min after unable to stand upright. Lethargic, pulse 110/min, BP 90/62. PE shows dry mucosa and
poor skin turgor. Midepigastric tenderness. Labs show:
Serum: Na 143, K 3.2, Cl 101, HCO3 11
ABG: pH 7.28, Pco2 23, Po2 98
Acid-base status in this patient?
- Metabolic acidosis

Why it’s right: This patient has a metabolic acidosis based on the following: first, the pH is <7.4 so regardless of any
compensating forces, this is an acidosis. Second, the HCO3 is less than 24, meaning it is the cause of the acidosis since
there is less of bicarb, which is basic and in excess would raise the pH. Because the HCO3 matches the acidotic status, it’s
a metabolic acidosis (vs. a respiratory acidosis, which would be the case if the pCO2 >40). Here, the pCO2 is <40, so the
patient is hyperventilating and blowing off CO2 to compensate for the acidosis. In terms of the cause, it is unclear based
on the given information, but could be due to salicylate poisoning because that can cause midepigastric pain. Another
component of this diagnosis that we are able to confirm is that it is an ANION GAP acidosis based on the equation: Na –
(Cl + HCO3) = 143 – 101 – 11 = 31.

Take home point: Diagnose a metabolic acidosis when the pH is < 7.4 and HCO3 < 24 and a respiratory acidosis when pH
< 7.4 and pC02 > 40.

11. 2mo boy given vaccine to convert T-independent antigens to T-dependent forms to enhance protection in young
children. Which vaccine given?
- Haemophilus influenzae type b

Why it’s right: Vaccines induce both cellular (mainly T cells) and humoral (mainly B cells) immunity. Cellular immunity is
more robust than humoral immunity and is induced by a live-attenuated pathogen (virus or bacteria) as opposed to an
inactivated or killed pathogen. H flu (haemophilus influenzae) induces mainly a humoral response based on the vaccine
components that target the main component of the bacterial capsule – polyribosyl ribitol phosphate (PRP). However,
when PRP of H flu is conjugated to a protein (i.e. diphtheria toxoid), it induces mainly a cellular response, described in
the question above as conversion of T-independent antigens to T-dependent forms.

Take home point: The H flu vaccine is conjugated to a protein such as diphtheria toxoid in order to enhance the vaccine
and induce a cellular immune response via T cells.

12. A study conducted to assess effectiveness of injections of lidocaine into "trigger points" of pain symptoms in patients
with fibromyalgia. Fifty patients randomly assigned - 0.9% saline only or saline plus lidocaine. Graph shows self-reported
pain scores. Explanation?
- Placebo effect

Why it’s right: The placebo effect is a false positive response to the control or sham drug and is used to compare against
the main intervention. Saline is injected in half of the patients to control for confounders and the placebo effect. These
patients had only saline injected, which would not relieve pain, and yet they still reported a positive response = =
placebo effect! This is the baseline placebo effect in the non-treatment arm, so if we look at the treatment arm, we can
subtract the placebo effect measured in the control arm from the treatment arm, and that yields the actual effect of the
treatment without the placebo effect confounding the results.

Take home point: The placebo effect is a false positive response to the control medication.
5 ©Test Pirates, LLC. All Rights Reserved.
Step 1 Practice Test #17 Explanations

13. 58yo F in ED for 2-hour hx of SOB and chest pain radiates to her back between the shoulder blades. Resp 28/min, PE
shows diaphoresis. ECG normal. Coronary angiography shows occlusion of marginal branch of LAD coronary artery.
Revascularization done with stent placement. CK-MB and troponin I increased. Mechanism of these lab findings?
- Peroxidase mediated membrane damage

Why it’s right: This patient is having a myocardial infarction evidenced by symptoms, history, and the presence of a
thrombus actually found in the LAD. The normal ECG is thrown in to make the question more difficult but is still
indicative of an MI because women defy the odds when it comes to heart attacks. Upon revascularization with the stent,
this patient suffers reperfusion injury. When fresh blood flows through the vessel that was being starved of blood during
the MI, reactive oxygen species form due to the sudden increase in oxygen supply, and damage the cell membranes via
lipid peroxidation. This results in cardiogenic factors spilling out of the cell, namely CK-MB and troponin.

Take home point: A rise in CK-MB and troponin during a heart attack results from lipid peroxidation of cell membranes,
leading to spillage of these factors into the bloodstream.

14. In a clinical study, a polymorphic marker with three alleles, 1, 2, and 3, is found to be tightly linked to the gene for
polycystic kidney disease. Pedigree shown. If III, 1 is unaffected by this disease, patient is most likely carrier of?
- 2,3

Why it’s right: This question shows that you must know how to read a pedigree and be able to identify inheritance
mode when the question does not directly tell you. We know polycystic kidney disease can be both AD and AR, so that
narrows it down a bit – and rules out other commonly tested pedigree-presented modes such as X-linked recessive not
to be confused with a tricky favorite, mitochondrial inheritance! Then basically to solve the question, draw the Punnett
square for each individual since they give information on one patient who is unaffected by the disease. And remember,
the question is asking about the unaffected individual and not the diseased individual.

Take home point: You must be able to identify modes of inheritance (including AR, AD, X-linked recessive, and
mitochondrial) based on a given pedigree.

15. 6yo girl with 4-day hx of round shiny bumps in areas where she has eczema. Her mother saw similar bumps on a
playmate at pool party 3 weeks ago. No other Sx. PE shows firm, smooth, umbilicated papules 2 to 4 mm diameter in
clusters. Causal organism?
- Poxvirus

Why it’s right: The diagnosis is molluscum contagiosum (MC) for which the causative virus is poxvirus, a cause of also
smallpox and vaccinia, or cowpox. MC is more common in children, especially when sharing towels or other items that
touch skin and spread the virus. It is also more common in immunocompromised patients. A tertiary version of this
question would be which type of virus is the causal virus? Answer: DNA, double stranded, linear virus with an envelope.

Take home point: Poxvirus causes molluscum contagiosum.

16. 80yo F in ED for 2-day hx of "feeling funny." "Lost my pep." Hx of poorly controlled hypertension. Just started
medication 2 weeks ago. BP 130/85. Physical exam is normal. Serum potassium is 3. Which drug?
- Hydrochlorothiazide

6 ©Test Pirates, LLC. All Rights Reserved.


Step 1 Practice Test #17 Explanations


Why it’s right: Generally, thiazide diuretics can cause lethargy because they decrease potassium, as in this patient. This
question could have made more challenging if they didn’t give the serum potassium and asked, which lab do you want
to order? Or even still, which diuretic is this patient taking? The real USMLE will give less supporting information and you
are expected to infer some facts – at least someone who is aiming for 240+ should be able to do this. Thiazide diuretics
can cause, besides a hypokalemic metabolic alkalosis, hyponatremia, hyperglycemia, hyperlipidemia, hyperuricemia, and
hypercalcemia (hyperGLUC).

Take home point: Hydrochlorothiazide can cause a hypokalemic metabolic alkalosis, which results in lethargy.

17. 69yo F with weakness of left leg since awakening. PE shows weakness. Babinski present on left. Decreased somatic
sensation in left foot, agraphesthesia on plantar surfaces of the toes, and decreased position sense in the toes. MRI
shows edematous area in cerebral cortex of right hemisphere. Lesion?
- G, the primary somatosensory/motor cortex

Why it’s right: When thinking about brain lesions, it is easiest to first think about the homunculus in a coronal cross
section, and then, within a sagittal section, if the lesion is anterior or posterior to the central sulcus. In terms of the
homunculus, the lower extremity is controlled by the top part of the brain, which is affected in this patient. In terms of
the central sulcus, mainly motor control is controlled anterior to the central sulcus, while sensory control is posterior to
it. This patient has both weakness and a positive Babinski (representing an upper motor neuron lesion), as well as
decreased proprioception and sensation. The closest letter to include all these findings is represented by G involving the
somatosensory and somatomotor cortex.

Take home point: Most brain lesions can be solved by a simple 2-step process: 1) determine which part of the
homunculus is affected; 2) whether the lesion is anterior or posterior to the central sulcus.

18. 68yo M in for a hemiorrhaphy. Surgeon gives info of risks and benefits. Patient says that he understands what he has
been told, and his family will be able to discuss later. In this patient, which combination of components fulfill the criteria
for fully informed consent?
- Information, competence, voluntariness

Why it’s right: Informed consent is not just about providing information on the procedure and obtaining a signature. A
patient has to be competent to sign, meaning the language is appropriate (both in basic terms as well as in the patient’s
native language) and the patient has competence to sign – i.e. no dementia, not under the influence of a
substance/drug. Also, it has to be voluntary, or in other words, the patient has to be willing to want to undergo the
surgery or procedure. If there is any reluctance sensed by the physician, then the risks and benefits should be re-
explained before obtaining a signature for consent or going an alternate route – seeking consent from the next of kin.

Take home point: Informed consent is not just about a signature on a form explaining the surgery. There must also be
competence and voluntariness on the part of the patient.

19. 47yo F with psoriasis for follow-up. Was given several topical creams, to be used in specified sequence twice daily. No
improvements apparent at this appointment. How to begin discussion of compliance?
- "Most people find it difficult to adhere to a routine, how did you do?"

7 ©Test Pirates, LLC. All Rights Reserved.


Step 1 Practice Test #17 Explanations

Why it’s right: Compliance can be difficult to address with patients because it is important not to come across as
accusatory. The correct answer here is the perfect choice (which is rare for an ethics questions, when usually all the
choices or none of them seem perfect) because the physician first empathizes with the patient (“I know it’s difficult to
adhere to routine”) and then asks an open-ended question (which the USMLE loves in these ethics questions), “how did
you do?” It would also be perfectly fine to ask, “realistically, how often did you use the cream?” Overall, it’s the open-
endedness that can apply to all types of these questions.

Take home point: Open-ended questions are the best way for a physician to start a discussion on medication
compliance.

20. Male newborn has macrocephaly with poor skull mineralization, shortened extremities with misshaped long bones,
and several fractures. Defect in which?
- Collagen

Why it’s right: The diagnosis is osteogenesis imperfecta (OI), a genetic bone disorder that results in abnormal type I
collagen. Specifically, there is a defect in forming the triple helix structure of collagen. These patients demonstrate poor
skull mineralization, fractures of long bones resulting from minimal trauma, blue sclera (due to a thin sclera allowing the
blue choroid to show through), hearing loss, and dentin loss in teeth. The most common form of OI is inherited in an
autosomal dominant pattern.

Take home point: Osteogenesis imperfecta is due to a defect in collagen formation, specifically the formation of the
triple helix that makes collagen strong.

21. 27yo F in ED 30 min after ejected through windshield during MVC. Unrestrained front-seat passenger. PE shows
marked edema and tenderness of the jaw. Panorex x-ray of mouth shown. Which is injured?
- Inferior alveolar nerve?

Why it’s right: The inferior alveolar nerve is a branch of the mandibular nerve and travels along the lower jaw, giving off
a branch to the mental nerve, which supplies the front chin and lower lip sensation. Sensation to all of the lower teeth is
also supplied by the inferior alveolar nerve. Another common version of this question is presented in the setting of
dental work for a root canal or filling. Oftentimes, an injection though the mouth aiming at the angle of the mandible is
performed to anesthetize the lower teeth on the side of the procedure. Sensation to the upper teeth is supplied by the
superior alveolar nerve, which is divided into the anterior, middle, and posterior branches.

Take home point: The inferior alveolar nerve supplies sensation to the lower teeth, lower lips, and through the mental
nerve, the chin.

22. Girl for well-child exam. Normal development includes pincer grasp, finger feeding, standing while holding onto a
table, and playing peekaboo. Age (in months)?
- 9

Why it’s right: The stages of development should be memorized because these types of questions will reappear on Step
2 and 3. Walking is always an easy one to remember – by age 12 months. If this child can stand while holding onto a
table but has not taken her first steps yet, then she is likely just shy of 12 months, and 9 months fits best with her other
milestones including the pincer grasp.

8 ©Test Pirates, LLC. All Rights Reserved.


Step 1 Practice Test #17 Explanations


Take home point: At 9 months, children should be able to perform the pincer grasp, play peekaboo, and stand with
support (but not yet walk).

23. 60yo M exam prior to employment. PE normal. Labs show


Hb 14, HCT 42%, WBC 12k (PMN 45%, lymphos 50%, Monos 5%), platelets 250k
Flow cytometry analysis of peripheral lymphocytes shows:
CD3 50%, CD4 40%, CD8 10%, Kappa 47%, lambda 3%
Which is most predictive of a clonal lymphoid proliferation?
- Surface kappa:surface lambda ratio

Why it’s right: This patient has a clonal lymphoid proliferation confirmed by the kappa:lambda ratio >2. The two types of
light chains (of immunoglobulins) are kappa and lambda. Because immunoglobulins are produced by B cells, each B cell
only produces one type of light chain. Therefore, if there is an excess of either light chain, it means there is a clonal
population of B cells. In this question, the ratio is 47:3 > > 2:1, so a monoclonal proliferation of was discovered on this
seemingly routine exam.

Take home point: A kappa:lambda ratio >2 signifies a malignant clonal lymphoid proliferative process.

24. 18yo M with yellow nodules on Achilles tendons of his feet and extensor tendons of his hands. Exam shows collection
of foamy histiocytes within the dermis. Serum cholesterol is 980, and lipoprotein electrophoresis shows a selective
increase in LDL. Underlying disorder?
- Absence of functional LDL receptors in hepatocytes

Why it’s right: There are three main familial dyslipidemias to be aware of, and each one of them is distinguished based
on the type of cholesterol increased in the blood. This patient has familial hypercholesterolemia, an autosomal
dominant disorder caused by absent or decreased LDL receptors in the liver, meaning LDL is not removed by the liver
form the bloodstream. This results in accelerated atherosclerosis, tendon xanthomas, and corneal arcus. The foamy
histiocytes represent accumulation of lipid within the cells.

Take home point: Elevated LDL in a young patient with tendon xanthomas represents familial hypercholesterolemia
caused by an absence of LDL receptors in hepatocytes.

25. 35yo M with 4-day Hx of high-grade fever, sever muscle aches, malaise, loss of appetite, and a nonproductive cough.
wife and kids had similar illness. Temp 39.2, PE normal. CBC and CXR normal. Causal virus replicates its genome within
the cell's nucleus. Organism?
- Influenza virus

Why it’s right: This is the typical history and disease course of the influenza virus – high-grade fever, muscles aches, and
cough. In addition, patients may also exhibit nausea, vomiting, and nasal congestion. Muscles aches are highly specific
for the flu, meaning that if this symptom is present in the setting of other cold/flu symptoms, the cause is likely
influenza. CBC and CXR are classically normal because the virus affects the upper respiratory system. However, a
secondary bacterial pneumonia can develop after. This virus is highly contagious, which explains why other family
members had similar symptoms.

9 ©Test Pirates, LLC. All Rights Reserved.
Step 1 Practice Test #17 Explanations

Take home point: Muscle aches in the setting of cold/flu symptoms (i.e. fever, cough) are highly specific for the flu. CBC
and CXR will typically be normal unless another disease process is ongoing.

26. 28yo F at 18 weeks' gestation has palpitations. Labs show increased serum total thyroxine (T4) concentration. Best
test to confirm hyperthyroidism?
- Free T4

Why it’s right: A normal pregnancy in the absence of hyperthyroidism leads to increased thyroid activity, since increased
thyroid hormones benefit the fetus. Most importantly, thyroid-binding globulin (TBG) concentration increases, binding
free T4, lowering free T4, which leads to an increase in thyroid stimulation through increased TSH. As a result, total
thyroid hormone increases due to increased TBG, free T4 remains the same and TSH also remains within reference
range. This is all NORMAL. In a hyperthyroid state, no matter the cause, the most important hormone to measure is free
T4. Since TBG is increasing the total T4 in a NORMAL state, an increase in free T4 would be indicative of
HYPERthyroidism.

Take home point: To confirm the diagnosis of hyperthyroidism during pregnancy, measure free T4.

27. 21yo M in ED 45 minutes after sustaining multiple injuries in a MVC. His BP is 90/50, PE shows diffuse abdominal
tenderness. Dx with laceration of the spleen and undergoes splenectomy. Predisposed to infection with?
- Streptococcus pneumoniae

Why it’s right: The spleen has a special function in clearing infections, in that it opsonized encapsulated bacteria; in
other words, it makes them more prone to immune system clearance by tagging them for attack by the human body.
Without the spleen tagging them, these encapsulated organisms cause serious infections including sepsis because the
immune system cannot clear them as easily or at all. Oftentimes, this question will be presented with just a history of a
motor vehicle accident, WITHOUT disclosing the previous surgery of a splenectomy. Because the spleen is the most likely
abdominal organ to be compromised, bleed, and require surgical removal after blunt trauma, you must assume the
patient had a splenectomy if they were in an MVC and now have sepsis due to an encapsulate organism. Remember,
these organisms include: Streptococcus pneumoniae, Haemophilus influenzae type B, Neisseria meningitides,
Escherichia coli, Salmonella, Klebsiella pneumoniae, and group B Strep.

Take home point: Splenectomy patients are at risk for serious infection by encapsulated bacteria because the spleen is
responsible for opsonizing encapsulated bacteria.

28. 70yo F in longitudinal study of effects of aging on pulmonary function tests. Which represents woman now compared
with results at age of 20 years?
- Residual volume UP, Arterial Po2 DOWN, Alveolar-arterial Po2 difference UP

Why it’s right: A pulmonary physiology question is guaranteed, especially in the format of up and down arrows! As a
person ages, their residual volume increases because the lungs are less elastic with time and cannot force out as much
air. Alveoli are delicate structures, and even in nonsmokers they lose some of their recoil ability with age. As a result, a
V/Q mismatch results with decreased ventilation and arterial partial pressure of oxygen decreases. If decreased arterial
pO2 is plugged into the A-a gradient calculation, then the gradient also goes up, which makes sense because there is loss
of functioning alveoli with the ability to deliver oxygen to the pulmonary blood circulation.

10 ©Test Pirates, LLC. All Rights Reserved.


Step 1 Practice Test #17 Explanations

Take home point: As a person ages, the lungs lose some of their elasticity and ability to recoil, leading to increased
residual volume, decreased pO2, and an increase in the A-a gradient.

29. 45yo F farmer in ED for 2-day hx of confusion, lethargy, fever, headache, muscle pain, vomiting, and a rash on her
wrists and ankles. Bitten by a tick a few days ago. T 38.5 C, Red-purple papules on distal extremities progress to trunk.
Rx?
- Doxycycline

Why it’s right: This patient has Rocky Mountain spotted fever due to Rickettsia rickettsii. The rash typically starts on the
wrists and ankles and then spread to the trunk and can include the palms and soles. Know the main infections for which
the rash includes the palms and soles since there are only a few: Rocky Mountain, hand foot mouth disease,
Coxsackievirus, and syphilis. The rash of Rocky Mountain is actually a vasculitis, and that’s why it’s described as red-
purple papules. The classic triad of the disease is headache (or other cerebral issues like confusion, mental status
changes), fever, and rash. Muscle pain is also characteristic of this infection. Almost all of the vector-borne
(meaning intermediate host like a tick) are treated with doxycycline.

Take home point: Treat Rocky Mountain spotted fever with doxycycline.

30. Study of breast cancer in women. Hundred healthy women observed for 10 years. Goal is to determine if number of
family members who previously received dx of breast cancer correlates with incidence of future development of cancer.
Best design?
- Cohort

Why it’s right: This is a cohort study because it is looking at risk factors (family history) and determining if that factor is
correlated with the development of disease. Cohort studies can be prospective and retrospective. This is a prospective
cohort. A retrospective cohort would look at charts of patients and find all those with a family history and see if they
developed breast cancer. Cohort differs from case-control, which first identifies people with disease (usually used for
rare diseases). Then it looks at people without the disease, and compares the different exposures between the two
groups. A case-control study can never be prospective for that reason. Finally, if some intervention was employed, then
this would be randomized clinical trial (RCT).

Take home point: A cohort study first looks at risk factors and determines if these risk factors correlate with the
development of a disease. A cohort can be prospective or retrospective, while a case-control study can never be a
prospective study.

31. 62yo F with recurrent pulmonary emboli comes for follow-up. PE normal. Labs show PT of 12 seconds. Warfarin
begun. Which clotting factors is first to be decreased by 50% after initiation of Rx?
- VII (proconvertin)

Why it’s right: The mechanism of action of warfarin is inhibition of epoxide reductase, which is an enzyme that turns
oxidized vitamin K into reduced vitamin K, a necessary cofactor for factor VII, as well as factors II, IX, X and proteins C
and S. Because factor VII has the shortest half-life, it is the first factor to decrease the most. This is also the reason that
the prothrombin time (PT) is followed in evaluating the appropriate warfarin dose – because factor VII is part of the
extrinsic pathway measured by the PT. Heparin is monitored via the PTT, or partial thromboplastin time, which
measures the intrinsic pathway.

11 ©Test Pirates, LLC. All Rights Reserved.


Step 1 Practice Test #17 Explanations


Take home point: Factor VII has the shortest half-life of the factors inhibited by warfarin and is the reason PT is
measured to monitor the dose response to warfarin.

32. 48yo F with 2-month hx of fatigue and intermittent headaches. BP 180/110, PE normal. Serum show a decreased
potassium concentration and increased aldosterone. CT abdomen shows tumor on adrenal gland. Which additional
findings supports aldosterone-secreting adrenal adenoma?
- Decreased plasma renin activity

Why it’s right: Lots of these positive and negative feedback questions will be on all the step exams, and most heavily
step 1 because understanding physiology is part of the basic science of medicine. This question gives signs and
symptoms of a pathological hormone process causing elevated blood pressure, which has led to chronic fatigue and
headaches. An aldosterone-secreting adenoma of the adrenal gland will increase blood pressure, because that is what
aldosterone does, and increase the excretion of K+ through the kidneys, decreasing serum K+. An increase in
aldosterone will have negative feedback on the RAAS system, decreasing renin. Renin increases when blood pressure is
low or when the kidneys are starved of blood – i.e. renal artery stenosis.

Take home point: An aldosterone-secreting adrenal adenoma will increase blood pressure, decrease serum K+, and
decreases renin.

33. 20yo F with 1-day hx of increasing urinary frequency and a burning sensation with urination. One sexual partner, uses
condoms. VSS. PE shows mild suprapubic tenderness to deep palpation. Urine shows rare epithelial cells and 10 WBC/hpf.
Urine grows E. coli > 100k. This bacterial strain has which virulence factors?
- Mannose-binding (type 1) fibriae

Why it’s right: In the setting of sexual activity (even with one partner, condom use, etc), a sexually transmitted infection
must always be considered. However, this patient shows signs and symptoms that are more specific for a urinary tract
infection (UTI) such as suprapubic tenderness to palpation. Urine analysis and culture confirm the diagnosis. The
microbiological high-yield fact here is that E. coli has fimbriae, or an attachment finger-like projection, also known as a
pilus. The fimbriae of E. coli can adhere to D-mannose residues on mucosal cells of the inner bladder wall, initiating and
perpetuating an infection that, if not treated, can turn into pyelonephritis.

Take home point: E. coli has a key virulence factor known as a fimbriae or pilus that binds mannose residues on the
mucosal cells of the inner bladder wall.

34. 6yo boy with 3 systemic infections with Neisseria meningitidis over the past 2 years. Healthy otherwise. Which lab
test is most likely abnormal?
- Total hemolytic complement concentration

Why it’s right: Because Neisseria is cleared by the immune system via complement, repeated infection by this organism
at a young age is a big clue that this patient suffers from complement deficiency. In complement deficiency, the total
hemolytic complement concentration, which essentially measures complement, would be abnormal.

Take home point: Patients deficient in complement immune factors suffer from repeated Neisseria infections because
this organism is primarily cleared by the immune system via complement.

12 ©Test Pirates, LLC. All Rights Reserved.


Step 1 Practice Test #17 Explanations

35. 13yo girl 2/6, holosystolic murmur heard best over left fifth intercostal space adjacent to the sternum; it increases
with inspiration. Abnormality of which valves?
- Tricuspid

Why it’s right: Diagnosing a heart murmur can seem overwhelming, especially if you have to put the headphone on to
hear it!, but it’s important to break it up into steps. First, is it systolic or diastolic? In this case, it’s systolic. On the left
side of the heart, systolic murmurs are aortic stenosis and mitral regurgitation (MR); while diastolic murmurs are the
converse. On the right side of the heart, systolic murmurs are pulmonic stenosis and tricuspid regurgitation (TR). A
holosystolic murmur is typically due to a regurgitative process rather than a stenotic one. By now, we’ve narrowed it
down to MR or TR. The last key piece of information is that it increases on inspiration, which is highly suggestive of a
right-sided valve condition. Tricuspid regurgitation is the diagnosis. We didn’t even have to know the murmur’s location
to make the diagnosis, though a left 5th intercostal space murmur confirms the diagnosis.

Take home point: Tricuspid regurgitation causes a holosystolic murmur heard in the left 5th intercostal space adjacent to
the sternum and increases with inspiration.

36. 55yo F with left flank pain and gross hematuria. Mass is palpable in LUQ of abdomen. Ultrasonography shows a 12-
cm solid mass on lower pole of left kidney. Angiograms show hypervascular mass. Dx?
- Renal cell adenocarcinoma

Why it’s right: Hematuria in a patient over 50 years old with a palpable mass is renal cell carcinoma (RCC) until proven
otherwise. Though the kidney is in the retroperitoneal space, a large RCC can be palpated on abdominal exam. The tests
confirm the diagnosis since the mass is solid (versus a more benign cystic mass) and hypervascular, demonstrated by
angiogram. This was a pretty straightforward question, but it’s important to pay attention to the extra information that
the test makers are giving you, that in future tests may be asked – i.e. they may ask, what’s the best next step? The
answer would be what they did in the stem – an ultrasound. A CT with contrast is also a good option because then it
would reveal the vascular component. The next, next best step would be excisional biopsy, namely a nephrectomy.

Take home point: Suspect renal cell carcinoma in any patient over 50 years with hematuria and a palpable abdominal
mass.

37. 28yo F with 3-week hx of double vision. Episode of blurry vision in left eye that improved during next 2 months w/o
Rx. Neuro exam shows right eye does not adduct past the midline on horizontal gaze when looking to the left. When
convergence is tested, the right eye is able to adduct past the midline. Rightward horizontal gaze is normal. Lesion is
where?
- C

Why it’s right: Based on the history and physical exam, this patient has the beginning signs of multiple sclerosis (MS), a
demyelinating neurological process that often presents with vision loss (that can be complete), which recovers
eventually over days to weeks (not always with a complete recovery, and with repeated episodes, permanent visual
damage). The vision loss is optic neuritis, inflammation or demyelination of the optic nerve. If the demyelination affects
eye muscle control, diplopia results like in this patient. The description of the patient’s eye movements is called
internuclear ophthalmoplegia (INO). INO is due to a demyelinating lesion of the medial longitudinal fasciculus in the
brainstem and characteristically causes the above findings with NORMAL convergence. INO is not specific to MS patients
– any lesion like a stroke or tumor in the MLF region can cause INO.

13 ©Test Pirates, LLC. All Rights Reserved.
Step 1 Practice Test #17 Explanations

Take home point: Internuclear ophthalmoplegia is due to a lesion (such as a demyelinating lesion in MS) at the medial
longitudinal fasciculus, which results in the inability to adduct one eye on side gaze but preserved adduction on
convergence testing.

38. 19yo M in MVC. Penetrating wound to right cerebral cortex with paralysis of the left lower extremity, fracture of right
mid humerus with severing of the radial nerve, and a fracture of right tibia. After 10 weeks, DTR strongest in which
locations?
- Left Achilles tendon

Why it’s right: This question is asking the difference between lower and upper motor neuron lesions, and which one
leads to an increased DTR – answer: an upper motor neuron lesion. The wound to the right cerebral cortex that resulted
in left lower extremity paralysis is an upper motor neuron lesion and therefore would lead to the greatest DTR increase.
Conversely, a lower motor lesion like a fracture of the humerus with severing of the radial nerve would cause a lower
motor neuron lesion with decreased or absent DTRs.

Take home point: Upper motor neuron lesions lead to increased DTRs while lower motor neuron lesions lead to
decreased DTRs.

39. 40yo F with 1-year hx of episodes of crampy abdominal pain, intermittent diarrhea, and rectal bleeding with passage
of mucus. BMI 18. Abdominal exam: diffuse tenderness with no rebound. Sigmoidoscopy shows diffuse ulcers. Initial Rx?
- Sulfasalazine

Why it’s right: This patient has the beginning signs and symptoms of ulcerative colitis (UC), which demonstrates a
bimodal distribution in terms of age – it can present around age 20 and around age 40. Of course, the giveaway finding
is the ulcers on sigmoidoscopy; however, crampy, intermittent diarrhea with mucus is highly specific for inflammatory
bowel disease. First-line for UC treatment is sulfasalazine, 6-mercaptopurine, and even infliximab for more severe
presentations. In severe cases of UC (like in this example), it is becoming more common practice to start two
medications that include an ASA preparation like sulfasalazine as well as a biologic like infliximab or adalimumab.
Another key point in treating IBD is that methotrexate is specific to treat Crohn’s diseases while 6-mercaptopurine is
more specific to treat UC.

Take home point: First-line treatment of UC is sulfasalazine. In severe presentations of UC, the addition of a biologic
such as infliximab is appropriate

40. 27yo M for employment exam. No Hx of major illness. Never been sexually active. Minimal contact with parents and
siblings, no hobbies. Doesn't feel depressed. Shrugs in response to congratulations about his new job. Flat affect.
Personality disorder?
- Schizoid

Why it’s right: Social isolation as demonstrated here by minimal contact with family, a flat affect, and no interests in the
absence of depressed feelings is typical of schizoid personality disorder because the social withdrawal is voluntary.
Schizoid patients also have limited emotional expression and are content with social isolation – unlike avoidant
personality type for which patients want social interaction and avoid it because they are afraid of rejection.

14 ©Test Pirates, LLC. All Rights Reserved.


Step 1 Practice Test #17 Explanations

Take home point: Schizoid personality disorder describes a patient who is socially isolated (and content with it) and
expresses little emotion in the absence of depressed feelings.

41. 30yo F has ptosis, ophthalmoplegia, and diplopia. Serum shows autoantibody with affinity for acetylcholine receptor
at the postsynaptic neuromuscular junction. Which neoplasm?
- Thymoma

Why it’s right: The diagnosis is myasthenia gravis (MG) secondary to a thymoma. The symptoms from the thymoma are
the same as MG not caused by a thymoma. MG is paraneoplastic syndrome due to the thymoma. About 15% of patients
with myasthenia have thymomas, so in anyone with a new diagnosis of myasthenia, obtain a chest X-ray (lateral) or CT
to rule out a curable cause of myasthenia – these patients improve in terms of symptoms after thymoma removal.
Interestingly, about half of patients with thymomas develop myasthenia gravis.

Take home point: In all patients newly diagnosed with myasthenia gravis, imaging (lateral X-ray or CT) should be
obtained to rule out a thymoma as a possible cause.

42. Exam scaled so scores are normally distributed with mean of 500 and SD of 100. Which % are between 400 and 600?
- 67%

Why it’s right: There are very few equations you have to memorize for the step 1 exam, and even fewer numbers. The
numbers to know that are 100% high-yield are the percentages that correspond to standard deviations in a normally
distributed dataset (other descriptors for “normal distribution” include Gaussian and bell-shaped). Normal distribution
means that the percentage a standard deviation covers is always fixed. So, memorize: 1 standard deviation (SD) = 68%
around the mean (or 34% more or less than the mean), 2 SDs = 95%, and 3 SDs = 99.7%. This means that if the SD for the
dataset above is 100 with a mean of 500, 1 SD above is 600 and one below is 400, and this range represents 68% of
exam scores. If they asked, what percent is between 500-600, it would be half of 68% or 34%. Even more difficult, if they
asked what percent of scores lie above 700? That is 2 SDs above the mean, which includes half of 95% of the data. So
the remaining is what is above it, or 2.5%. If you don’t understand the numbers, draw the bell curve and write in the
percentages, the math is much simpler in a pictorial version.

Take home point: Memorize: 1 standard deviation (SD) = 68% around the mean (or 34% more or less than the mean), 2
SDs = 95%, and 3 SDs = 99.7%

43. 62yo M with pericardial friction rub 3 days after acute myocardial infarction. Cause of rub?
- Fibrinous pericarditis

Why it’s right: There are only two time periods a patient can develop fibrinous pericarditis after an MI. One is 3 days
after (well, a range of 1-3 days), which is due to acute inflammation, and the other is several weeks after an MI, also
known as Dressler’s syndrome, which is an autoimmune reaction. A friction rub is a tell-tale sign of fibrinous pericarditis
in both cases. Shortly after an MI, neutrophils migrate into the ischemic tissue to clean up. Necrosis leads to an
inflammation that lays down fibrin and dense scar tissue. If this fibrin and scar tissue is extensive enough, fibrinous
pericarditis results. Fluid also accumulates in the form of serous and purulent fluid and may result in a pericardial
effusion.

Take home point: Pericarditis 1-3 days after an MI is due to a fibrinous pericarditis and presents with a friction rub.

15 ©Test Pirates, LLC. All Rights Reserved.


Step 1 Practice Test #17 Explanations

44. 42yo F with 1-mo Hx of abdominal pain, after eating fatty meals. BMI 31. PE shows jaundice and tenderness of RUQ.
Increase of which liver function?
- Cholesterol synthesis

Why it’s right: This patient has cholecystitis, or inflammation of the gallbladder, and in this case is due to gallstones. In
fact, she may also have cholangitis from a gallstone blocking the bile duct based on Charcot’s triad of cholangitis:
jaundice (she has), RUQ pain (she has), and fever. Cholecystitis is not always due to gallstones (it can be due to ischemia
or infection). However, it is most commonly due to gallstones, and this patient’s history fits an at-risk profile for
cholesterol gallstones: she is female, forty, and fat (and likely fertile). Patients do not always fit this profile, however. The
stones are due to fatty food intake, which triggers the synthesis of cholesterol, and an excess of cholesterol forms
stones.

Take home point: The most common cause of cholecystitis is gallstones, and cholesterol stones make up 80% of stones.
Cholesterol stones are present in patients with high cholesterol, most commonly due to poor diet.

45. 70yo M with recent loss of mental function. Hx of weight loss. No drugs. VSS, not dehydrated. Mild anemia. Labs
show: Na 110, Cl 85, K 4.4, BUN 15, Cr 15; Plasma osmolality 250; Urine osmolality 750. Dx?
- Pulmonary neoplasm

Why it’s right: The diagnosis is SIADH (syndrome of inappropriate ADH secretion) as a paraneoplastic syndrome of lung
cancer, specifically small cell carcinoma. Other paraneoplastic syndromes to know associated with small cell include
ACTH production and Lambert-Eaten syndrome. This patient has ADH production in excess, leading to H20 retention by
the kidneys – therefore, plasma osmolality is low because water is being retained and not passed into the urine, so urine
osmolality is high. The serum Na is 110, which is really a measure of total body water! (not of sodium). A very dilute
serum Na means total body water is HIGH. In this patient, he has so much body water and dilution of ions/cations, that
he’s experiencing mental status changes, a common symptom when serum Na <115.

Take home point: A paraneoplastic syndrome due to pulmonary small cell carcinoma is SIADH. Serum Na is a measure of
total body WATER not of sodium.

46. 38yo F with 2-year hx of pain and heavy flow with menses causes her to miss work. Ibuprofen not effective. One 6yo
daughter; unable to conceive another child. PE shows mildly enlarged left ovary and retroflexed uterus. U/S shows
substantial evidence of ovarian cysts and one peritoneal cyst. Bx of cyst will show?
- Proliferative endometrial tissue

Why it’s right: Endometriosis involves a proliferation (hyperplasia) of endometrial tissue not just in the uterus, but in the
abdomen, rectum, ovaries (most common extrauterine site), fallopian tubes, and ligaments of the uterus. The tissue is
non-neoplastic, but can cause debilitating symptoms including very painful menses, painful intercourse, and infertility.
This patient already had one child, but is not unable to conceive after. Endometriosis in this case is from retrograde
menstrual flow because the uterus is retroflexed. The key differentiator between this disease and adenomyosis is that in
endometriosis the uterus is of normal size, while in adenomyosis it is enlarged.

Take home point: In endometriosis, proliferation of endometrial tissue is not always restricted to the uterus – it can
occur in the abdomen, ovaries (most common extrauterine site), and the rectum.

16 ©Test Pirates, LLC. All Rights Reserved.


Step 1 Practice Test #17 Explanations

47. 40yo M skin extremely sensitive to sunlight, which causes formation of vesicles and blisters on the skin which take
weeks to heal. Diagnosed with disorder caused by increased synthesis of compounds in the skin that are subject to
excitation by visible light. Which biochemical pathway is defective?
- Heme synthesis

Why it’s right: The diagnosis is porphyria cutanea tarda (PCT), due to a defect involved in heme synthesis – specifically
uroporphyrinogen decarboxylase. The accumulated substrate is uroporphyrin, which results in tea-colored urine. This is
in contrast to the port wine-colored urine of acute intermittent porphyria. PCT patients develop painful skin blisters
when exposed to sunlight. PCT is also the most common porphyria. These patients also have elevated LFTs,
hypertrichosis (increased hair growth over body), and facial hyperpigmentation.

Take home point: Porphyria cutanea tarda results in tea-colored urine and skin blisters on sun exposure. It is due to a
defect in heme synthesis (uroporphyrinogen decarboxylase).

48. 60yo M with 6-month hx of fatigue. Four years ago, had subtotal gastrectomy after gunshot wound. Drinks 6-8 beers
daily. PE shows paresthesias of both hands. Labs show: Hb 8, HCT 24%, MCV 115, WBC 5k, Platelets 165k, RBC Folic acid
500 (N = 125-600), B12 10 (N = 160-195). Cause is absence of?
- Parietal cells

Why it’s right: This patient is vitamin B12 deficient due to pernicious anemia, which is lack of intrinsic factor resulting in
anemia. Intrinsic factor is secreted by parietal cells of the stomach – this patient lost a majority of these when he
underwent a gastrectomy. Intrinsic factor is needed to absorb B12 in the terminal ileum. He is symptomatic with
paresthesia, low H&H, and labs are also given showing normal folic acid (commonly confused with B12 deficiency) and
low B12. Both B12 and folic acid deficiency result in a macrocytic (elevated MCV >100) anemia.

Take home point: Pernicious anemia is due to parietal cell loss that results in low intrinsic factor required for vitamin
B12 absorption. It can occur after a complete or partial gastrectomy.

49. RCT to compare wound healing and cosmetic differences between two surgical procedures for closing skin after C
section. Surgical specialists cannot be blinded to the procedure. Results show procedure A has a lower rate of wound
infection compared with B (relative risk 0.66, 95% CI 0.3 - 1.45). Which accurately represents comparison between A and
B?
- Neither procedure is superior

Why it’s right: Whenever the confidence interval for an odds ratio or relative risk crosses 1.0, it is not significant. This is
because it is a RATIO – one outcome compared to another. And if the outcomes are equal (neither is superior or
inferior), the RR or OR is 1.0. Though the relative risk is <1.0 implying less wound healing in A compared to B, the
confidence interval crosses 1.0, meaning that the range of values that we are 95% confident of includes 1.0, equating
with an insignificant result.

Take home point: When the confidence interval for OR or RR includes 1.0, the results are not significant.

50. 40yo M with interstitial pulmonary fibrosis has greater maximal expiratory flow rate than predicted. Which best
explains this?
- Increased radial traction on airways
17 ©Test Pirates, LLC. All Rights Reserved.
Step 1 Practice Test #17 Explanations

Why it’s right: Fibrosis and obstructive lung disorders should be thought of as opposites: one leads to increased traction
on airways and the other decreased. Fibrosis creates scar tissue between airways, increasing traction and allowing
alveoli, bronchioles, and bronchi to remain open longer. This results, when breathing air out, in an increased rate of
expiratory flow because the airways are held open longer before they collapse. Conversely, in obstructive lung diseases
like COPD and emphysema, there is loss of elastic tissue à loss of traction à airways collapse faster on expiration à
decreased expiratory flow rate and increased residual volume or air trapping.

Take home point: Fibrosis leads to increased expiratory flow rate due to increased radiation traction on alveoli,
bronchioles, and bronchi.




18 ©Test Pirates, LLC. All Rights Reserved.

You might also like